X



トップページ数学
1002コメント517KB
不等式への招待 第9章 [無断転載禁止]©2ch.net
レス数が1000を超えています。これ以上書き込みはできません。
0448132人目の素数さん
垢版 |
2018/02/18(日) 19:05:09.70ID:eirA2Wob
n∈N、r∈R、r≧1 に対して、
{(n+1)^(r+1)*n^r}/{(n+1)^(r+1) - n^(r+1)} ≧ Σ[k=1 to n] k^r ≧{(n+1)^r*n^(r+1)}/{(n+1)^(r+1) - n^(r+1)}

油断、怠慢、即ち怠惰!
0449132人目の素数さん
垢版 |
2018/02/24(土) 01:29:07.74ID:j7T05hBb
(1) a>b>c> 0 のとき、ax^2+bx+c=0 の解αは、|α|<1 をみたすことを示せ。
(2) x^3 + 3x^2 + 5x + 7 = 0 の解αは、1<|α|≦3 をみたすことを示せ。

あぁぁ、脳が…震える…
                     -‐. . : ヘ三≧-_
                  <: : /: : : : : : : ̄<三≧
               /: : : : : /: : : : : : :/: : : : : : <≧
              /: : : : : : / : : : : : :./: : : : : : : : : : : :<=
             /l:冓: : : :/:/: : : : : /: : : : : : : : | : : i: : : \
           /´/ ´ 刃: :i :i :{ : : : : :/ : : : : : : : : :| : : | : : : : :.i
           / /イ´./: :l  ‐ 、 :i: :i : : : :i.: : : :.:. | : : |: : : : : :ヘ
          レ / .// ii|  ● i:::::i : : : .::: : : : : :i : : i: : :i : : : :i
         i´   ´ / .   ヾ、. _ ノ::::..::. : .::::: : : : : :i : : i: : :i : : : :|
三=- _____.|     |.\.    ̄   /.,::::::  ‐ 、: :/: : /: : :i.: : : : |
三=---- 三三/    .}  冫  ─i´ ,,,. ´  |  ● i::: : :, :_., : |: : : :.|
´   /  i /i.    ./ / ヘ   ヾ丶 i  ヾ、. _ ノ /4/‖:::| : : : |
   冫"~  ̄     / / i .  丶   ̄i    ̄  ./ i i .|}::/: : : :/
  /        イ ´ / `メ ノ \   \     |/ | |.|.: : : :./
 |    ==   /| i  ` <丿メノ ` う   ア ´  ///: : / \     ___
  ┐  __  −=三〈 `ー-\ \ ノメ//''"      .//  ̄ _ - = 三三三三
  i-'".|三三三三=丶- 、  \ `、ノ//     ,.-'''"  -=三三三三三三=
  /  /三三三三三=┐ 丶  - ´, /     / -=三三三三三三三=
. 〈  〈 三三三三三三 .| | └‐- ´i/     /=三三三三三三三三 〈
0450132人目の素数さん
垢版 |
2018/02/24(土) 04:17:06.67ID:xZc/HRib
>>449
(1)
・解が実数のとき(bb-4ac≧0)
 |x|≧1 ⇒ axx+bx+c ≧ a|x| -b|x| +c =(a-b)|x|+ c > c > 0
 ∴|α|< 1
・解が共軛複素数のとき(bb-4ac<0)
 |α| = √(αα~)= √(c/a)< 1,

(2)
x≧-2 ⇒ x^3 +3x^2 +5x +7 =(x+2)(xx+x+3)+ 1 ≧ 1,
x≦-3 ⇒ x^3 +3x^2 +5x +7 =(x+3)(xx+5)-8 ≦ -8
中間値の定理より(-3,-2)に実解rがある。

x^3 +3x^2 +5x +7 =(x-r){xx +(3+r)x - 7/r},
xx +(3+r)x - 7/r = 0 は複素数解αをもつ。
解と係数の関係から rαα~ = -7,
|α|= √(αα~) =√(-7/r),
∴ √(7/3)< |α| < 2,

・蛇足
r = -1 +(1/3){6(-9+√87)}^(1/3)-(1/3){6(9+√87)}^(1/3)
 = -2.1795090246
|α|= 1.79213072
0452132人目の素数さん
垢版 |
2018/02/24(土) 18:59:44.58ID:xZc/HRib
>>436

s = 5/4 とおく。
133 > 131.25 = 84 ss,
110 > 105 = 84 s,
84 = 84,
n^5 > 84^5 *(s^10 + s^5 +1)
= 84^5 *{(5/4)^10 +(5/4)^5 + 1}
> 84^5 *{9 + 3 + 1}
= 84^5 * 13
>(84 * 5/3)^5
= 140^5,

∴ 141 ≦ n ≦ 147   (>>440-441)
0453132人目の素数さん
垢版 |
2018/02/25(日) 02:17:24.53ID:gGaVEUAO
>>449
(1)は新潟大学ですね。

>>451
〔掛谷の定理〕
F(x)= a_0 x^n + a_1 x^{n-1} + …… + a_{n-1}x + a_0,
 a_0 > a_1 > a_2 > …… > a_{n-1}> a_n > 0
ならば、F(x)= 0 の解の絶対値は1より小さい。

(略証)
F(0)= a_n >0,F(1)> 0,
(1 - 1/x)F(x)/ x^n = a_0 - Σ[j=1,n](a_{j-1} - a_j)/x^j - a_n / x^(n+1),
x=1 のときは
 0 = a_0 - Σ[j=1,n](a_{j-1} - a_j)- a_n,
辺々引いて
(1 - 1/x)F(x) / x^n = Σ[j=1,n+1](a_{j-1} - a_j)(1 - 1/x^j) + a_n(1 - 1/x^{n+1}),
ここで、|x|≧1, x≠1 ならば
Re{1/x^j}≦|1/x|^j ≦ 1,
であるが、等号成立は x=1 に限るので
Re{1 - 1/x^j}> 0,
∴ Re{右辺}> 0,
∴ F(x)≠ 0,
∴|α|<1
http://www004.upp.so-net.ne.jp/s_honma/solution/solution.htm

「經濟研究」の別証明は、あまりにも迂回的で逆行的でござるな。
市大とちゃんと統合成立するかなぁ?
0454132人目の素数さん
垢版 |
2018/02/25(日) 02:38:57.60ID:gGaVEUAO
>>453

などと嘯いてたら、間違えてしまった......orz

F(x)= a_0 x^n + a_1 x^{n-1} + …… + a_{n-1}x + a_n,
0455132人目の素数さん
垢版 |
2018/02/25(日) 10:26:23.15ID:utpborzn
>>453-454
あぁぁ…、あなたはなんと勤勉なる事か!
偶々ネットで見かけた掛谷のpdfを見て出題したのでござるが、入試問題まで探してくるとはとはとはとは…!
0459132人目の素数さん
垢版 |
2018/02/25(日) 16:08:11.57ID:gGaVEUAO
逆数バージョン

〔掛谷の定理〕 
正係数のn次多項式
 F(x)= a_0 x^n + a_1 x^{n-1} + …… + a_{n-1}x + a_n,
 0 < a_0 < a_1 < a_2 < …… < a_{n-1}< a_n
について、F(x)= 0 の解の絶対値は1より大きい。

(略証)
F(0)= a_n >0,F(1)> 0,
(x-1)F(x)= a_0 x^(n+1)+ Σ[j=1,n](a_{n-j+1}- a_{n-j}) x^j - a_n,
x=1 のときは
 0 = a_0 + Σ[j=1,n](a_{n-j+1}- a_{n-j})- a_n,
辺々引いて
(x-1)F(x)= a_0(x^{n+1} -1) + Σ[j=1,n](a_{n-j+1}- a_{n-j})(x^j -1),
ここで、|x|≦ 1, x≠1 ならば
Re{x^j}≦|x|^j ≦ 1,
であるが、等号成立は x=1 に限るので
Re{x^j -1}< 0,
∴ Re{右辺}< 0,
∴ F(x)≠ 0,
∴|α|> 1

(系) x → 1/x とすれば >>453
http://www004.upp.so-net.ne.jp/s_honma/solution/solution.htm
0460132人目の素数さん
垢版 |
2018/02/26(月) 05:41:30.17ID:+hLINPlt
>>453 >>459

(1)
n次方程式 F(x)= 0 の根{r_k}がすべて実数のとき、
 F(x)が極値・停留値をとる点b{F '(x)= 0 の実数解}は次をみたす。
 r_min ≦ b ≦ r_max

(2)
n次多項式 F(x)が停留値をとる点β{F '(x)= 0 の解}は、
 F(x)= 0 のすべての根を含む凸領域内にある。

例) すべて単根{α_k}のときは
 β = Σ[k=1,n]t_k α_k
 重み t_k = |β-α_k|^(-2)/{Σ[k=1,n]|β-α_k|^(-2)}
0462132人目の素数さん
垢版 |
2018/02/28(水) 05:08:58.38ID:W7HTMDJw
>>448

r=1 のときは等号になるので r>1 とする。中辺を

S(n)= Σ[k=1,n] k^r

とおく。問題の式は

1/n > S(n)/n^(r+1)- S(n)/(n+1)^(r+1)> 1/(n+1),

S(n)/S(n-1)>{(n+1)/n}^(r+1)> S(n+1)/S(n),

S(n)/(n+1)^(r+1)> S(n-1)/n^(r+1) …… 増加列
S(n)/n^(r+1)> S(n+1)/(n+1)^(r+1) …… 減少列

{1/(n+1)}Σ[k=1,n]{k/(n+1)}^r >(1/n)Σ[k=1,n-1](k/n)^r
(1/n)Σ[k=0,n](k/n)^r > {1/(n+1)}Σ[k=0,n+1]{k/(n+1)}^r

となる。
0463132人目の素数さん
垢版 |
2018/02/28(水) 05:42:35.66ID:W7HTMDJw
>>448 (続き)

f(x)= x^r (r≧1)は下に凸だから、下の補題より

S(n)/{n(n+1)^r}≧ S(n-1)/{(n-1)n^r},

S(n)/{(n+1)n^r}≧ S(n+1)/{(n+2)(n+1)^r},

これと n/(n-1)>(n+1)/n >(n+2)/(n+1) から >>462 が出る。

なお、n >> r では S(n)〜{1/(r+1)}(n + 1/2)^(r+1)


〔補題〕
f(x)が 0<x<1 で下に凸ならば

1)(1/n)Σ[k=1,n]f(k/(n+1))≧{1/(n-1)}Σ[k=1,n-1]f(k/n),

2){1/(n+1)}Σ[k=0,n]f(k/n)≧{1/(n+2)}Σ[k=0,n+1]f(k/(n+1)),

(略証)
1)
凸性からJensenにより
{(n-k)/n}f(k/(n+1))+(k/n)f((k+1)/(n+1))≧ f(k/n),
k=1 から k=n-1 まで加えて(n-1)で割る。
2)
凸性からJensenにより
{k/(n+1)}f((k-1)/n)+{(n+1-k)/(n+1)}f(k/n)≧ f(k/(n+1)),
k=0 から k=n+1 まで加えて(n+2)で割る。
0466132人目の素数さん
垢版 |
2018/03/02(金) 05:19:49.90ID:IyAEI4K5
〔Popoviciuの不等式〕
f(x) が下に凸ならば、 (a+b+c)/3 = m に対して
f(a) + f(b) + f(c) + 3f(m) ≧ 2f((a+b)/2) + 2f(a+c)/2) + 2f((b+c)/2),

(略証)
a≦b≦c としてよい。
(i) a,b ≦ m ≦ c のとき
f(a) + f(b) ≧ 2f((a+b)/2),
f(m) + f(c) ≧ 2f((m+c)/2),
2f(m) + 2f((m+c)/2) ≧ 2f((a+c)/2) + 2f((b+c)/2),
辺々たす。

(ii) a ≦ m ≦ b,c のとき
f(a) + f(m) ≧ 2f((a+m)/2)
2f((a+m)/2) + 2f(m) ≧ 2f((a+b)/2) + 2f((a+c)/2),
f(b) + f(c) ≧ 2f((b+c)/2),
辺々たす。

文献[9]佐藤淳郎(訳)p.41 演習問題1.89
0469132人目の素数さん
垢版 |
2018/03/03(土) 02:38:06.57ID:MMc1xkls
a、b、c ∈ (1、∞) または、a、b、c ∈ (0、1) のとき、
log_a(bc) + log_b(ca) + log_c(ab) ≧ 4{ log_(ab)c + log_(bc)a + log_(ca)b }

(参考)
過去スレに、a、b、c ∈ (1、∞) のとき、
 左辺 > 定数
 定数 > 右辺 > 定数
というのを収集して貼ったような希ガス、ハロゲンガス…。


詳細は…不明ですか? あなた、怠惰…ですね
           ____________
        >/////////::::::丶
      //////////:::::::::::::::::::::\
      ////////,/:::::::::\:::::::::::::::::::丶
     .//////, イ::::::::::::::::::::::::::ヽ::::::::::::::::::\
    ////::::::::::::\:::::\::::::::::::::::::::::::::::::::::::::`i
    //:::::::::::::::::::::::::\:::::\:::::::::::::::::::::::::::::::::::i      , __
   /::::i:::::::::ヾi:::::::::::::::::::::::::::::\:::::::::::: rヘ:::::::::ト    /  !
   |:::::|::i::::::::::リ:::::::::/:::::,:::f´⌒ヽ .V:::::',::ヾi:::i::::! /      !
   i:::::|:::|:::::::/:i:i::`:::,:イ≡i ゚゙● i .V::::::'::::y:ィリ ̄       !
   .|::::::::|::::::::::::j,:イ≡彡 `==彳 ,, V::::::':::/ ヽ///,i      /
    |::::::::ト::::イi´●゙    ー ´ 7 Y:::::::∧ 人///     /----y
    i::::::::::::::::::弋゙_丿',      / ;} i:::::i  ゞ  /     /ヽ  /
    i:::::::::::\:::∧   '- ´  /ヽイ;;;} !::/i  ',./i     /ソ  ノ
    \::::ヽ:::::::└-i- ム-<丶tt i };} レ丶  ./ /    /::/ /\
      \::::::::/   `\\,エィ ´ ;;  / ´/ ./    ‖/     .\
       ヽ::/     !/,`丶;   ; ;イ ノ /! /     ! j       ヾ
        `ト     ヽ///!ー イ ´//// ソ               i
          \   \//////////./                i
            \   .\///////                  |
0470132人目の素数さん
垢版 |
2018/03/03(土) 07:13:06.83ID:hUH2ieOC
>>469

A = ln(a),B = ln(b),C = ln(c) とおく。
題意により、A,B,Cは同符号。
正であるとしても一般性を失わない。

S=A+B+C,T=AB+BC+CA,U=ABC とおく。
(左辺) = (B+C)/A + (C+A)/B + (A+B)/C = (ST-3U)/U,
(右辺) = 4{C/(A+B) + A/(B+C) + B/(C+A)} = 4{S(SS-2T) + 3U}/(ST-U),
(左辺) - (右辺) = {(ST-9U)T + 3SU・F_{-1}) + SSU・F_{-2} }/{T(ST-U)} ≧ 0,
ここに
U・F_{-1} = TT -3SU ≧ 0,
UU・F_{-2} = T^3 -4STU +9UU ≧ 0,

なお、
(右辺) ≧ 6 (Nesbitt、Shapiro-3)
0471132人目の素数さん
垢版 |
2018/03/03(土) 07:18:06.63ID:hUH2ieOC
>>466

f(m) + f((m+c)/2) ≧ f((a+c)/2) + f((b+c)/2),
f(m) + f((a+m)/2) ≧ f((a+b)/2) + f((a+c)/2),
のところが分からん?

〔補題〕
f(x) は m,nを含む区間で下に凸
m+d,n-d が mとnの中間にあるとき
 f(m) + f(n) > f(m+d) + f(n-d)

(略証)
m≠n、0<λ<1 に対して
(1-λ)f(m) + λf(n) > f((1-λ)m + λn) = f(m+d)
 λf(m) +(1-λ)f(n) > f(λm + (1-λ)n) = f(n-d)
辺々たす。
ここに、d = λ(n-m) とおいた。
0472132人目の素数さん
垢版 |
2018/03/03(土) 19:46:02.49ID:hUH2ieOC
>>469

〔補題〕
A,B,C が同符号のとき
(B+C)/A + (C+A)/B + (A+B)/C ≧ 4{A/(B+C) + B/(C+A) + C/(A+B)},

(略証)
AM-HM より
 A(1/B + 1/C)≧ 4A/(B+C),
 B(1/C + 1/A) ≧ 4B/(C+A),
 C(1/A + 1/B) ≧ 4C/(A+B),
辺々たす。
0473132人目の素数さん
垢版 |
2018/03/04(日) 03:52:33.46ID:IhTCj0CK
>>470

〔Nesbitt、Shapiro-3〕

A/(B+C) + B/(C+A) + C/(A+B) ≧ 3/2,

(左辺) = (A+B+C) {1/(B+C) + 1/(C+A) + 1/(A+B)} - 3
≧ (A+B+C) * 9/ {2(A+B+C)} - 3  (← AM-HM)
= 9/2 - 3
= 3/2.

(左辺) = (A+B+C) {1/(B+C) + 1/(C+A) + 1/(A+B)} - 3
= (1/2) {(B+C)+(C+A)+(A+B)} {1/(B+C) + 1/(C+A) + 1/(A+B)} - 3
≧ (1/2)(1+1+1)^2 - 3  (← コーシー)
= 9/2 - 3
= 3/2.
0474132人目の素数さん
垢版 |
2018/03/13(火) 11:09:49.42ID:IdxYrbr8
C[n,r]は二項係数とする。

(1) n ∈N (n≧2) に対して、2^{2n-1}/\sqrt(n) < C[2n, n] < 2^{2n-1} を示せ。

(2) n+1 以上 2n-1 以下の素数の積は、2^{2n-2} より小さいことを示せ。
  ただし、該当する素数がないときは、積を1とする。

(3) n 以下の素数の積は、2^{2n-1} 以下であることを示せ。
0476132人目の素数さん
垢版 |
2018/03/14(水) 03:30:09.01ID:hdbbxtzk
>>474
(1)
2^(2n-1) /√n < C[2n,n] < 2^(2n-1) √{2/(n+1)},

nについて帰納法による。
n=2 のとき、8/√2 < C[4,2] < 8√(2/3) ゆえ成立。
n-1 について成り立つならば
 2^(2n-3)/√(n-1) < C[2n-2,n-1] < 2^(2n-3)√(2/n),
 4√{(n-1)/n} < 4 (2n-1)/2n < 4√{n/(n+1)},
 辺々かけて
 2^(2n-1)/√n < C[2n,n] = 2^(2n-1)√{2/(n+1)},
∴ n についても成立。
0477132人目の素数さん
垢版 |
2018/03/14(水) 03:46:47.54ID:KXfS4lra
>>475
いつもながら実に実に実に〜ぃ、素晴らしいデス!
参考資料まで探して頂き、感謝の極みでござるぞ!
0479132人目の素数さん
垢版 |
2018/03/15(木) 16:54:44.46ID:HCGObM3W
>>476

Stirlingの近似
 n!≒ √(2π)n^(n+1/2)e^{-n + 1/(12n)},
から
 C[2n,n]=(2n)!/(n!・n!)≒(4^n)/√(πn)・e^{-1/(8n)},
0483132人目の素数さん
垢版 |
2018/03/21(水) 04:54:09.91ID:Y0EoMfqc
>>480

左側:
 y=√x は上に凸だから、接線が上。
 √k > ∫[k-1/2,k+1/2]√x dx =(2/3){(k+1/2)^(3/2)-(k-1/2)^(3/2)},
右側:
 y=√x は上に凸だから、割線が下。
 {√k + √(k+1)}/2 < ∫[k,k+1] √x dx =(2/3){(k+1)^(3/2)- k^(3/2)},
 ただし、1≦k≦4 は別途たす。
0484132人目の素数さん
垢版 |
2018/03/21(水) 04:59:29.95ID:Y0EoMfqc
〔応用問題〕
 {√k + √(k+1)}/2 < ∫[k,k+1] √x dx =(2/3){(k+1)^(3/2)- k^(3/2)},
を用いて次を示せ。

(2) √2 < 99/70 = 1.41428571…    (k=8)
   √2 > 1393/985 = 1.41421320… (k=49)
   √2 < (19601/6)/2310 = 1.4142135642… (k=288)

(3) √3 < (1351/6)/130 = 1.73205128… (k=48)

(5) √5 < 2889/1292 = 2.236068111…  (k=80)

(6) √6 < (485/6)/33 = 2.4494949…  (k=24)

(7) √7 < 2024/765 = 2.645751634…  (k=63)

(10) √10 > 117/37 = 3.16216216…   (k=9)
   √10 < (27379/6)/1443 = 3.1622776622… (k=360)

(11) √11 < 3970/1197 = 3.316624895… (k=99)

(17) √17 > 268/65 = 4.123076923…   (k=16)

(37) √37 > 882/145 = 6.08275862…  (k=36)

面白スレ26 - 109〜110,117
0485132人目の素数さん
垢版 |
2018/03/21(水) 05:17:17.36ID:Y0EoMfqc
>>484 に追加

(15) √15 < 244/63 = 3.87301587… (k=15)

(35) √35 < 846/143 = 5.9160839…  (k=35)

(101) √101 > 4030/401 = 10.049875311… (k=100)
0486132人目の素数さん
垢版 |
2018/03/23(金) 01:31:09.69ID:fa7L1pkR
>>428
分子を変えた場合に最大最小値はどうなるのか気になるでござる ( ゚∀゚) ウヒョッ!

a、b、c、p、q >0 に対して、
(1) a/(pa+qb) + b/(pb+qc) + c/(pc+qa)
(2) b/(pa+qb) + c/(pb+qc) + a/(pc+qa)
(3) c/(pa+qb) + a/(pb+qc) + b/(pc+qa) ≧ 3/(p+q)
0487132人目の素数さん
垢版 |
2018/03/23(金) 07:43:00.89ID:EuazrwzR
>>486
(3)
c(pa+qb)+ a(pb+qc)+ b(pc+qa)=(p+q)(ab+bc+ca),
コーシーにより
(左辺)≧(a+b+c)^2 /{(p+q)(ab+bc+ca)}≧ 3/(p+q),
でござるか。
0488132人目の素数さん
垢版 |
2018/03/23(金) 07:49:46.24ID:EuazrwzR
>>483
左側:
積分計算を避けるなら、
AM-GM より
(kk -1/4)^3 ≧(kk)(kk -3/8)(kk -3/8),

{(k+1/2)^(3/2) - (k-1/2)^(3/2)}^2 = 2k(kk +3/4) -2(kk -1/4)^(3/2)
 ≦ 2k(kk +3/4) -2k(kk -3/8)
 = 9k/4,

√k ≧ (2/3){(k+1/2)^(3/2) - (k-1/2)^(3/2)},
以下は同様。
0489132人目の素数さん
垢版 |
2018/03/24(土) 07:45:51.61ID:Q7DsXoRu
>>483
右側:
{√(k+1)- √k}^2 = 1/{√(k+1)+ √k}^2 ≧ 1/{2(k+1)+ 2k}= 1/{2(2k+1)},
より
(右辺)^2 -(左辺)^2 =(4/9){(k+1)^(3/2)- k^(3/2)}^2 -(1/4){√k + √(k+1)}^2
 =(1/36)[{4(2k+1)^2 +5}{√(k+1)- √k}^2 - 2(2k+1)]
 ≧(1/36)[{4(2k+1)^2 +5}/{2(2k+1)}- 2(2k+1)]
 =(5/36)/{2(2k+1)},

{√k + √(k+1)}/2 <(2/3){(k+1)^(3/2)- k^(3/2)},
以下は同様。
0490132人目の素数さん
垢版 |
2018/03/24(土) 19:30:42.65ID:UFxacGAG
>>486
(1) 3/p > a/(pa+qb) + b/(pb+qc) + c/(pc+qa) > 0
(2) 3/q > b/(pa+qb) + c/(pb+qc) + a/(pc+qa) > 0

( ゚∀゚) プゥ
ノヽノ) =3'A`)ノ ヒャー
  くく へヘノ
0492132人目の素数さん
垢版 |
2018/03/27(火) 07:17:22.34ID:H3+XdNyv
>>486

(1) Max{2/p,3/(p+q)} > a/(pa+qb) + b/(pb+qc) + c/(pc+qa) > min{3/(p+q),1/p},

(2) Max{2/q,3/(p+q)} > b/(pa+qb) + c/(pb+qc) + a/(pc+qa) > min{3/(p+q),1/q},

(3) c/(pa+qb) + a/(pb+qc) + b/(pc+qa) ≧ 3/(p+q),   >>487
0494132人目の素数さん
垢版 |
2018/03/29(木) 07:31:07.02ID:9DT8+Pw9
>>492
ど、どう証明するのかな? ・・・・・・・・ゴクリ。

    ヽ|/
   / ̄ ̄ ̄\
   /      ヽ
  / \ /  |
  | (●)(●)|‖|
  | / ̄⌒ ̄ヽ U|
  ||i二二ヽ| |
  |U\___ノ |
  |       |
0495132人目の素数さん
垢版 |
2018/03/30(金) 06:53:28.40ID:yIR/xm3k
>>474
(1)
2^(2n-1) /√n < C[2n,n] < 2^(2n) /√(πn),

大関:参考書[3]、p.53 例題10 (1987)
W.F.Sierpinski: "Elementary theory of numbers",PWN-Polish Sci. Publ. (1964)
0496132人目の素数さん
垢版 |
2018/04/02(月) 10:26:49.91ID:qYYBXa/1
〔問題983〕
実数 0 < x < π/6 に対して、 不等式
 sin(x) < 2x/(x +π/2)
を示せ。

分かスレ441-983、分かスレ442-10,28,47
0497132人目の素数さん
垢版 |
2018/04/02(月) 11:35:20.76ID:QTiBZAaI
>>496
問題だけじゃなく、証明も貼っておこうぜ! ここは不等式のコレクターのためのスレなんだからな。

> y=π/2 で成り立てば、
>  2{1-sin(x)}/(π/2 -x) > sin(x)/x,    >>28
>  x/sin(x) > (π/2 +x)/2,
> ならば十分。そこで
>  g(x) = x/sin(x),
> とおく。
> |x|<π/2 で g(x) は下に凸。    … (*)
> g(π/6)=π/3 と g(π/2)=π/2 を通る割線を曳く。
>  z = (π/2 +x)/2,
> -π/2 < x < π/6 のとき g(x) > (π/2 +x)/2,
>
> (*)
> 1-cos(x) ≧ 0,
> x-sin(x) = ∫[0,x] {1-cos(t)} dt > 0 (x>0)
> sin(x)-x・cos(x) = ∫[0,x] t・sin(t) dt > 0 (0<x<4.4934094579)
> より
> g '(x) = {sin(x)-x・cos(x)}/sin(x)^2,
> g "(x) = {1-cos(x)}/sin(x)・g '(x) + {1+cos(x)}{x-sin(x)}/sin(x)^3 > 0,
0498132人目の素数さん
垢版 |
2018/04/03(火) 14:36:03.43ID:RjUb/qt3
〔問題〕
(1)
f(x)g(x) = 1ならば
 f '(x)g '(x) < 0,
さらに f(x)f "(x) < 0 のとき
 f "(x)g "(x) < 0,

(2)
 g(x) = x/sin(x) について、
 |x| < 2.081575977818 ⇒ g "(x) > 0,

分かスレ442-069
0500132人目の素数さん
垢版 |
2018/04/03(火) 22:49:34.80ID:RjUb/qt3
>>499

(1) そうです。(微分可能な…)

(2)
|x|< 2.0815759778181 ⇒ {sin(x)/x}" < 0
を使っていいらしい。
0512132人目の素数さん
垢版 |
2018/04/12(木) 10:53:44.62ID:8y+syOJY
非負実数 a_1、…、a_n に対して、
(Σ[k=1 to n] a_k){Σ[k=1 to n] (a_k)^(n-1)} ≦ n*Π[k=1 to n]a_k + (n-1)*Σ[k=1 to n] (a_k)^n

昔の手書きメモから発掘、詳細不明 ( ゚∀゚) ウヒョッ
0513132人目の素数さん
垢版 |
2018/04/14(土) 00:31:34.32ID:Rl6BZiHz
>>512

兩n = (右辺) - (左辺)
= (n-1)Σ[k=1,n] (a_k)^n - {Σ[k=1,n] a_k}{Σ[k=1,n] (a_k)^(n-1)} + n・a_1・a_2…a_n,
a_1 = a,a_2 = b,a_3 = c,a_4 = d,a_5 = e,
とおいてシューア展開すると、

兩1(a) = 0,
兩2(a,b) = 0,
兩3(a,b,c) = F_1(a,b,c)
兩4(a,b,c,d) = (2/3){F_2(a,b,c) + F_2(b,c,d) + F_2(c,d,a) + F_2(d,a,b)} + (1/3){F_1(a,b,c)d + F_1(b,c,d)a + F_1(c,d,a)b + F_1(d,a,b)c},
兩5(a,b,c,d,e) = (1/2)Σ[a,b,c] F_3(a,b,c) + (1/6)Σ[a,b,c] F_2(a,b,c)(d+e) + (1/6)Σ[a,b,c] F_1(a,b,c)de,

ここに Σ[a,b,c] は C[5,3] = 10項の和
0514132人目の素数さん
垢版 |
2018/04/14(土) 00:53:24.57ID:Rl6BZiHz
>>513

〔Schurの不等式〕
F_m(x,y,z) = (x^m)(x-y)(x-z) + (y^m)(y-z)(y-x) + (z^m)(z-x)(z-y) ≧ 0,

文献[3] 大関(1987) p.28
文献[8] 安藤(2012) p.27〜28
文献[9] 佐藤(訳)(2013) p.40
0515132人目の素数さん
垢版 |
2018/04/15(日) 23:04:58.95ID:gKDKhRE9
a, b, c >0 かつ abc=1 のとき、 (a^10 + b^10 + c^10)^2 ≧ 3*(a^13 + b^13 + c^13).



   ┏  ━ゝヽ''人∧━∧从━〆A!゚━━┓。
 < ゝ\',冫。’  ,,,,  ∧,,∧ ' ゛△´ ' ゝ'┃
∇  ┠─Σ┼  ,ニ,◎、・ω・') 冫/ そ', .┨'゚,。
.。冫▽ < 冫 r'/ミ/〉⊂ノ  乙 ≧  ▽
 。 ┃   Σ. 〈/")、〉ノノ  、'’ ≦ │て く
   ┠─ム┼ (_/_iiiノ     、,,’.┼ ァ Ζ┨ ミo'’`
.。○.〆  `、,~´+ ! .!  √ ▽ ',! ヽ.◇ o.┃
   ┗〆━┷. Z,..`"┷━''o.ヾo┷+\━┛,゛;
0516132人目の素数さん
垢版 |
2018/04/16(月) 12:01:31.88ID:I9VNB52o
兩n = Σ[m=1,n-2] 2(n-m-2)! m!/(n-1)! Σ[a,b,c] F_m(a,b,c) S_{n-m-2}(A-{a,b,c}) ≧0,

 A = {a_1,a_2,…,a_n}
 納a,b,c] は C[n,3] = n(n-1)(n-2)/6 項の和。
 S_j(B) は集合Bの要素で作ったj次の基本対称式
 A = {a_1,a_2,…,a_n}

ついで乍ら
納a,b,c] F_m(a,b,c) = {m(m+1)2}Σ[k=1,n] (a_k)^(m+2) S_{n-m-2}(A-{a_k})
  - m(n-m-1)納k=1,n] (a_k)^(m+1) S_{n-m-1}(A-{a_k})
  + {(n-m)(n-m-1)/2}納k=1,n] (a_k)^m S_{n-m}(A-{a_k})
0517132人目の素数さん
垢版 |
2018/04/16(月) 12:04:49.28ID:I9VNB52o
>>513

兩n = Σ[m=1,n-2] 2(n-m-2)! m!/(n-1)! Σ[a,b,c] F_m(a,b,c) S_{n-m-2}(A-{a,b,c}) ≧0,

 A = {a_1,a_2,…,a_n}
 Σ[a,b,c] は C[n,3] = n(n-1)(n-2)/6 項の和。
 S_j(B) は集合Bの要素で作ったj次の基本対称式
 A = {a_1,a_2,…,a_n}

ついで乍ら
Σ[a,b,c] F_m(a,b,c) = {m(m+1)2}Σ[k=1,n] (a_k)^(m+2) S_{n-m-2}(A-{a_k})
  - m(n-m-1)Σ[k=1,n] (a_k)^(m+1) S_{n-m-1}(A-{a_k})
  + {(n-m)(n-m-1)/2}Σ[k=1,n] (a_k)^m S_{n-m}(A-{a_k})
0518132人目の素数さん
垢版 |
2018/04/17(火) 03:17:29.93ID:+pEnOXwO
(1/2)*(3/4)*…*(999999/1000000) < 1/1000 を示せ。


  ∧_∧ 
 ( ´・ω・)  先月の数蝉に不等式の問題があったような…
 (つ旦と) 
  と_)_)
0519132人目の素数さん
垢版 |
2018/04/17(火) 22:54:13.31ID:PQyFkARt
>>518

√((2k-1)(2k+1)) = √(4kk-1) < 2k,

(左辺) = {√(1・3)/2}{√(3・5)/4}…{√((2n-1)(2n+1))/(2n)} / √(2n+1)
  < 1/√(2n+1)
  < 1/√(2n)
  = 0.001
 
(別法)
 Stirling の公式から

(左辺) = (2n-1)!! / (2n)!!
 = (2n-1)!! / {(2^n) n!}
 = (1/4)^n・C(2n,n)
 = 1/√(nπ)・{1 - 1/(8n) + 1/(128n^2) + 5/(1024n^3) - …… }
 < 1/√(nπ)
 = 1/√(500000π)
 = 0.00079788456080
 
なお、(左辺) = 0.00079788436133
0520132人目の素数さん
垢版 |
2018/04/18(水) 13:35:34.72ID:aemp1B+Z
コレクションになかったのを拾い集めてきた。(A1)以外は、たぶん過去スレにもないと思ふ。


【絶対値絡み】

(A1) a, b, c >0 に対して、|(a-b)/(a+b)| + |(b-c)/(b+c)| ≧ |(a-c)/(a+c)|

(A2) [宜蘭 2007]
相異なる a, b, c >0 に対して、|(a+b)/(a-b) + (b+c)/(b-c) + (c+a)/(c-a)| > 1

(A3) [疑問]
a, b, c >0 に対して、|(a-b)/(a+b) + (b-c)/(b+c) + (c-a)/(c+a)| のとりうる値の範囲は?


【分数式とか】

(B1) [中国 2008]
a, b, c >0 に対して、ab/c + bc/a + ca/b ≧ 2*(a^3 + b^3 + c^3)^(1/3)

(B2) [宜蘭 2010]
a, b, c >0 に対して、1/(a^2) + 1/(b^2) + 1/(c^2) + 1/{(a+b+c)^2} ≧ (7/25)*{1/a + 1/b + 1/c + 1/(a+b+c)}^2

(B3) [IMO short list 2008]
a, b, c, d >0 に対して、(a-b)(a-c)/(a+b+c) + (b-c)(b-d)/(b+c+d) + (c-d)(c-a)/(c+d+a) + (d-a)(d-b)/(d+a+b) ≧ 0

(B4) [不等式bot]
a, b >0 に対して、
(ab)^3 + (bc)^3 + (ca)^3 + 9(abc)^2 + abc(a^3 + b^3 + c^3 + 9abc) ≧ 3abc(a+b)(b+c)(c+a)


不等式botってのを最近見つけたんだけど、何これ? botって何ぞや?
同じ問題を繰り返し吐き出してるから、自動なのか?
登録してある問題をダブりなしに全部見てみたい。
0521132人目の素数さん
垢版 |
2018/04/18(水) 13:36:45.65ID:aemp1B+Z
【√がらみ】

(C1) [宜蘭 2008]
a, b, c >0 に対して、次式をみたす実数 k の最小値を求めよ。
a√b + b√c + c√a ≦ k√{(a+b)(b+c)(c+a)}

(C2) [香佐富斯坦 2010]
a, b >0 に対して、
√{(a^2-a+1)(b^2-b+1)} + √{(a^2+a+1)(b^2+b+1)} ≧ 2(x+y)

(C3) [スポック 2012]
a, b, c >0 に対して、
(a+b)√{(b+c)(c+a)} + (b+c)√{(c+a)(a+b)} + (c+a)√{(a+b)(b+c)} ≧ 4(ab+bc+ca)

(C4) [中国 2012]
a, b, c∈[0,1] のとき、√|a-b| + √|b-c| + √|c-a| の最大値を求めよ。

(C5) [波蘭 2004]
a, b, c∈R に対して、
√(2a^2+2b^2) + √(2b^2+2c^2) + √(2c^2+2a^2) ≧ √{3(a+b)^2 + 3(b+c)^2 + 3(c+a)^2}

(C6) [墺太利 2008]
a, b, c >0、a+b+c=1 に対して、
√{a^(b+c)*b^(c+a)*c^(a+b)} ≦ 1/3

(C7) [土耳古 2005]
a, b, c, d ∈R に対して、
√(a^4 + c^4) + √(a^4 + d^4) + √(b^4 + c^4) + √(b^4 + d^4) ≧ (2√2)*(ad+bc)
0522132人目の素数さん
垢版 |
2018/04/18(水) 13:37:59.14ID:aemp1B+Z
【微積分絡み】

(D1) [Putnum 1999]
実関数 f がC^3級で、任意の x∈R に対して、
 0 < f'(x)、 0 < f''(x)、 0 < f'''(x) ≦ f(x)
をみたすとき、f'(x) < 2f(x) を示せ。

(D2) [AoPS]
f は [0,1] で単調増加な凸関数で、f(0)=0、f(1)=1 をみたす。
g を fの逆関数とするとき、x^2 ≧ f(x)g(x) を示せ。

(D3) [近大 2008]
実関数 f がC^2級で、任意の x∈R に対して f''(x)≧f(x) をみたすとき、
f(x) ≧ f(0)*{e^x + e^(-x)}/2 + f'(0)*{e^x - e^(-x)}/2

(D4) [山梨医改、不等式bot]
f(0) = f(1) = 0、f'は[0,1]で連続のとき、∫[0,1] {f'(x)}^2 dx ≧ (π^2)*∫[0,1] {f(x)}^2 dx

(D5) [京大院 2011]
実連続関数 f,φ は区間[a,b]上で狭義単調増加のとき、
∫[a,b] f(x)dx = 0 ならば、∫[a,b] f(x)φ(x)dx > 0 を示せ。

(D6) [羅馬尼亜 2004]
fが[0,1]で積分可能で、∫[0,1] f(x)dx = ∫[0,1] xf(x)dx = 1 のとき、∫[0,1] {f(x)}^2 dx ≧ 4
0523132人目の素数さん
垢版 |
2018/04/18(水) 13:38:24.42ID:aemp1B+Z
          ;ヾ ;ヾ ;";ヾ;"  ;ヾ ;ヾ ;
        ;ヾ ;ヾ ;ヾ";ヾ;ヾ ';ヾ ;ヾ ;ヾ ;ヾ ;ヾ
       ;ヾ.;ヾ ';ヾ ;ヾ ;ヾ;ヾ ';ヾ ;ヾ ;ヾ ;ヾ ;ヾ ;ヾ"゙
    ""ヾ゙;ヾ〃;ヾ ;ヾ゙;ヾ ;ヾ ;ヾ"〃ヾ ;ヾ ;ヾ ;ヾ;ヾ
  "';ヾ;ヾ ;ヾ;ヾ ;ヾ ;ヾ ;ヾ ;ヾ ;ヾ ;//;ヾ;ヾ〃゙;ヾ ;ヾ;ヾ"
   """;ヾ ;ヾ ;ヾ ;ヾ ;ヾ ;ヾ ;ヾ" ;ヾ ;ヾ ;ヾ ;ヾ ;ヾ ;ヾ;ヾ"
  ""ヾ;ヾ ;ヾ ;ヾ ゙;ヾ〃ミヾ ;ヾ゙;ヾ ;ヾ 〃;ヾ ;ヾ ;ヾ ;ヾ;ヾ
     ;ヾ ;ヾ 〃;ヾ ;ヾ ;ヾ ;ヾ ;ヾ ;ヾ ;ヾ ;ヾ ;ヾ ;ヾ ;ヾ"
     "" ;ヾ ;ヾ゙ヾ ;ヾ ;ヾ;ヾ"゙ "iヾ;ヾ;ヾ" ゙ヾ;ヾ;ヾ"
        ""   'ヾ;ヾ" || l | ゙|/;ヾ"     "
             "   |l i  l゙l|
,,,,   ",,,," ,,, " ∧ ∧ ,,, |l | ゙ || '' ,, "  " ,,    春は不等式!
         ( ゚∀゚)∬ ノノ 从ヾ ヽ、   ,,, ''  やうやう白くなりゆく山際
 '' ` ` /   (_)旦.   /              少し明かりて、
     /         /    ''''     "" 紫だちたる雲の細くたなびきたる
      ̄ ̄ ̄ ̄ ̄ ̄ ̄
0524132人目の素数さん
垢版 |
2018/04/19(木) 02:39:28.61ID:jDhUrl+i
>>520

(A2) [155]
 (左辺) = |(p+q)/(p-q)|,
 ここに p = aab+bbc+cca -3abc ≧ 0,q = abb+bcc+caa -3abc ≧ 0,
 q-p = (a-b)(b-c)(c-a) = ,
 佐藤(訳) 問題3.103

(A3)
 絶対値の中身 = (a-b)(b-c)(c+a)/{(a+b)(b+c)(c+a)},
 -1 〜 +1

(B1) [96]
  (ab/c + bc/a + ca/b)^3 ≧ 8(aaa+bbb+ccc) + 3abc,
 bc/a=x,ca/b=y,ab/c=z とおく。

(B2) [198]
 a+b+c = s,1/a+1/b+1/c = 3/h とおく。
 s-3h ≧ 0,
 (左辺) ≧ 3/hh + 1/ss,
 (右辺) = (7/25)(3/h+1/s)^2,
 (左辺) - (右辺) ≧ 6(2s-h)(s-3h)/(5hs)^2 ≧ 0,
 等号成立は s-3h = 0,a=b=c.

(B3) [100]
 a-c,b-d の2次形式として正定値。

(B4) [107]
 (左辺) - (右辺) = (sssu+ttt+27uu) - 9stu ≧0 (←AM-GM)
 ここに、s = a+b+c、t = ab+bc+ca、u = abc.
0525132人目の素数さん
垢版 |
2018/04/19(木) 02:45:19.23ID:jDhUrl+i
>>521

(C1) [70]
 コーシーにより、
 (左辺)^2 ≦ (a+b+c)(ab+bc+ca) = (a+b)(b+c)(c+a) + abc ≦ (9/8)(a+b)(b+c)(c+a),
 K = 3/√8.
 佐藤(訳) 問題3.113

(C2) [114]
(左辺)^2 = 2{(aa+1)(bb+1) + ab + √(a^4+aa+1)√(1+bb+b^4)}
 = 2{(aa+3ab+bb) + (ab-1)^2 + √(a^4+aa+1)√(1+bb+b^4)}
 ≧ 2{(aa+3ab+bb) + (aa+ab+bb)}
 = 4(a+b)^2   (←コーシー)
 等号成立は xy=1.

(C3) [16]
 (a+b)√{(b+c)(c+a)} ≧ (a+b){c+√(ab)} ≧ (a+b)c + 2ab,
 循環的にたす。

(C4) [62]
 bはaとcの中間にあるとする。
 √|a-b| + √|b-c| + √|c-a| ≦ (1+√2)|c-a| ≦ 1 + √2、
 等号は(a,b,c)=(0,1/2,1)

(C5) [86]
 2√{3(aa+bb+cc)} ≧ √(2aa+2bb) + √(2bb+2cc) + √(2cc+2aa) ≧ √{3(a+b)^2+3(b+c)^2+3(c+a)^2} ≧ 2(a+b+c),
 (左辺)^2 = 4(aa+bb+cc) + 4√(aa+bb)√(bb+cc) + … + …
 ≧ 4(aa+bb+cc) + 2(a+b)(b+c) + 2(b+c)(c+a) + 2(c+a)(a+b)
 = 3(a+b)^2 + 3(b+c)^2 + 3(c+a)^2
 = (中辺)^2.

(C6) [49]
 f(x) = (1-x)log(x) ≦ -(1-x)^2 は 0<x<1 で上に凸。
 f(a) + f(b) + f(c) ≦ 3f((a+b+c)/3) = 2log(1/3)

(C7) [71]
 √(xx+yy) ≧ (x+y)/√2 を使う。
0526132人目の素数さん
垢版 |
2018/04/19(木) 02:51:51.34ID:jDhUrl+i
>>522

(D1) [143]
 未だ解けぬ〜

(D2) [164]
{f(x)/x} ' = {xf '(x) - f(x)}/xx =∫[0→x] {f '(x) - f '(t)}dt/xx > 0(←fは凸)
 f(x)/x は単調増加,
 x < g(x) < 1,
 f(x)/x ≦ f(g(x))/g(x) = x/g(x),

(D3) [144]
 0 ≦∫[0,x] {f ''(t) - f(t)}sinh(x-t)dt = [ f(t)cosh(x-t)+f '(t)sinh(x-t) ](t=0,x) = f(x) - f(0)cosh(x) - f '(0)sinh(x).

(D4) [121] (Wirtingerの不等式)
 g(x) = cot(x)とおく。
 g '(x) + g(x)g(x) = -1,
 [f(x)f(x)g(x)](x=0-π) = 0,
 ∴ 0 ≦∫{f '(x)−f(x)g(x)}^2 dx
 = ∫f '(x)f '(x)dx - [f(x)f(x)g(x)](x=0-π) + ∫f(x)f(x){g '(x) + g(x)g(x)}dx
 = ∫f '(x)f '(x)dx - ∫f(x)f(x)dx,
 大関・青柳「不等式」槇書店 p.204

(D5) [211]
 中間値の定理から、a<c<b なるcがあって f(c)=0,
 単調性から、(x-c)f(x)≧0、(x-c){φ(x)-φ(c)}≧0,
 これを入れる。

(D6) [54]
 0 ≦∫{f(x)+2-6x}^2 dx
 = ∫f(x)^2 dx + 4∫f(x)dx -12∫f(x)・x dx +4∫(1-3x)^2 dx
 = ∫f(x)^2 dx + 4 -12 +4.

[ ]内は Inequalitybot の番号ですぅ。
0527132人目の素数さん
垢版 |
2018/04/19(木) 05:10:55.02ID:gkRveId7
(C8) [月即別 2013] [187]
a≧b≧0 のとき、(a^2+b^2)^(1/2) + (a^3+b^3)^(1/3) + (a^4+b^4)^(1/4) ≦ 3a+b
0528132人目の素数さん
垢版 |
2018/04/19(木) 13:00:30.09ID:jDhUrl+i
>>527

(C8) [187]
 (a^2 + b^2)^(1/2) ≦ a + (√2−1)b,
 (a^3 + b^3)^(1/3) ≦ a + {2^(1/3)−1}b,
 (a^4 + b^4)^(1/4) ≦ a + {2^(1/4)−1}b,
辺々たす。
 (左辺) ≦ 3a + 0.8633417b
0529132人目の素数さん
垢版 |
2018/04/21(土) 09:22:11.44ID:I5oMZRza
>>87

 a^(2/3) = A,b^(2/3) = B,c^(2/3) = C とおくと、

 aa+bb+cc - 2(ab+bc+ca) + 2abc+1
 ≧ A^3 + B^3 + C^3 - 2{AB(A+B) +BC(B+C) +CA(C+A)} + 3ABC
 = 兩3(A,B,C)     >>513
 = F_1(A,B,C)
 ≧ 0,
0530132人目の素数さん
垢版 |
2018/04/22(日) 13:39:47.36ID:7rjXNdwL
>>163

〔Turkevici の不等式〕 - 改

a^4 + a^4 + a^4 + a^4 + 2abcd - {ab(aa+bb) +ac(aa+cc) +ad(aa+dd) +bc(bb+cc) +bd(bb+dd) +cd(cc+dd)}/2
= a^4 + a^4 + a^4 + a^4 + 2abcd - {(a^3)(b+c+d) -(b^3)(c+d+a) -(c^3)(d+a+b) -(d^3)(a+b+c)}/2
= (1/2)兩4
≧ 0,     >>513
0531132人目の素数さん
垢版 |
2018/04/22(日) 13:55:51.43ID:7rjXNdwL
>>163

〔Turkevici の不等式〕- 改

a^4 + b^4 + c^4 + d^4 + 2abcd - {ab(aa+bb) +ac(aa+cc) +ad(aa+dd) +bc(bb+cc) +bd(bb+dd) +cd(cc+dd)}/2
= {3(a^4 + b^4 + c^4 + d^4) -(a+b+c+d)(a^3+b^3+c^3+d^3) + 4abcd}/2
= (1/2)兩4
≧ 0     >>513
0533132人目の素数さん
垢版 |
2018/04/24(火) 00:18:06.70ID:43b+grE0
>>515
 a^(10/3) = A,b^(10/3) = B,c^(10/3) = C とおくと本題は
 (A^3+B^3+C^3)^2 ≧ 3(A^4+B^4+C^4)

〔補題〕
 (A^3+B^3+C^3)^2 ≧ (AB+BC+CA)(A^4+B^4+C^4)

m = min{A,B,C} とおき、
{A,B,C} = {m,m+x,m+y} (x≧0,y≧0)
とする。
(左辺) - (右辺) = (A^3+B^3+C^3)^2 - (AB+BC+CA)(A^4+B^4+C^4) 
 = (m^4)(xx-xy+yy) + (2m^3)xy(x+y) + (2m^2){2xx(x-y)^2 +5xxyy +2yy(x-y)^2} + m(x+y){xx(2x-2.5y)^2 +(7/2)xxyy +yy(2.5x-2y)^2} + (x-y)(x^5-y^5) + 2(xy)^3
 ≧ 0,
0534132人目の素数さん
垢版 |
2018/04/24(火) 11:04:06.96ID:43b+grE0
>>515 >>533

〔補題〕
 (A^3+B^3+C^3)^2 ≧ (AB+BC+CA)(A^4+B^4+C^4)

(左辺) - (右辺) = F_0(A,B,C) F_0(AA,BB,CC) + (ABC)^2 F_{-2}(A,B,C) ≧ 0,

F_0(A,B,C) = (A-B)(A-C) + (B-C)(B-A) + (C-A)(C-B) = {(A-B)^2 + (B-C)^2 + (C-A)^2}/2 ≧ 0,

F_{-2}(A,B,C) = (A-B)(A-C)/AA + (B-C)(B-A)/BB + (C-A)(C-B)/CC = ABC F_1(1/A,1/B,1/C) ≧ 0,
0535132人目の素数さん
垢版 |
2018/04/24(火) 23:16:08.34ID:43b+grE0
>>533 >>534

〔補題〕
1≦n≦3,A〜C≧0 のとき
 (A^n + B^n + C^n)^2 ≧ (AB+BC+CA) {A^(2n-2)+B^(2n-2)+C^(2n-2)} ≧ 3ABC {A^(2n-3)+B^(2n-3)+C^(2n-3)},

右側はチェビシェフなど。
0536132人目の素数さん
垢版 |
2018/04/25(水) 00:55:21.59ID:i3CGBkWM
>>533 >>534 >>535

〔補題〕
1≦n≦5,A〜C≧0 のとき
 (A^n + B^n + C^n)^2 ≧ 3ABC {A^(2n-3) + B^(2n-3) + C^(2n-3)},

(例)
n=3 のとき
(左辺) - (右辺) = (A^3 +B^3 +C^3) (A^3 +B^3 +C^3 -3ABC) ≧ 0,

n=4 のとき
(左辺) - (3/2) {(a^3)(b+c) + (b^3)(c+a) + (c^3)(a+b)}
 = (1/2) {(aa-ab+bb)(a-b)^2 + (bb-bc+cc)(b-c)^2 + (cc-ca+aa)(c-a)^2}
 ≧ 0,
ここに、a=AA,b=BB,c=CC.

(3/2) {(a^3)(b+c) + (b^3)(c+a) + (c^3)(a+b)} - (右辺)
 = (3/2) {(A^6)(B-C)^2 + (B^6)(C-A)^2 + (C^6)(A-B)^2}
 ≧ 0,
0537132人目の素数さん
垢版 |
2018/04/25(水) 22:02:16.65ID:CPKgHcHK
三角形の辺長 a、b、c に対して、
  {√(a+b-c)}/(√a + √b - √c) + {√(b+c-a)}/(√b + √c - √a) + {√(c+a-b)}/(√c + √a - √b) ≦ 3
0538132人目の素数さん
垢版 |
2018/04/27(金) 02:42:51.45ID:MuDbcQQ+
>>537 [6]

A = √b+√c-√a > 0,
B = √c+√a-√b > 0,
C = √a+√b-√c > 0,
とおく。
b+c-a = AA - (A-B)(A-C)/2,
√(b+c-a) ≦ A - (A-B)(A-C)/4A,
(左辺) = √(b+c-a) /A + √(c+a-b) /B + √(a+b-c) /C
 ≦ 3 - (A-B)(A-C)/(4AA) - (B-C)(B-A)/(4BB) - (C-A)(C-B)/(4CC)
 = 3 - (1/4) F_{-2}(A,B,C)
 = 3 - (ABC/4) F_1(1/A,1/B,1/C)
 ≦ 3.

 IMOSL-2006 予選 A.6、JMO春合宿
 文献[8] 安藤 (2012),p.147 例題3.2.3(9),
 http://www.casphy.com/bbs/highmath/ 不等式1-307、434、437
0540132人目の素数さん
垢版 |
2018/04/27(金) 03:03:26.10ID:WAhBLXIV
実数 a_k、b_k (1≦k≦n)) に対して、
1 + Σ[k=1 to n] (a_k + b_k)^2 ≦ (4/3)*{1 + Σ[k=1 to n] (a_k)^2}*{1 + Σ[k=1 to n] (b_k)^2}
0542132人目の素数さん
垢版 |
2018/04/27(金) 11:37:15.92ID:MuDbcQQ+
>>540

A = Σ[k=1,n] (a_k)^2,
B = Σ[k=1,n] (b_k)^2,
C = Σ[k=1,n] a_k b_k,
とおく。
A+B-2C = Σ[k=1,n] (a_k - b_k)^2 ≧ 0,
AB-CC = Σ[1≦j<k≦n] (a_j b_k - a_k b_j)^2 ≧ 0  (←コーシー)

(右辺) - (左辺) = (4/3)(1+A)(1+B) - (1+A+B+2C)
= (1/3) (1+A+B+4AB-6C)
= (1/3) {(A+B-2C) + 4(AB-CC) + (1-2C)^2}
≧ 0,

等号成立は a_k = b_k,A = B = C = 1/2.
0543132人目の素数さん
垢版 |
2018/04/28(土) 00:44:27.52ID:9CKS2DSq
>>541

gcd(m,n) | (m-n)
gcd(m+1,n+1) | (m-n)
左辺は互いに素ゆえ、 (←背理法で)
gcd(m,n)gcd(m+1,n+1) | (m-n)

lcm(m,n) + lcm(m+1,n+1)
= mn/gcd(m,n) + (m+1)(n+1)/gcd(m+1,n+1)
> mn{1/gcd(m,n) + 1/gcd(m+1,n+1)}
> 2mn/√{gcd(m,n)gcd(m+1,n+1)}    (←AM-GM)
≧ 2mn/√(m-n),
0544132人目の素数さん
垢版 |
2018/04/29(日) 13:40:34.24ID:LZWvDOTX
>>374 >>398 >>399 >>416 >>417

 nΣ[k=1,n] s_k (a_k)^2 ≧ M_n (s_n)^3,
とおく。

M_2 = 0.7377393811182 = 2(47-14√7)/27
   (a,b) =(√7 -1,4-√7)(3+√7,2+√7)

M_3 = 0.6481616033162
   (a,b,c) = (1.38436,1.13916,1)

M_4 = 0.60233351875
   (a,b,c,d) = (1.52472,1.25465,1.10139,1)

M_5 = 0.574255

M_6 = 0.5551782

M = 0.444444 = 4/9   (n→∞),
0545132人目の素数さん
垢版 |
2018/04/29(日) 22:17:02.33ID:LZWvDOTX
>>544
 Memo.

漸化式は
 a_{n+1} = (1/2) {√(2x-1) - 1} s_n,
 s_{n+1} = s_n + a_{n+1},
 M_n = (n/3) (x-1),
ここに
 x = (1 + a_n/s_n)^2.

(例)

M_1 = 1
 a_1 = s_1 = 1

M_2 = 2(47-14√7)/27 = 0.7377393811182
 a_2 = (√7 -1)/2 = 0.8228756555323
 s_2 = (√7 +1)/2 = 1.8228756555323

M_3 = 0.64816160331616
 a_3 = 0.72235563718495
 s_3 = 2.54523129271725

M_4 = 0.60233351872589
 a_4 = 0.65585825517001
 s_4 = 3.20108954788726

M_5 = 0.57425545264547
 a_5 = 0.60768519695068
 s_5 = 3.80877474483794

M_6 = 0.55517800140267
 a_6 = 0.57066170678793
 s_6 = 4.37943645162587

本題から逸れてしまった…
0546132人目の素数さん
垢版 |
2018/04/29(日) 22:33:49.66ID:LZWvDOTX
>>374 (改)

Σ[k=1,n] (s_{k-1} + s_k)/2 ・ (a_k)^2 > (4/9n) (s_n)^3,

便宜上 s_0 = 0 とおいた。

* 中点 (s_{k-1} + s_k)/2 で接線を曳く。
0547132人目の素数さん
垢版 |
2018/04/30(月) 07:31:33.28ID:iiZ/CJ+E
>>544
>>545 Memo. の続き

M_10 = 0.51565443182467
 a_10 = 0.47804498656917
 s_10 = 6.41086198943751

M_100 = 0.45433807243808
 a_100 = 0.21749813721698
 s_100 = 32.0226683930223

M_1000 = 0.44575956171259
 a_1000 = 0.10051892239154
 s_1000 = 150.383787216053

M_10000 = 0.44460977509949
 a_10000 = 0.04662595061307
 s_10000 = 699.152499550131
0548132人目の素数さん
垢版 |
2018/05/02(水) 00:00:30.49ID:mJGWQQsb
>>545
Memo.

(略証)
nについての帰納法による。
 a_{n+1} = A と略す。
まず s_n を固定して a_1 〜 a_n を動かしたときの最小値は、
 Σ[k=1,n+1] s_k (a_k)^2 - μ(s_{n+1})^3
 = Σ[k=1,n] s_k (a_k)^2 + (s_n + A)A^2 - μ(s_n + A)^3
 ≧ (M_n /n)(s_n)^3 + (s_n + A)A^2 - μ(s_n + A)^3 = f(A)
つまり a_1 〜 a_n の比はnの場合と同じでよい。

次に f(A) = 0 が重根をもつようにμを決めるのだが、言い換えれば
f(A) = 0 と f '(A) = 0 が共通根をもつことである。

f(A) = (M_n /n)(s_n)^3 + (s_n + A)A^2 - μ(s_n + A)^3 = 0,
f '(A) = 2(s_n)A + 3A^2 - 3μ(s_n + A)^2 = 0,
から A とμを決める。
まずμを消去すれば
 A(s_n + A) - 3M_n /(2n・s_n) = 0,
∴ A = (1/2){√[1 + 6M_n /(n・(s_n)^3)] -1}s_n,
これを使うとμが求まり
 M_{n+1} = (n+1)μ = {(n+1)/3}([1 + A/s_{n+1}]^2 - 1),
 s_{n+1} = s_n + A,
と表わせる。
0549132人目の素数さん
垢版 |
2018/05/02(水) 03:13:46.81ID:mJGWQQsb
〔問題8〕
閉区間 [0,1] で定義された連続関数f(x)は、次の条件を満たすとする。
ある正の実数Lが存在して、[0,1] 上のすべての実数xにおいて
 0 ≦ f(x) ≦ L∫[0,x] f(t)dt
が成り立つ。
このとき、[0,1] 上のすべての実数xにおいてf(x)=0であることを示せ。
http://www.toshin.com/concours/mondai/mondai8.php

〔問題12〕
p_1,p_2,…,p_k を m 以下のすべての素数とする。
この時、以下の不等式が成り立つことを示せ。
 log(m) - 1 ≦ (1/m)log(m!) < Σ[i=1,k] log(p_i)/(p_i - 1)
http://www.toshin.com/concours/mondai/mondai12.php

〔問題18〕
正の実数 a,b,c が ab+bc+ca=1 を満たすとき
 (b+c) {√(aa+1) +a} ≧ 2,
 (c+a) {√(bb+1) +b} ≧ 2,
 (a+b) {√(cc+1) +c} ≧ 2,
が成り立つことを示せ。
http://www.toshin.com/concours/mondai/mondai18.php

〔問題32〕
nを2以上の整数とする。正の実数 a_1,a_2,…,a_n に対して不等式
 Σ[k=1,n] (kk-2k+2)a_k + Σ[k=1,n-1] (1/a_k)(a_{k+1})^2 ≧ (n^2)a_n
が成り立つことを示せ。また、等号が成立する条件を求めよ。
http://www.toshin.com/concours/mondai/mondai32.php
0552132人目の素数さん
垢版 |
2018/05/02(水) 22:45:46.08ID:G30sYyfK
[bot 5]
a, b, c≧0 のとき、a(a-b)(a-2b) + b(b-c)(b-2c) + c(c-a)(c-2a) ≧0

(1) この証明は?
(2) a(a-mb)(a-nb) + … とイパーン化できるでござるか?
0553132人目の素数さん
垢版 |
2018/05/03(木) 00:16:14.95ID:CZ0Fa01r
>>552 [111]

(1)
min{a, b, c} = M, {a, b, c} = {M, M+x, M+y} とすると、
(左辺) = 2M(xx-xy+yy) + x(x-2y)^2 + (x-y)^2・y ≧0.

USA.ELMO-2009 day1-Q.3
0554132人目の素数さん
垢版 |
2018/05/05(土) 12:58:01.27ID:cos8i+vX
>>552 [111]

(2)
m(m+1) ≦ 3 + 4√2 のとき
 a(a-b)(a-mb) + b(b-c)(b-mc) + c(c-a)(c-ma) ≧ 0

m(m+1) = 3 + 4√2 の根は
 m_1 = -{√(13+16√2) +1}/2 = -3.4844353317658568752
 m_2 = {√(13+16√2) -1}/2 = 2.4844353317658568752

等号成立条件
・m_1 < m < m_2 のとき (a,b,c) = (1,1,1)
・m = m_1,m_2 のとき
 (a,b,c) = (1,1,1)、(0,t1,1)、(0,1,t2) とそのrotation
 t_1,t_2 は tt - (1+√2)t + 1 = 0 の根
 t_1 = {1+√2 -√(2√2-1)}/2 = 0.531010056459569184633
 t_2 = {1+√2 +√(2√2-1)}/2 = 1.883203505913525864169
0556132人目の素数さん
垢版 |
2018/05/07(月) 23:55:33.94ID:LPTyY7qu
〔問題〕
自然数nに対して

(1) C[2n,n] = (2n)! / (n!)^2 ≧ 4^n / (2√n),

(2) C[3n,n] = (3n)! / {n!・(2n)!} ≧ (27/4)^n ・4/(9√n),

等号成立は n=1

>>512 >>513
Janos Suranyi の不等式と云うらしい…
0557132人目の素数さん
垢版 |
2018/05/08(火) 00:30:20.27ID:rSTdfkqz
>>556

(1) は >>474 >>476 >>495 と同様

(2) もnについての帰納法で

C[3n+3,n+1] / C[3n,n] = {(3n+1)(3n+2)(3n+3)}/{(2n+1)(2n+2)(n+1)}
= (27/4) {(n+1/3)(n+2/3)} / {(n+1/2)(n+1)}
= (27/4) (N + 2/9) / {(n+1/2)(n+1)}    ← N=n(n+1) とおいた。
> (27/4) (N + 1/8) / {(n+1/2)(n+1)}
≧ (27/4) √{N(N+1/4)} / {(n+1/2)(n+1)}
= (27/4) (n+1/2)√{n(n+1)} / {(n+1/2)(n+1)}
= (27/4) √{n/(n+1)},
により成立
0568132人目の素数さん
垢版 |
2018/05/08(火) 18:14:59.57ID:rSTdfkqz
>>556 の系

C[3n,2n] ≧ 4/(9√n)・(27/4)^n,

C[3n-1,2n-1] = (2/3)C[3n,2n] ≧ (2/√n)・(27/4)^(n-1),
0571132人目の素数さん
垢版 |
2018/05/19(土) 05:31:14.93ID:xhdfIuy0
>>549

〔問題8〕の解答

h(x) = e^(-Lx) ∫[0,x] f(t)dt とおくと題意により
 h(x) ≧ 0 = h(0)   …… (1)
また h(x) は(0,1) 上で微分可能で
 h '(x) ≦ 0,
∴ h(x) = h(0) + ∫[0,x] h '(t)dt ≦ h(0)  …… (2)
(1) (2) により [0,1] 上で h(x) = h(0) = 0 が成り立つ。
したがって、[0,1] 上のすべての実数xにおいて
 0 ≦ f(x) ≦ L ∫[0,x] f(t)dt = 0,
より、f(x) = 0 である。   ■

http://www.toshin.com/concours/mondai/mondai8.php
0572132人目の素数さん
垢版 |
2018/05/19(土) 07:16:34.16ID:xhdfIuy0
>>549

〔問題12〕の解答

(左側)
 任意の正の整数mに対し、
 log(m!) = Σ[L=1,m-1] log(L+1) ≧ Σ[L=1,m-1] ∫[L,L+1] log(t)dt = ∫[1,m] log(t)dt = m{log(m) -1} +1,
∴ log(m) - 1 ≦ (1/m)log(m!)

(右側)
実数xに対し、x以下の最大の整数を [x] で表わす。
また、0でない整数nと素数pに対し、v_p(n) で、nの素因数分解に現れるpの回数を表わすものとする。
ここで、m! はm以下の素数しか素因数に持たないので、
 log(m!) = Σ[i=1,k] v_pi(m!) log(p_i)
と表わされる。ここで、
 v_p(m!) < m/(p-1)
が分かるのでこれを上の式と組み合わせて
 (1/m)log(m!) < Σ[i=1,k] log(p_i)/(p_i -1)
が示された。(終)

http://www.toshin.com/concours/mondai/mondai12.php
0573132人目の素数さん
垢版 |
2018/05/19(土) 07:59:43.93ID:xhdfIuy0
>>549

〔問題32〕の解答

(a_{k+1})^2 / a_k ≧ 2k・a_{k+1} - kk・a_k,
辺々たして
Σ[k=1,n-1] (a_{k+1})^2 / a_k ≧ Σ[k=2,n] 2(k-1) a_k - Σ[k=1,n-1] kk・a_k
 = nn・a_n - Σ[k=1,n] a_k - Σ[k=1,n] (k-1)^2・a_k
 = nn・a_n - 1 - Σ[k=1,n] (k-1)^2・a_k
を導く。等号成立条件は、各 k=1,2,…,n-1 で a_{k+1} = k・a_k である場合だから、すべての i=1,2,…,n に対し
 a_i = (i-1)! /{Σ[k=1,n] (k-1)!}
が成立することである。

http://www.toshin.com/concours/mondai/mondai32.php
0576132人目の素数さん
垢版 |
2018/05/20(日) 18:26:26.97ID:1IiDnvUy
>>575 [192]

任意の実数a,b,cに対し、
(a-b)(a-c)(aa-bc)^2 + (b-c)(b-a)(bb-ca)^2 + (c-a)(c-b)(cc-ab)^2 ≧ 0,
を示せ。

 //www.casphy.com/bbs/highmath/不等式2-188 (じゅー)
0577132人目の素数さん
垢版 |
2018/05/20(日) 18:37:09.27ID:1IiDnvUy
>>574 [104]

s = a+b+c,t = ab+bc+ca,u = abc, = (a-b)(b-c)(c-a) とおく。
ss-3t≧0,
(左辺) = (ss-2t)^2 -tt
 = (ss-t)(ss-3t)
 = (1/3){2ss + (ss-3t)}(ss-3t)
 ≧ {(2√2)/3}|s|(ss-3t)^(3/2),
 ≧ (√6)|s處,
∵ 4(ss-3t)^3 = 27刧 + {(a+b-2c)(b+c-2a)(c+a-2b)}^2 ≧ 27刧,
等号成立は等間隔かつ ss+3t = 0 より{1-√6,1,1+√6}

http://www.casphy.com/bbs/highmath/ 不等式2-197
0578132人目の素数さん
垢版 |
2018/05/20(日) 19:27:29.09ID:1IiDnvUy
>>572 (右側) 補足

自然数nと素数pに対し、v_p(n) で、nの素因数分解に現れるpの回数を表わすものとする。
 v_pi(m!) = [ m/p ]+ [ m/p^2 ] + [ m/p^3 ] + … + [ m/p^d ]
ここに、d = [ log(n)/log(p) ].

これもルジャンドルの定理と云うらしい。
http://mathtrain.jp/legendretheorem

〔補題12〕
 v_p(m!) < m/(p-1)

(略証)
d = [ log(n)/log(p) ] とおくと
 v_pi(m!) ≦ m/p + m/p^2 + m/p^3 + … + m/p^d < m/(p-1),
0579132人目の素数さん
垢版 |
2018/05/20(日) 20:56:05.37ID:TfngVWLR
a, b, c > 0 に対して、a/(b+c) + 20b/(c+a) + 17c/(a+b) > 8

best possible かどうか分からん
0582132人目の素数さん
垢版 |
2018/05/21(月) 04:03:40.39ID:9YF4F+CN
>>581

B.4925 (改) (KoMaL,h=201801)
 0<a<n のとき
 a/{a^(n+1) + (n-a)} ≦ 1/n

(略解)
 a^(n+1) -(n+1)a + n
 = (a-1){a^n + a^(n-1) + … + a -n}
 = Σ[k=1,n] (a-1)(a^k -1)
 ≧ 0,


B.4931 (KoMaL,h=201802)
 {aa(b+c) + bb(a+c)}/abc > 3,

(略解)
 aa(b+c) + bb(a+c) = ab(a+b-c) + (a-b)^2・c + 3abc ≧ 3abc,


B.4953 (KoMaL,h=201804)
 log(n) + Σ[k=2,n] √{(k-1)/k} < Σ[k=2,n] √{k/(k-1)},

(略解)
 x>0 ⇒ x < sinh(x),
 a>1 ⇒ 2log(a) < a - 1/a,
 a = √{k/(k-1)} とおく。
 log(k) - log(k-1) < √{k/(k-1)} - √{(k-1)/k},
k=2 から k=n までたす。


Math. Excalibur,Vol.21,No.4,p.1 (2018)
Problem 1.
 a,b,c >0,a+b+c=1 のとき
 a√(2b-1) + b√(2c+1) + c√(2a+1) ≦ √{2-(aa+bb+cc)},

(略解)
関数f(x) = √x は上に凸ゆえ、Jensenで
(左辺) ≦ √{a(2b+1) + b(2c+1) + c(2a+1)}
 = √{(a+b+c) + 2(ab+bc+ca)} / (a+b+c)
 = √{1 +2(ab+bc+ca)}
 = (右辺)
等号成立は (a,b,c) = (1/3,1/3,1/3) および (1,0,0) など。
0583132人目の素数さん
垢版 |
2018/05/21(月) 15:35:20.78ID:9YF4F+CN
>>579

左辺が最小になる点では
(b+c)^2 : (c+a)^2 : (a+b)^2 = 1 : 20 : 17,
(b+c) : (c+a) : (a+b) = √1 : √20 : √17,
b+c = √1,
c+a = √20,
a+b = √17,
a = (-√1 +√20 +√17)/2,
b = (+√1 -√20 +√17)/2,
c = (+√1 +√20 -√17)/2,

(左辺) ≧ a√1 + b√20 + c√17
 = √(1・20) +√(20・17) +√(17・1) -19
 = 8.0343304952
0584132人目の素数さん
垢版 |
2018/05/21(月) 18:58:12.63ID:9YF4F+CN
>>579 >>583

b+c = A,c+a = B,a+b = C とおくと

(左辺) = 1・(B+C-A)/(2A) + 20(C+A-B)/(2B) + 17(A+B-C)/(2C)
 = (1/2)(1・B/A + 20A/B) + (1/2)(20C/B + 17B/C) +(1/2)(1・C/A + 17A/C) - (1+20+17)/2
 ≧ √(1・20) + √(20・17) + √(17・1) - 19  (← AM-GM)
等号成立は A:B:C = √1:√20:√17
0586132人目の素数さん
垢版 |
2018/05/22(火) 05:22:53.47ID:RuE2vaj6
>>486 >>487 (3)

 文献[9] 佐藤(訳) (2013) p.48 演習問題 1.101

・p=1,q=2 の例
 文献[9] 佐藤(訳) (2013) p.48 例 1.6.7 及び p.131 問題 3.30
 チェコ-スロバキアMO-1999
0587132人目の素数さん
垢版 |
2018/05/22(火) 05:40:43.12ID:RuE2vaj6
>>575 >>576 [192]

一次式:φ(x) = (a+b+c)x−(ab+bc+ca)により、
A = φ(a) = aa-bc,
B = φ(b) = bb-ca,
C = φ(c) = cc-ab.
A - B = (a+b+c)(a-b)、etc.
i)a+b+c≠0 のとき、
 (左辺) = {AA(A-B)(A-C) + BB(B-C)(B-A) + CC(C-A)(C-B)}/(a+b+c)^2 = F_2(A、B、C)/(a+b+c)^2 ≧0、
ii)a+b+c=0 のとき、A=B=C.

 http://www.casphy.com/bbs/highmath/ 不等式2-188
0588132人目の素数さん
垢版 |
2018/05/22(火) 21:32:13.53ID:+yEhb6+c
一松のじっちゃんが「大学への数学2018年6月号」に不等式の記事を書いておられる。
エルデシュの不等式とか
0589132人目の素数さん
垢版 |
2018/05/27(日) 01:54:57.05ID:4AQyIVUB
不等式と聞ゐちゃあ捨て置けねゑ…。このためだけに買ってきた。

タイトル 「三角形に関する不等式のいくつか」、4ページ

レムスの不等式と、求角不等式。
内角のcosの等式から、a^2+b^2+c^2 と8R^2の大小関係。
 (中略)
エルデシュの不等式。
過去スレで見たことある不等式。

あと、「老人のグチだが、(中略)近年お数学検定で、不等式の証明問題は成績が悪い傾向が見られる。」
とあるが、検定問題で出題されている不等式を全て公開してほしい。
0590132人目の素数さん
垢版 |
2018/05/27(日) 05:51:38.11ID:pjauAWRB
ここで不等式解いてる人って50後半の会社員だったりする?
0593132人目の素数さん
垢版 |
2018/05/28(月) 09:56:02.44ID:TTo2rnUU
>>589

(aa+bb+cc) - 8RR = 4RR {sin(A)^2 + sin(B)^2 + sin(C)^2 - 2}
= 4RR {1 - cos(A)^2 - cos(B)^2 - cos(C)^2}
= 8RR cos(A) cos(B) cos(C),

〔補題〕
A+B+C = π のとき
 cos(A)^2 + cos(B)^2 + cos(C)^2 + 2cos(A)cos(B)cos(C) = 1,

〔ライプニッツの不等式〕
 9RR - (aa+bb+cc) = 9(OG)^2 ≧ 0,
 O:外心 G:重心

・文献[9] 佐藤(訳) 朝倉書店 (2013) p.87-89 定理2.4.4 定理2.4.5
0594132人目の素数さん
垢版 |
2018/05/29(火) 02:45:55.24ID:n11ck1yy
>>591 は相加-相乗平均(AM-GM)で出るらしい。 (出題者・談)

{1,1,…,(n-1)/n} ⇒ a_n > a_{n-1} > … > a_1 = 2

{1,1,…,n/(n-1)} ⇒ b_n < b_{n-1} < … < b_1 = 4

しかし c_n = (1 + 1/n)^(n +1/2) が減少するのを出すのは難しい。
2項定理を使うか?
0595132人目の素数さん
垢版 |
2018/05/29(火) 03:01:10.20ID:n11ck1yy
>>594

2項定理により、

(1 - 1/nn)^(2n+1) = 1 -2/n + 1/nn -1/(3n^3) +2/(3n^4) -3/(5n^5) +53/(90n^6) -…

 < (1 - 1/n)^2
0597132人目の素数さん
垢版 |
2018/05/30(水) 08:18:45.80ID:FdL02lRD
Crux PROBLEMS
(2012年のが公開された。5年以内のはパスワードがないと見れない)

3690, 3703, 3706, 3709
https://cms.math.ca/crux/v38/n1/Problems_38_1.pdf

3712, 3715, 3719(←破棄)
https://cms.math.ca/crux/v38/n2/Problems_38_2.pdf

3719(←Replacement), 3723, 3726, 3729
https://cms.math.ca/crux/v38/n3/Problems_38_3.pdf

3731, 3735, 3737, 3740
https://cms.math.ca/crux/v38/n4/Problems_38_4.pdf

3741, 3744, 3747, 3749
https://cms.math.ca/crux/v38/n5/Problems_38_5.pdf

3752, 3754, 3757, 3759
https://cms.math.ca/crux/v38/n6/Problems_38_6.pdf

3763, 3767, 3769
https://cms.math.ca/crux/v38/n7/Problems_38_7.pdf

3773, 3774, 3776, 3779
https://cms.math.ca/crux/v38/n8/Problems_38_8.pdf

(3781), 3783, (3784), (3786), 3788, 3789
https://cms.math.ca/crux/v38/n9/Problems_38_9.pdf

3793, 3795, 3797,
https://cms.math.ca/crux/v38/n10/Problems_38_10.pdf

(*゚∀゚)=3ハァハァ
0598132人目の素数さん
垢版 |
2018/06/01(金) 08:31:09.16ID:PJfeGZ2B
>>597 から

3690.(v38_n1)
 Let a, b, and c be three distinct positive real numbers with a+b+c=s. Show that
  (5xx-6xy+5yy)(a^3+b^3+c^3) + 12(xx-3xy+yy)abc > (x-y)^2・s^3,

3709.(v38_n1)
 Let a, b, and c be non-negative real numbers, k and L≧0 and define
  (a+b)/2 - √ab = k^2,  (a+b+c)/3 - (abc)^(1/3) = L^2.
 Prove that
  max(a,b,c) - min(a,b,c) ≧ (3/2)(k-L)^2.

3712.(v38_n2)
 Prove that for any positive numbers a,b,c
  √{a(aa+bc)/(b+c)} + √{b(bb+ca)/(c+a)} + √{c(cc+ab)/(a+b)} ≧ a+b+c.

3719.(v38_n3,Replacement)
 Prove that if a,b,c>0, then
 a/√{bb+(1/4)bc+cc} + b/√{cc+(1/4)ca+aa} + c/√{aa+(1/4)ab+bb} ≧ 2.

3723.(v38_n3)
 Let a,b,c be positive real numbers such that a+b+c=s. If n is a positive integer, prove that
 (3a)^n /{(b+s)(c+s)} + (3b)^n /{(c+s)(a+s)} + (3c)^n /{(a+s)(b+s)} ≧ (27/16)s^(n-2).

3731.(v38_n4)
 Let a,b,c be positive real numbers such that a+b+c=s. Prove that
  a^(n+1) + b^(n+1) + c^(n+1) ≧ (aa+bb+cc)^n / s^(n-1),
 for all non-negative integers n.

3737.(v38_n4)
 Four non-negative real numbers a,b,c,d are given. Show that
  1/(a^3+b^3) + 1/(b^3+c^3) + 1/(c^3+d^3) + 1/(a^3+c^3) + 1/(b^3+d^3) + 1/(a^3+d^3) ≧ 243/{2(a+b+c+d)^3},
 Equality: {a,b,c,d} = {0,1,1,1}

3741.(v38_n5)
 Find the largest value of a and the smallest value of b for which the inequalties
  ax/(a+xx) < sin(x) < bx/(b+xx)
 hold for all 0<x<π/2.

3744.(v38_n5)
 Let a,b,c be positive real numbers with sum s. Prove that
  (a^8+b^8)/(aa+bb)^2 + (b^8+c^8)/(bb+cc)^2 + (c^8+a^8)/(cc+aa)^2 ≧ (a^3+b^3+c^3-abc)s/4.

3752.(v38_n6)
 Show that if n≧2 is a positive integer then
 (1/2)(1 +1/n -1/nn)^2 < (1 - 1/2^3)(1 - 1/3^3) … (1 - 1/n^3).

Crux mathematicorum, Vol.38 (2012)、一部改作
0599132人目の素数さん
垢版 |
2018/06/01(金) 08:45:52.63ID:PJfeGZ2B
>>597 から

3763.(v38_n7)
 Let a,b,c be positive real numbers. Prove that
  a/(2a+b+c) + b/(2b+c+a) + c/(2c+a+b) ≦ a/(2b+2c) + b/(2c+2a) + c/(2a+2b).

3793.(v38_n10)
 Let a, b, and c be positive real numbers such that
  √a + √b + √c = 2014/√2.
 Show that
  2014 ≦ √(a+b) + √(b+c) + √(c+a) ≦ 2014√2,
 Equality:(LHS) √a = √b = √c = 2014/(3√2),(RHS) √a = 2014/√2,b=c=0,

・三角形関係

3726.(v38_n3)
 Let a,b,c,s,r,R represent the angles (measured in radians),the semi-perimeter,the in-radius and the circum-radius of a triangle,respectively.
 Prove that
  (A/B + B/C + C/A)^3 ≧ 2ss/(Rr).

3729.(v38_n3)
 If a,b,c are the side lengths of a triangle,prove that
  (b+c)/(aa+bc) + (c+a)/(bb+ca) + (a+b)/(cc+ab) ≦ 3(a+b+c)/(ab+bc+ca).

3757.(v38_n7)
 Let A, B, C be the angles (measured in radians),R the circum-radius and r the in-radius of a triangle.
 Prove that
 1/A + 1/B + 1/C ≦ (9/2π)(R/r).

3767.(v38_n7)
 Let R,r be the circum-radius and in-radius of a right-angled triangle.
 Prove that
  R/r + r/R ≧ 2√2.

3776.(v38_n8) 別名「富士山」
 In △ABC prove that
 tan(A/2) + tan(B/2) + tan(C/2) ≧ (1/2){1/cos(A/2) + 1/cos(B/2) + 1/cos(C/2)}.

Crux mathematicorum, Vol.38 (2012)、一部改作
0600132人目の素数さん
垢版 |
2018/06/01(金) 11:22:47.44ID:PJfeGZ2B
>>598

3690.
軸を45°回して (x+y)/√2 = u,(x-y)/√2 = v とおく。
5xx-6xy+5yy = 2uu +8vv,
12(xx-3xy+yy) = -6uu +30vv,
(x-y)^2 = 2vv,
これを入れて
(左辺) - (右辺) = (2uu+8vv)(a^3+b^3+c^3) + (-6uu+30vv)abc -2vv(a+b+c)^3
 = 2(a^3+b^3+c^3 -3abc)uu + 6F_1(a,b,c)vv   (←シューア)
 ≧ 0,

>>3723.
通分すると
(分子) = (a+s)(3a)^n + (b+s)(3b)^n + (c+s)(3c)^n
 ≧ (4s/3){(3a)^n + (3b)^n + (3c)^n}  (←チェビシェフ)
 = (4s)(3^n)(a^n + b^n + c^n)/3
 ≧ (4s)s^n,
(分母) = (a+s)(b+s)(c+s) ≦ (4s/3)^3,   (← GM-AM)
(左辺) ≧ (27/16)s^(n-2),

>>3731.
コーシーの拡張より
 (a+b+c)(a+b+c) … (a+b+c){a^(n+1) + b^(n+1) + c^(n+1)} ≧ (aa+bb+cc)^n,
    (n-1)個
0601132人目の素数さん
垢版 |
2018/06/02(土) 06:34:19.54ID:qc99k5Fr
>>598

3741.
a = ππ/{2(π-2)} = 4.322734721
 b = 6

 cos(t) < 1 を [0,x] で逐次積分すると、
 sin(x) < x,  (x>0)
-cos(x) < -1 + xx/2!,
-sin(x) < -x + (x^3)/3!,  (x>0)
 cos(x) < 1 - xx/2! + (x^4)/4!,
 sin(x) < x - (x^3)/3! + (x^5)/5!
  = {x - ((14-xx)/720)x^5}/(1+xx/6)
  < x/(1+xx/6),   (0<x<π/2)

3752.
 a_n = 1 +1/n -1/nn = (nn+n-1)/nn,
とおく。
 a_n / a_{n-1} = (n-1)^2・(nn+n-1)/{nn(nn-n-1)}
 = 1 - (nn-3n+1)/{nn(nn-n-1)}
 ≦ 1 - 1/(2nn)     (n≧5)

∵ 2(nn-3n+1) - (nn-n-1) = n(n-5) + 3 ≧ 3  (n≧5)

 (a_n / a_{n-1})^2 ≦ {1 - (1/2nn)}^2
 = 1 - 1/nn + 1/(4n^4)
 < 1 - 1/n^3,
0602132人目の素数さん
垢版 |
2018/06/02(土) 13:03:43.11ID:bqWw3wOw
正整数nと1より大きい正の実数xに対し、
Σ[k=1,n]{kx}/[kx]<Σ[k=1,n]1/(2k-1)
{kx}はkxの小数部分を表し、[kx]はkxの整数部分を表すものとする
0603132人目の素数さん
垢版 |
2018/06/05(火) 10:01:49.78ID:RI7aB28L
>>599

3763.
(左) HM-AM より
 a/(2a+b+c) ≦ (1/4){a/(a+b) + a/(a+c)},
 b/(2b+c+a) ≦ (1/4){b/(b+c) + b/(b+a)},
 c/(2c+a+b) ≦ (1/4){c/(c+a) + c/(c+b)},
辺々たすと
 (左辺) ≦ 3/4,

(右)
 a/(2b+2c) = (a+b+c)/(2b+2c) - 1/2
 b/(2c+2a) = (a+b+c)/(2c+2a) - 1/2
 c/(2a+2b) = (a+b+c)/(2a+2b) - 1/2
辺々たすと
 (右辺) = (a+b+c){1/(2b+2c) + 1/(2c+2a) + 1/(2a+2b)} - 3/2
 ≧ (a+b+c)・9/{4(a+b+c)} - 3/2 (← AM-HM)
 = 9/4 - 3/2
 = 3/4,      (Nesbitt,Shapiro-3)
0604132人目の素数さん
垢版 |
2018/06/06(水) 08:56:29.19ID:xxwxn7ab
>>593

〔Chapple - Euler の不等式〕
外接円の半径をR、内接円の半径をrとするとき
 R(R-2r) = OI^2 ≧ 0
 O:外心 I:内心
0605132人目の素数さん
垢版 |
2018/06/06(水) 17:21:28.63ID:xxwxn7ab
>>602

 x → x+1 とすれば分母が k 増えるので左辺は減少する。1≦x≦2 で考える。
 (m-1)/n ≦ {x} < m/n となるmをとる。
 m = [nx] - n[x] +1, (1≦m≦n)

〔補題〕
 Σ[k=1,n] {kx}/[kx] < Σ[k=1,m-1] 1/(2k-1) + ({x} - (m-1)/n)/(2m-1),

右辺は、(1,0) - (1+1/n,1) - (1+2/n,1+1/3) - …… - (1+m/n,Σ[k=1,m] 1/(2k-1)) - …… (2,Σ[k=1,n] 1/(2k-1)) を結んだ折れ線を表わす。
0606132人目の素数さん
垢版 |
2018/06/09(土) 21:53:12.92ID:TZRRIZyQ
bot[195]
6(x^3 + y^3 + z^3)^2 ≦ (x^2 + y^2 + z^2)^3

これはシュワちゃんと関係あるん?
0607132人目の素数さん
垢版 |
2018/06/10(日) 17:09:15.57ID:KetZUwRK
>>606 [195]

x+y+z = 0 より
 x^3+y^3+z^3 = 3xyz,
 xx+yy+zz = [(x-y)^2 + 3zz]/2,
xとyは同符号とすれば
0 ≦ 4xy ≦(x+y)^2 = zz,

(左辺) = 6(3xyz)^2 = 54(xy)(xy)(zz) ≦ (3zz/2)^3 ≦ {[(x-y)^2 +3zz]/2}^3 = (xx+yy+zz)^3.

 蕪湖市数学競技会
0609132人目の素数さん
垢版 |
2018/06/12(火) 00:18:29.69ID:BRfgTgz+
以下、x、y、z∈R とする。

(1) (x^2 + y^2 + z^2)^3 ≧ 6(x^3 + y^3 + z^3)^2
(2) (x^2 + y^2 + z^2)^3 ≧(x^3 + y^3 + z^3 - 3xyz)^2 + (ab+bc+ca)^3
(3) (x^2 + y^2 + z^2)^3 ≧ 2{(x-y)(y-z)(z-x)}^2
(4) 2(x^2 + y^2)(y^2 + z^2)(z^2 + x^2) ≧ {(x-y)(y-z)(z-x)}^2
(5) 合体 or 改造できるかな?

出典
(1) >>606、bot195、蕪湖市数学競技会
(2)(3)(4)は過去に扱ったと思うが、元ネタを記録していないので詳細不明

  ∧_∧
  ( ;´∀`) < むむむ…、我慢できないでござる!
  人 Y /   
 ( ヽ し
 (_)_)
0612132人目の素数さん
垢版 |
2018/06/12(火) 16:00:37.51ID:YFJLrlqV
>>609

(1) x+y+z=0 のとき、…

(2)
 xx+yy+zz = S2,xy+yz+zx = t,
とおく。
 S2 - t = {(x-y)^2 + (y-z)^2 + (z-x)^2}/2 ≧ 0,
(S2)^3 - t^3 = {(S2)^2 + S2・t +tt}(S2-t)
 ≧ {(S2)^2 + S2・t - 2tt}(S2-t)
 = (S2+2t)(S2-t)^2
 = (x+y+z)^2・{(xx+yy+zz) -(xy-yz-zx)}^2
 = (x^3+y^3+z^3 -3xyz)^2,

(3)
 yはxとzの中間にあるとしてよい。
 0 ≦ (x-y)(y-z) ≦ (1/4)(x-z)^2,
 xx+yy+zz = (1/2)(x+z)^2 + (1/2)(x-z)^2 + yy ≧ (1/2)(x-z)^2,
 (左辺) ≧ (1/8)(x-z)^6 ≧ 2(x-z)^2 {(x-y)(y-z)}^2 = (右辺),
0613132人目の素数さん
垢版 |
2018/06/14(木) 22:38:12.04ID:6IyvuHHw
Asia Pacific Mathematical Olympiad APMO 2004
でググって5番目あたりに出てくるPDFの Problem 5。

模範解答がワケワカメ…。
これより強い不等式を、前スレでやったような排気ガス…
0614132人目の素数さん
垢版 |
2018/06/15(金) 00:29:19.87ID:UYSEHwOg
数学セミナーエレガントな解法2月号にある不等式の問題の正解率が異様に低かったらしい
そもそも問題すら理解してない回答が多かったって講評だった
0615132人目の素数さん
垢版 |
2018/06/15(金) 02:05:18.57ID:mm39PC7P
>>609
(4)
(1-i)(x+iy)(y+iz)(z+ix) = (1-i){-(xyy+yzz+zxx-xyz) +i(xxy+yyz+zzx-xyz)}
= -(x-y)(y-z)(z-x) +i{(x+y)(y+z)(z+x)-4xyz},
絶対値の2乗をとって
 2(xx+yy)(yy+zz)(zz+xx) = {(x-y)(y-z)(z-x)}^2 + {(x+y)(y+z)(z+x) -4xyz}^2,

>>613
 [前スレ.456]
(abc)^2 +aa +bb +cc +2 -2(ab+bc+ca)≧ 0 を使う?
文献[9] 佐藤(訳)、問題3.85改、練習問題1.90(i)
0616132人目の素数さん
垢版 |
2018/06/15(金) 02:30:14.62ID:d1fXPxbR
なるほど!

>>613
x、y、z∈R のとき、(aa+2)(bb+2)(cc+2) ≧ 9(ab+bc+ca)

[前スレ.456]
x、y、z∈R のとき、(aa+2)(bb+2)(cc+2) ≧ 3(a+b+c)^2

合体!
(aa+2)(bb+2)(cc+2) ≧ 3(a+b+c)^2 ≧ 9(ab+bc+ca)
0617132人目の素数さん
垢版 |
2018/06/15(金) 02:32:47.15ID:mm39PC7P
>>609 (4) >>615
 s = x+y+z,
 t = xy+yz+zx,
 u = xyz,
  = (x-y)(y-z)(z-x),
で表わせば
 2(ss-2t)(tt-2su) -2uu = 刧 + (st-5u)^2,
0618132人目の素数さん
垢版 |
2018/06/15(金) 02:38:30.76ID:d1fXPxbR
左辺を見て、昨夏の不等式三昧の夜を思い出す ( ゚∀゚) ウヒョッ!

[前スレ.469前後]
x、y、z∈R 、k≧0 のとき、(aa+k)(bb+k)(cc+k) ≧ (3kk/4)*(a+b+c)^2 などなど…
0620132人目の素数さん
垢版 |
2018/06/17(日) 01:22:10.06ID:8Ln3gkjC
立ち読みで疎覚えだが、数蝉NOTE。

a、b、c >0、a+b+c=1 のとき、Σ[cyc] a/(b^2+bc+c^2) ≧3.
0621132人目の素数さん
垢版 |
2018/06/17(日) 01:33:38.57ID:lI+JiKnS
>>619

〔Igarashi の不等式〕
 a,b,c>0 のとき、
 a/(bb+bc+cc) + b/(cc+ca+aa) + c/(aa+ab+bb) ≧ (a+b+c)/(ab+bc+ca) ≧ 3/(a+b+c),
 2018年7月号NOTE

(略証)
 a ' = bb + bc + cc,
 b ' = cc + ca + aa,
 c ' = aa + ab + bb,
とおくと
 aa ' + bb ' + cc ' = (a+b+c) (ab+bc+ca),  … これがミソ(?)
コーシーにより
 (左辺) = a/a ' + b/b' + c/c' ≧ (a+b+c)^2 /(aa ' + bb ' + cc ') = (a+b+c)/(ab+bc+ca),
0622132人目の素数さん
垢版 |
2018/06/17(日) 01:44:34.71ID:8Ln3gkjC
>>621
おお、これだ。さんくす。
解説でZZZが一般化してたけど、なんかよく分からなかった…。
0623132人目の素数さん
垢版 |
2018/06/17(日) 01:45:36.54ID:lI+JiKnS
>>620 >>621
 被りました。

 f(x) = 1/x は下に凸だから、Jensenにより
 (左辺) = a f(a ') + b f(b ') + c f(c ')
  ≧ (a+b+c) f((aa'+bb'+cc')/(a+b+c))
  = (a+b+c) f(ab+bc+ca)
  = (a+b+c)/(ab+bc+ca),
0624132人目の素数さん
垢版 |
2018/06/17(日) 01:46:16.03ID:8Ln3gkjC
>>621
>  a ' = bb + bc + cc,
>  b ' = cc + ca + aa,
>  c ' = aa + ab + bb,
> とおくと
>  aa ' + bb ' + cc ' = (a+b+c) (ab+bc+ca),  … これがミソ(?)

この変形は初めて見た。コレクションに入れておこう。
0625132人目の素数さん
垢版 |
2018/06/17(日) 01:48:52.60ID:8Ln3gkjC
あと一松じっちゃんの不等式の解説で、s(2(s^2-2t)-5t)+27u の因数分解があったような。
立ち読みだったんで s、t、u で覚えて帰ったから怪しいが…。
手計算で因数分解しようとして挫折した。 手計算でできるのか?
0627132人目の素数さん
垢版 |
2018/06/18(月) 22:44:29.14ID:wEh7fB1P
>>622
Nesbittと合体したでござるか…

〔Nesbitt-Igarashi の不等式〕
 a,b,c>0 のとき、
 (a+b+c) {a/(bb+bc+cc) + b/(cc+ca+aa) + c/(aa+ab+bb)}
 ≧ 2 {a/(b+c) + b/(c+a) + c/(a+b)}
 ≧ (a+b+c)^2 /(ab+bc+ca)
 ≧ 3,
 数セミ、2018年7月号NOTE-改
0628132人目の素数さん
垢版 |
2018/06/19(火) 02:28:33.39ID:8eLVrD8z
>>614
よく分からぬ難しげな不等式で、攻めづらかったかも。
この式が出てきた背景は、解説で触れていたけれど。
0631132人目の素数さん
垢版 |
2018/06/19(火) 13:57:17.17ID:8eLVrD8z
>>630 [101]

a〜d>0、a+b+c+d-1=0 のとき
 6(a^3 + b^3 + c^3 + d^3) ≧ aa+bb+cc+dd + 1/8.
 フランス TeamSelectionTest-2007 Q.2

(略解)
 f(x) = 6x^3 - (xx + 1/32)
 = (5/8)(x-1/4) + 2(3x+1)(x-1/4)^2
 ≧ (5/8)(x-1/4),
より
 f(a) + f(b) + f(c) + f(d) ≧ (5/8)(a+b+c+d-1) = 0.

{x = 1/4 で接線を曳く。f '(1/4) = 5/8}
0632132人目の素数さん
垢版 |
2018/06/19(火) 17:30:30.95ID:/rZEmPAN
>>631
さんくす。4月から見てるけど、101だけ出てこないのだ。
画像のない192は頻繁に出てくるのにな。偏りすぎている。
0633132人目の素数さん
垢版 |
2018/06/20(水) 02:26:05.78ID:ZoYl55O4
>>632 [192]
 
任意の実数a,b,cに対し、
 (a-b)(a-c)(aa-bc)^2 + (b-c)(b-a)(bb-ca)^2 + (c-a)(c-b)(cc-ab)^2 ≧ 0,
を示せ。
 casphy! - highmath(高校数学) - 不等式2-188
 じゅー君が高校生のとき作ったヤツ(?)

(略証)
i)a+b+c≠0 のとき、
 A = aa-bc,B = bb-ca,C = cc-ab,
とおくと
 A-B = (a+b+c)(a-b),etc.
(左辺) = {AA(A-B)(A-C)+BB(B-C)(B-A)+CC(C-A)(C-B)}/(a+b+c)^2
 = F_2(A,B,C)/(a+b+c)^2  (←シューア)
 ≧0,
ii)a+b+c=0 のとき、
 A = B = C,
 (左辺) = AA F_0(a、b、c) ≧ 0.
これで ☆9 だって。
0634132人目の素数さん
垢版 |
2018/06/20(水) 23:16:58.87ID:ZoYl55O4
>>613 >>615

〔補題〕
a,b,c≧0 のとき
(abc)^2 +aa +bb +cc +2 -2(ab+bc+ca)≧ 0,

(略証)
a = A^(3/2),b = B^(3/2),c = C^(3/2) とおくと
 (abc)^2 + 2 -3ABC = (ABC)^3 +1 +1 -3ABC ≧ 0,  (←AM-GM)
 A(A-B)(A-C) + B(B-C)(B-A) + C(C-A)(C-B) = F1(A,B,C) ≧ 0,
 AB(A+B) -2ab = AB(√A - √B)^2 ≧ 0,etc.
辺々たす。
0638132人目の素数さん
垢版 |
2018/06/23(土) 18:12:52.09ID:BnO9HX6O
〔問題677〕

Pを凸多面体とし、Pの辺を L_1,L_2,…,L_n とする。
各 1≦i≦n について L_i を辺にもつPの2つの面を考え、
その2つの面のなす角を外側から測ったものを θ_i とする。
(2面の外向き法線のなす角。2面角)

このとき、Σ[i=1,n] θ_i ≧ 3π であることを示せ。

JMO夏季セミナー
http://jmoss.jp/mon/old.php → 第9回 (G,入江)

面白スレ26-677
0639132人目の素数さん
垢版 |
2018/06/23(土) 22:49:56.64ID:BnO9HX6O
[213]
正の実数列 {a_k} が各自然数kに対して
a_{k+1} ≧ k・a_k / {(a_k)^2 + (k-1)}
を満たすとする。すべての n≧2 に対して
a_1 + a_2 + … + a_n ≧ n,
を示せ。
 IMO Shortlist 2015 A.2 ☆2
0640132人目の素数さん
垢版 |
2018/06/23(土) 23:13:45.08ID:BnO9HX6O
>>639 [213]
nについての帰納法による。

・n=2 のとき
 a_1 + a_2 ≧ a_1 + 1/a_1 ≧ 2  (← AM-GM)

・n>2 のとき
 a_n ≧1 のときは明らかに成立つ。
 a_n ≦1 のとき 題意より
 k/a_{k+1} ≦ (k-1)/a_k + a_k,
 a_k ≧ k/a_{k+1} - (k-1)/a_k,
 k=1,…,n-1 でたす。
 a_1 + a_2 + … + a_{n-1} ≧ (n-1)/a_n,
 a_1 + a_2 + … + a_n ≧ (n-1)/a_n + a_n
  = n + (n-1 - a_n)(1 - a_n)/a_n
  ≧ (n-2) + 1/a_n + a_n
  ≧ n,   (← 0 < a_n ≦1)
0641132人目の素数さん
垢版 |
2018/06/23(土) 23:13:45.10ID:BnO9HX6O
>>639 [213]
nについての帰納法による。

・n=2 のとき
 a_1 + a_2 ≧ a_1 + 1/a_1 ≧ 2  (← AM-GM)

・n>2 のとき
 a_n ≧1 のときは明らかに成立つ。
 a_n ≦1 のとき 題意より
 k/a_{k+1} ≦ (k-1)/a_k + a_k,
 a_k ≧ k/a_{k+1} - (k-1)/a_k,
 k=1,…,n-1 でたす。
 a_1 + a_2 + … + a_{n-1} ≧ (n-1)/a_n,
 a_1 + a_2 + … + a_n ≧ (n-1)/a_n + a_n
  = n + (n-1 - a_n)(1 - a_n)/a_n
  ≧ (n-2) + 1/a_n + a_n
  ≧ n,   (← 0 < a_n ≦1)
0643132人目の素数さん
垢版 |
2018/06/24(日) 05:10:52.64ID:dz2BpZ6O
>>642
きたか…!!

  ( ゚д゚ ) ガタッ
  .r   ヾ
__|_| / ̄ ̄ ̄/_
  \/    /
0644132人目の素数さん
垢版 |
2018/06/24(日) 05:15:05.23ID:dz2BpZ6O
あとは消失した192を作り直してもらうことと、224問目以降を作ってもらうことだな
0645132人目の素数さん
垢版 |
2018/06/25(月) 23:38:45.92ID:qOAzU6BU
>>611 >>637

基本対称式を x+y+z = s,xy+yz+zx = t,xyz = u とおく。
 xx-yz = xs-t,yy-zx = ys-t,zz-xy = zs-t,
より
 (左辺) - (右辺) = (ss-2t)^3 - 8(xs-t)(ys-t)(zs-t)
 = (ss-2t)^3 - 8(us^3 - t^3)
 = ss{(ss-3t)^2 + (8/3)(tt-3su) + (1/3)tt}
 ≧ 0,
等号成立は x+y+z = 0.
0646132人目の素数さん
垢版 |
2018/06/25(月) 23:48:14.02ID:qOAzU6BU
>>611 の〔類題〕

x,y,z ∈ R のとき
-(35+13√13)/486 ≦ (xx-yz)(yy-zx)(zz-xy)/(xx+yy+zz)^3 ≦ 1/8,
 -0.1684612481

 左側等号は (x,y,z) = ((3-√13)/2,1,1) など。  -0.302775637732
0647132人目の素数さん
垢版 |
2018/06/29(金) 11:42:15.93ID:kgKL/5Ht
正の実数a,b,cはa+b+c=3を満たす。このとき、
1/(2+aa+bb)+1/(2+bb+cc)+1/(2+cc+aa)≦3/4

2009 イランTST
0649132人目の素数さん
垢版 |
2018/07/01(日) 11:44:56.53ID:o+nodY1/
>>647

左辺を f(a,b,c) とおく。
1≦c とし、(a+b)/2 = (3-c)/2 = m とおく。
 f(a,b,c) ≦ f(m,m,c) ≦ 3/4
を示す。

(左)
aa+bb ≧ 2mm より
1/(2+aa+bb) = 1/{2 +2mm +(1/2)(a-b)^2} ≦ 1/(2+2mm),
1/(2+cc+bb) + 1/(2+cc+aa) = 2{2+cc+(aa+bb)/2}/{(2+cc+bb)(2+cc+aa)}
 ≦ 2/(2+cc+mm),
∵ (2+cc+bb)(2+cc+aa) -(2+cc+mm){2+cc+(aa+bb)/2}
 = (1/4)(a-b)^2 (2+cc-3mm) + (1/16)(a-b)^4
 = (1/4)(a-b)^2 {2+cc-(3/4)(3-c)^2} + (1/16)(a-b)^4
 = (1/32)(a-b)^2 (19+c)(c-1) + (1/16)(a-b)^4
 ≧ 0,   (← c≧1)

(右)
 f(m,m,c) = 1/(2+2mm) + 2/(2+cc+mm)
 = (3/4){1 - (c-1)^2・(5cc-26c+37)/[8(2+2mm)(2+cc+mm)] }
 ≦ 3/4.
0650132人目の素数さん
垢版 |
2018/07/01(日) 14:00:30.20ID:AVymxtb0
実数x_1,x_2,…,x_nに対して次の不等式が成立することを示せ
Σ[i,j=1,n]|x_i+x_j|≧nΣ[i=1,n]|x_i|

2006 イランTST
0652132人目の素数さん
垢版 |
2018/07/02(月) 07:40:46.47ID:dZnBLmxp
>>650

x_1, x_2, …, x_p > 0,
x_{p+1}, …, x_n ≦ 0, とする。(0≦p≦n)

(左辺) = Σ[i,j=1,p] |x_i+x_j| + Σ[i,j=p+1,n] |x_i+x_j| + Σ[i=1,p][j=p+1,n] |x_i+x_j|
= Σ[i,j=1,p] (|x_i|+|x_j|) + Σ[i,j=p+1,n] (|x_i|+|x_j|) + Σ[i=1,p][j=p+1,n] |x_i+x_j|
= 2p S_p + 2(n-p) S_n + 2S~,
ここに
 S_p = Σ[i=1,p] |x_i|, S_n = Σ[j=p+1,n] |x_j|, S~ = Σ[i=1,p][j=p+1,n] |x_i+x_j|,
とおいた。

・p = n/2 のときは成立する。(S~≧0)

・0 ≦ p < n/2 のとき
 S~ ≧ Σ[i=1,p][j=p+1,n] (|x_i|-|x_j|) = (n-p) S_p - p S_n,
 0 < (n-2p)/(n-p) ≦ 1 を掛けて
 S~ ≧ {(n-2p)/(n-p)}S~ ≧ (n-2p){S_p - [p/(n-p]S_n},
 (左辺) ≧ n S_p + {n + (n-2p)^2 /(n-p)}S_n ≧ n(S_p + S_n),

・n/2 < p ≦ n のとき
 S~ ≧ Σ[i=1,p][j=p+1,n] (|x_j|-|x_i|) = -(n-p) S_p + p S_n,
 0 < (2p-n)/p ≦ 1 を掛けて
 S~ ≧ {(2p-n)/p}S~ ≧ (2p-n){-[(n-p)/p]S_p + S_n},
 (左辺) ≧ {n + (2p-n)^2 /p}S_p + n S_n ≧ n(S_p + S_n),
0653132人目の素数さん
垢版 |
2018/07/02(月) 16:23:14.90ID:dZnBLmxp
>>652 訂正

はじめの方で
(左辺) = … + … + 2Σ[i=1,p][j=p+1,n] |x_i+x_j| 
の係数2が抜けてました。(後の論証に影響ないと思いますが…)
0654132人目の素数さん
垢版 |
2018/07/03(火) 11:38:56.42ID:F6g7HQZx
>>652
混乱しているので修正

(左辺) = 2p S(+) + 2(n-p) S(-) + 2S~,
ここに
 S(+) = Σ[i=1,p] |x_i|, S(-) = Σ[j=p+1,n] |x_j|, S~ = ……
とおいた。

結論は
 (左辺) ≧ …… ≧ n{S(+) + S(-)},
0655132人目の素数さん
垢版 |
2018/07/05(木) 18:50:30.31ID:da/jl28d
非負実数a,b,c,dと1≦p≦2なる実数pに対して、次の不等式が成立することを示せ
(a+b)^p+(c+d)^p+(a+c)^p+(b+d)^p≦a^p+b^p+c^p+d^p+(a+b+c+d)^p
0658132人目の素数さん
垢版 |
2018/07/06(金) 08:14:08.28ID:Fbh8MKIz
>>37(1) >>40 >>41 >>44

〔Redhefferの不等式〕
a_1 〜 a_n >0 のとき
G_k = (a_1・a_2…a_k)^(1/k) とおくと
G_1 + G_2 + …… + G_n ≦ Σ[k=1,n] (1+1/k)^k・a_k - n・G_n,

 和書[3] (大関, 1987) p.114-115 例題3
 文献 Ray Redheffer: Proc. London Math. Soc., Vol. s3-17, Iss. 4, p.683-699 (1967/Oct)
    "Recurrent inequalities"
0659132人目の素数さん
垢版 |
2018/07/13(金) 09:38:28.60ID:/EP6VcDe
>>658

(G_{k-1},G_{k-1},…,G_{k-1},(1+1/k)^k・a_k)のk個ででAM-GM する。
  (k-1)個

 (k+1)G_k - (k-1)G_{k-1} ≦ (1+1/k)^k・a_k,

k=1〜n でたす。(便宜上、G_0=0)
0661132人目の素数さん
垢版 |
2018/07/14(土) 22:00:02.78ID:fIrZynJm
>>660

「円に外接する三角形の面積だろ!」
とかツッコミたくないが。
 
その場合は
 a = {cot(B/2)+cot(C/2)} r/2 などより、
S = {cot(A/2)+cot(B/2)+cot(C/2)} rr
 ≧ 3cot((A+B+C)/6) rr  (←下に凸)
 = 3cot(π/6) rr
 = (3√3) rr,
0662132人目の素数さん
垢版 |
2018/07/15(日) 22:13:34.38ID:8ME/vsb7
>>609 (2)
>>612 (2)

 [x,y,z] [x,z,y] [S2,t,t]
 |z,x,y| |y,x,z| = |t,S2,t|
 [y,z,x] [z,y,x] [t,t,S2]
の行列式は
 D(x,y,z)^2 = D(S2,t,t).
ここに
 D(x,y,z) = x^3 + y^3 + z^3 -3xyz
 = (x+y+z)(xx+yy+zz-xy-yz-zx)
 = (x+y+z)(S2-t).
0665132人目の素数さん
垢版 |
2018/07/17(火) 23:51:33.85ID:GyPvcBOe
>>662
F(x,y,z) は既約かつ対称な多項式で
 F(x,y,z)^2 = F(xx+yy+zz,xy+yz+zx,xy+yz+zx)
を満たすとする。

F(x,y,z) = x+y+z,
F(x,y,z) = xx+yy+zz -xy-yz=zx,
以外にも解があるかな。
0666132人目の素数さん
垢版 |
2018/07/18(水) 22:49:25.94ID:QRQo+1y+
>>655 >>656

f(x) = x^(p-1) とおくと、
x>0 で f '(x) = (p-1)x^(p-2) > 0, f "(x) = (p-2)(p-1)x^(p-3) ≧0.

f"(x) ≧ 0(下に凸)だから、(*)
 f(a+b) + f(a+c) ≦ f(a) + f(a+b+c),
 f(a+b) + f(b+d) ≦ f(b) + f(a+b+d),
 f(a+c) + f(c+d) ≦ f(c) + f(a+c+d),
f(b+d) + f(c+d) ≦ f(d) + f(b+c+d),
各式に a,b,c,d を掛けて足す。
f '(x) >0(単調増加)を使うと
 g(a+b) + g(c+d) + g(a+c) + g(b+d) ≦ g(a)+g(b)+g(c)+g(d) + g(a+b+c+d),
ここに g(x) = x・f(x)

(略証)
 0 < ∫[0,b]∫[0,c] f "(a+u+v) du dv
  = f(a+b+c) + f(a) - f(a+b) - f(a+c),
0667132人目の素数さん
垢版 |
2018/07/24(火) 00:24:54.43ID:jKOHqerG
一辺の長さが1である辺を奇数個もつ任意の多角形の面積をSとすると次の不等式が成立
S≧√3/4
0669132人目の素数さん
垢版 |
2018/07/24(火) 03:44:42.26ID:6NVSU4Ku
〔問題2018〕
a>0,b>0,c>0,a+b+c=3 のとき次を示せ。

 a^(1/2018) + b^(1/2018) + c^(1/2018) + (2/√3) √{a(1-b)+b(1-c)+c(1-a)} ≧ 3,

 (K. Chikaya, 2018/June/19)
すうじあむ //suseum.jp/gq/question/2884 を改良
 casphy.com/bbs/highmath/472060/ 不等式2-304
0671132人目の素数さん
垢版 |
2018/07/24(火) 07:01:13.36ID:jKOHqerG
>>668
偶数角形でもいい
例としては四角形のうち三つの辺が長さ1で他は長さ1ではないものだったり、一つの辺のみが1でほかは長さ1でないようなもの
0672132人目の素数さん
垢版 |
2018/07/24(火) 07:44:00.98ID:bmjGlIcJ
凸とは限らない3角形または4角形または5角形または……
であって辺の長さはすべて1であるもの
ですね。凸とは限ってないので内角が180°も桶なので>>697でもいいけど “凸とは限らない” がある方が良かったかも。
0673132人目の素数さん
垢版 |
2018/07/25(水) 00:53:28.01ID:9UVBQKrl
>>671

> 偶数角形でもいい

辺長1の正N角形の、連続する2m個の頂点を結んでできる凸2m角形を考える。(N ≫ m^3)

外接円の半径は R = 1/{2sin(π/N)},

S < (弓形の面積)
 = (扇形の面積) - (三角形の面積)
 = (1/2)RR{(4mπ/N) - sin(4mπ/N)}
 < (1/12)RR(4mπ/N)^3    (*)
 = 1/{48sin(π/N)^2}(4mπ/N)^3
 〜 (4/3) m^3 (π/N)
 → 0  (N→∞)

*) x>0 のとき x - (1/6)x^3 < sin(x) < x,
0674132人目の素数さん
垢版 |
2018/07/25(水) 01:35:01.94ID:9UVBQKrl
>>673

訂正スマソ
 (4mπ/N) → (2(2m-1)π/N)

或いは
(弦の長さ) < (2m-1)
(幅) = R {1-cos((2m-1)π/N)}
 < R (1/2) {(2m-1)π/N}^2   (**)
 = 1/{4sin(π/N)} {(2m-1)π/N}^2
 〜 (1/4)(2m-1)^2 (π/N)
 → 0   (N→∞)

**) 1 - (1/2)xx < cos(x) ≦ 1
0675132人目の素数さん
垢版 |
2018/07/25(水) 12:53:08.48ID:aSG76bcm
問題の見栄え良くするために、問題文はしょりすぎなんだよ。
偶数角形でもいいといってるのは例えば四角形ABCDで
AB=BC=CD=1、DA=2でもいいって意味だろ?
あくまで辺の長さの和は奇数。
この場合は五角形ABCDEでAE=DE=1、角Eは180°とみなして
1辺の長さ1の5角形とみなす。
そういう場合、面積は√3/4より大きくなる。

偶数角形で辺の長さ1で反例出したいなら平たいひし形で終わり。
0676132人目の素数さん
垢版 |
2018/07/25(水) 13:10:46.86ID:ia9tQLFJ
で結局問題は>>667でいいの?
真偽は別としてこれだけで問題の条件は十分伝わるよね
勝手に凸がどうたらって条件を加えてる>>672は別の問題ってことでいいの?
0677132人目の素数さん
垢版 |
2018/07/25(水) 14:13:22.87ID:h4vuL6tT
>>675 辺の長さの和が奇数とは書いてない。そもそも辺の長さは整数とは限らないし

[667(元問題)] 多角形Pは次の条件を満たすとき S >= sqrt(3) / 4
【条件】Pの辺のうち長さが1であるものは奇数個
[672] すべての辺が1である多角形Pの面積は S >= sqrt(3)/4
(凸の条件が何を言ってるのかよくわからない)
[673-4] 「辺が1の正多角形Pの面積は S >= sqrt(3)/4」を否定する証明(たぶん)

なんかごちゃごちゃしたけど問題は667でいいんだよね
0678132人目の素数さん
垢版 |
2018/07/25(水) 15:17:43.00ID:9UVBQKrl
>>675 >>677
 すべての辺の長さを自然数に限定?
 >>673 >>674 の例で、辺長1を固定しつつ端の2点を持って引っぱると、
 自然数にならぬか…

>>675
 菱形だと長さ1の辺が4つになる。 3辺長を1に固定して1点をずらす。
0679132人目の素数さん
垢版 |
2018/07/25(水) 17:44:43.59ID:LF80ki5t
点列P0.‥Pnは以下を満たす。
・nは奇数、P0=Pn
・隣接2点間の距離は1
・点列を順に結んで得られる曲線は単純閉曲線C
この時、Cで囲まれる領域の面積は√3/4以上であることを示せ。
ですな
0681132人目の素数さん
垢版 |
2018/07/25(水) 19:07:44.57ID:LF80ki5t
>>667
これは成り立つ。
私は>>667もこの意味だと思う。
長さ1の辺が奇数個でそうでない辺がいくらあっても桶
みたいな設定で何かいえると思えない。
0682132人目の素数さん
垢版 |
2018/07/25(水) 20:08:15.16ID:ia9tQLFJ
>>681
問題が間違えてるってことね
679っぽい状況は数オリ辞典かなんかで見た記憶あるけど思い出せない
0685132人目の素数さん
垢版 |
2018/08/03(金) 04:53:25.06ID:tRRMlHHD
>>520 (B3) [100]
 49th IMO spain 2008, SL-A7

s = a+b+c+d,
p = s+a+c,
q = s+b+d,
M = (s-d)(s-b) = (s+a+c)s + bd,
N = (s-a)(s-c) = (s+b+d)s + ac,
W = (b+d)M-(a+c)N = bd(b+d) - ac(a+c),   …(3)
とおく。
2(左辺) = p(a-c)^2 /M + 3(a-c)(b-d)W/MN + q(b-d)^2 /N,
これは a-c,b-d の斉2次式なので、判別式(Hessian)を調べる。
pq = 2ss + (a+c)(b+d) > 2ss,
MN = {(s+a+c)s+bd} {(s+b+d)s+ac}
 = (s+a+c)(s+b+d)ss + ac(s+a+c)s + bd(s+b+d)s + abcd
 > 2s^4 + 2ac(a+c)s + 2bd(b+d)s,  (← s>a+c,s>b+d)
辺々掛けて
4pqMN > 8ssMN
 > 16(s^3){s^3 + ac(a+c)+bd(b+d)}
 > 192{ac(a+c)+bd(b+d)}^2   {← s^3 > 3ac(a+c)+3bd(b+d)}
 > 192{bd(b+d)-ac(a+c)}^2
 = 192WW
 ≧ 9WW.
∴ 判別式(Hessian) < 0
∴ 正定値。

http://www.imo-official.org/problems/IMO2008SL.pdf
 IMO-2008, SL-A7, Solution-2
0686132人目の素数さん
垢版 |
2018/08/03(金) 05:18:52.07ID:tRRMlHHD
>>685 訂正

M = (s-d)(s-b) = (a+c)s + bd,
N = (s-a)(s-c) = (b+d)s + ac,

MN = {(a+c)s+bd} {(b+d)s+ac}
 > {ac(a+c) + bd(b+d)}s,

4pqMN > 8ssMN
 > 8(s^3){ac(a+c) + bd(b+d)}
 > 8ac(a+c)^4 + 8bd(b+d)^4
 ≧ 32{ac(a+c)}^2 + 32{bd(b+d)}^2
 > 32{bd(b+d) - ac(a+c)}^2
 = 32WW
 ≧ 9WW,
0687132人目の素数さん
垢版 |
2018/08/12(日) 09:29:55.12ID:QnRFj99l
〔問題670〕
nを自然数、xを実数とするとき
 [nx] ≧ Σ(k=1,n) [kx]/k
を示せ。ただし [x] はガウス記号である。

[面白スレ26-670,同27-144]
0688132人目の素数さん
垢版 |
2018/08/19(日) 09:59:40.08ID:nbZP1PKM
すうじあむの解答見てきたけど、h(t)の最大値を求めるところまでは分かった。
で、h(t)の最大値がf(x,y)の最大値になるのは明らかなん?
バラバラに動く変数を1変数に置き換えたものを調べて間違いないん?
0690132人目の素数さん
垢版 |
2018/08/19(日) 17:14:39.44ID:oIedIwUK
>>688
f(x,y) の最大値 ≦ h(t) の最大値 = e^{-2} で
f(1,1) = e^{-2}.
からそう結論しました。
もしも反例が見つかったら晒してください。遠慮はいりません。

>>689
たぶんこれ。
Find the maximum value of the following function for all positive real numbers x,y.
f(x,y) = e^(-x-y) {ln(x)+ln(y)+1}.

http://suseum.jp/gq/question/2901
0692132人目の素数さん
垢版 |
2018/08/29(水) 17:19:12.01ID:7MjvDqGy
>>691
そのサイトFlashがないと読めないみたい。
もうこのご時世Flashないと読めないサイトわざわざ見る気になれん。
0693132人目の素数さん
垢版 |
2018/08/29(水) 20:45:48.67ID:2P4d/CfT
>>692
つ Chrome
0694132人目の素数さん
垢版 |
2018/08/29(水) 23:01:07.06ID:gzTBhfMR
>>692

「三次方程式の解の素朴な性質」 Q.2876
a, b, c を任意の複素数とする。 3次方程式 z^3 + az^2 + bz + c = 0 の解αで
 | 2bα + 3c | ≦ | 3α^3 |
をみたすものが存在することを示してください。
 (2018/04/01 アンドロメダ)
0695132人目の素数さん
垢版 |
2018/09/07(金) 00:33:08.04ID:KiGUc8ne
>>520 (B3)
>>524

まづ 左辺を a-c, b-d の斉2次式で表わす。
 2 (左辺) = F (a-c)^2 + H (a-c)(b-d) + G (b-d)^2,
ここに
 F = 1/(c+d+a) + 1/(a+b+c),
 G = 1/(b+c+d) + 1/(d+a+b),
 H = 3{ -(a+c)/(c+d+a)(a+b+c) + (b+d)/(d+a+b)(b+c+d)},
とおいた。この斉2次式が正定値となる条件は,
 (判別式) = HH - 4FG < 0,
そこで F, G, H を評価する。
AM-HM より
 F = 1/(c+d+a) + 1/(a+b+c) ≧ 4/{2(a+c)+(b+d)},
 G = 1/(b+c+d) + 1/(d+a+b) ≧ 4/{(a+c)+2(b+d)},
∴ FG ≧ 16/{2(a+c)^2+5(a+c)(b+d)+2(b+d)^2} ≧ 16/{(9/4)(a+b+c+d)^2} > 7/(a+b+c+d)^2,

0 < (a+c)/(c+d+a)(a+b+c) = (a+c)/{(a+c)(a+b+c+d)+bd} < 1/(a+b+c+d),
0 < (b+d)/(b+c+d)(c+d+a) = (b+d)/{(b+d)(a+b+c+d)+ac} < 1/(a+b+c+d),
∴|H| < 3/(a+b+c+d)、

以上により (判別式) = HH - 4FG < 0 したがって左辺は正定値。

 IMO-2008 Short list A.7
 不等式bot(@inequalitybot) [100]  ☆12
 面白スレ26-535,961  面白スレ27-354,356
//www.casphy.com/bbs/highmath/472060/126 (7), 311
0696132人目の素数さん
垢版 |
2018/09/10(月) 16:31:37.54ID:EoRvG7WQ
正の数 a, b が a+b=1をみたすとき、任意の正の数 x, y に対して ax+by ≧ x^a y^b.

これはAM-GMの一般化でござるかな?
0700132人目の素数さん
垢版 |
2018/09/11(火) 07:28:10.88ID:cF4T1n2w
>>699

(a-1)x + (b-1)y +(c-1)z = 0 をみたすとき

a log(a) = -a log(1/a) ≧ -a(1/a -1) = a-1,
b log(b) ≧ b-1,
c log(c) ≧ c-1,
ax log(a) + by log(b) + cz log(c) ≧ (a-1)x + (b-1)y + (c-1)z = 0,
0701132人目の素数さん
垢版 |
2018/09/11(火) 11:30:17.92ID:J1j+ih4S
>>700
むむむ…、さすがでござるな。

>>696
正の数 a, b が a+b=1をみたすとき、任意の実数 x, y に対して (ax+by)(ay+bx) ≧ xy.
0702132人目の素数さん
垢版 |
2018/09/11(火) 12:34:01.15ID:cF4T1n2w
>>701

(ax+by)(ay+bx) = (a+b)^2・xy + 2ab(x-y)^2 ≧ (a+b)^2・xy,

またはコーシーで
(ax+by)(ay+bx) ≧ (a√xy + b√xy)^2 = (a+b)^2・xy,
0703132人目の素数さん
垢版 |
2018/09/11(火) 13:50:37.80ID:J1j+ih4S
>>702
むむむ…、さすがでござるな。
一つ目の解法の式変形は思いつかぬ…。
ただ、係数の2は不要ですな。
0704132人目の素数さん
垢版 |
2018/09/11(火) 18:22:10.58ID:J1j+ih4S
非負実数 a,b,c に対して、
(a-b)(a-c)a^4 + (b-c)(b-a)b^4 + (c-a)(c-b)c^4 ≧ 5{(a-b)(b-c)(c-a)}^2.

不等式の秋でござるな。(AA略)
0706132人目の素数さん
垢版 |
2018/09/13(木) 12:38:23.72ID:FJXqJvMw
>>705
 非負実数に限れば。

 偶数次の Schur はすべての実数で成立つが、>>704 は (a, b, c) = (a, 0, -a) で不成立。
0708132人目の素数さん
垢版 |
2018/09/14(金) 01:54:32.08ID:E4BddXGB
>>704
(左辺)-(右辺)
= F_4 - 5Δ^2
= s^6 - 7s^4t + 28s^3u + 8s^2t^2 - 112stu + 16t^3 + 63u^2

苦しいでござる。
別の方法を考えた方がいいか…。
0709132人目の素数さん
垢版 |
2018/09/14(金) 04:17:45.76ID:E4BddXGB
去年の秋にやっていたΔがらみの不等式が、このスレを (c-a) で検索するとたくさん出てくる。
それらの中にない(と思う)ものを見つけたのでメモ。

a, b, c > 0に対して、
(a^2 + 3b^2)(b^2 + 3c^2)(c^2 + 3a^2) ≧ {(a-b)(b-c)(c-a)}^2.

https://artofproblemsolving.com/community/c6h1595851p9908733

リンク先の模範解答を見る限りでは、任意の実数で成り立っているんじゃないの?
0710132人目の素数さん
垢版 |
2018/09/14(金) 05:11:17.74ID:0c+5G0AL
>>704

bはaとcの中間にあるとする。
(左辺) = (a^n)(a-b)^2 + (a^n-b^n+c^n)(a-b)(b-c) + (b^n)(b-c)^2,

(a,b,c) を(間隔を変えず一斉に)d減らしたとき、a^n, c^n は減少する。(0<d≦a,b,c)
では a^n -b^n +c^n はどうか?

Max{a,c} = M, min{a,c} = m とおくと
M^n - b^n = (M-d)^n - (b-d)^b + ∫[0,d] n{(M-d+t)^(n-1) - (b-d+t)^(n-1)} dt
   ≧ (M-d)^n - (b-d)^b,
m^n ≧ (m-d)^n,       (0<d≦m)
辺々たして
 M^n -b^n +m^n ≧ (M-d)^n -(b-d)^n +(m-d)^n,
 a^b -b^n +c^n ≧ (a-d)^n -(b-d)^n +(c-d)^n,
すなわち a^n -b^n +c^n も減少する。
よって(左辺)は減少するから、c=0 の場合に成立てば十分である。

(左辺) ≧ (a-b)a^(n+1) - (a-b)b^(n+1)
   = (a-b)^2・{a^n+a^(n-1)・b+……+a・b^(n-1)+b^n}
   ≧ (n+1)(a-b)^2・(ab)^(n/2)       (AM-GM)
   = (右辺),
0711132人目の素数さん
垢版 |
2018/09/14(金) 06:40:30.61ID:E4BddXGB
おお! なるほど! かたじけない!

       |
   \  __  /
   _ (m) _ピコーン
      |ミ|
    /  `´  \
     (゚∀゚ )
     ノヽノヽ
       くく
0712132人目の素数さん
垢版 |
2018/09/14(金) 13:35:31.86ID:0c+5G0AL
>>709

 (aa+3bb)(bb+3cc)(cc+3aa) ≧ {(a+b)(b+c)(c+a)}^2,

左辺は a,b,c の符号によらない。
a,b,c の符号だけを変えたとき、右辺が最も大きいのは a,b,c が同符号のもの。
∴ a,b,c >0 に対して成立てば十分。

(左辺)/(右辺) = (aa+3bb)/(a+b)^2・(bb+3cc)/(b+c)^2・(cc+3aa)/(c+a)^2 = f(a/b) f(b/c) f(c/a),
ここに f(x) = (xx+3)/(x+1)^2,

a,b,c >0 ⇒ f(a/b) f(b/c) f(c/a) ≧ 1 を示す。

(1) a/b, b/c, c/a の1つが 0 < x ≦ (-16+√333)/7 = 0.321184 にあるとき。

 [4f(x)-3](x+1)^2 = 4(xx+3) -3(x+1)^2 = (x-3)^2 ≧ 0,
∴ f(x) の最小値は f(3) = 3/4
 f(x) ≧ (4/3)^2 となるものが1つでもあれば 成立する。
 その条件は [16f(x)-9](x+1)^2 = 16(x+1)^2 -9(xx+3) = 7xx +32x -11 ≦ 0,
 -4.8926125 = (-16-√333)/7 ≦ x ≦ (-16+√333)/7 = 0.321184

(2) a/b, b/c, c/a ≧ (-16+√333)/7 = 0.321184 のとき。
 x ≧ (-16+√333)/7 = 0.321184 のとき
 x(x+1)^4 - (xx+3)^2 = (x^3 +x^2 +3x-1)(x-1)^2 ≧ 0,
∴ f(x) ≧ 1/√x,
∴ f(a/b) f(b/c) f(c/a) ≧ √(b/a) √(c/b) √(a/c) = 1,

以上により成立つ。

>>710 訂正
(左辺) = (a^n)(a-b)^2 + (a^n-b^n+c^n)(a-b)(b-c) + (c^n)(b-c)^2,
0713132人目の素数さん
垢版 |
2018/09/14(金) 13:51:02.18ID:0c+5G0AL
>>712 訂正スマソ

その条件は [16-9f(x)](x+1)^2 = …

x(xx+3)^2 - (x+1)^4 = (x^3 +x^2 +3x-1)(x-1)^2 ≧ 0,
0715132人目の素数さん
垢版 |
2018/09/16(日) 05:37:44.55ID:dbi3zUMB
>>714
「EMV inequality」でググると、一番上に
A large sieve inequality of Elliott-Montgomery-Vaughan type for automorphic forms and two applications
というのがヒットするんだけど、さっぱり分からん…。
0716132人目の素数さん
垢版 |
2018/09/16(日) 06:51:57.72ID:liVS5BiP
>>714

http://artofproblemsolving.com/community/c6h205183p1130901
の Theorem 1 でござるな。n=3 のときは

〔EMV定理〕
f(x, y, z): R^3 → R は連続で C^1 級函数とする。次の2つの条件
 (i) xyz = 0 ⇒ f(x, y, z) ≧ 0,
 (ii) x, y, z ≧ 0 ⇒ ∂f/∂x + ∂f/∂y + ∂f/∂z ≧ 0,
を同時に満たすならば
 x, y, z ≧0 ⇒ f(x, y, z) ≧ 0.

Example 3. (Suranyi) >>512-513
Problem 1. (Schur, n=1) >>514
Problem 2. (Turkevici) >>163-164, 185, 530-531 (一般化 >>189)
Problem 6. >>486-487, 492
0717132人目の素数さん
垢版 |
2018/09/16(日) 09:55:39.99ID:liVS5BiP
>>709 >>712

{a+b√(-3)}{b+c√(-3)}{c+a√(-3)} = -(3q +8abc) + p√(-3),

ここに p = aab+bbc+cca -3abc,q = abb+bcc+caa -3abc,

(左辺) - (右辺) = (aa+3bb)(bb+3cc)(cc+3aa) - {(a+b)(b+c)(c+a)}^2

 = (3q+8abc)^2 + 3pp - (p+q+8abc)^2

 = 2pp -2pq +8qq +16abc(2q-p)

 = 2pp -2pq +(8/3)qq + (16/3) {(q+3abc)^2 -3abc(p+3abc)}

 = 2pp -2pq +(8/3)qq + (16/3) {(abb+bcc+caa)^2 -3caa・abb -3abb・bcc -3bcc・caa}

 = 2pp -2pq +(8/3)qq + (8/3) {[a(ca-bb)]^2 + [b(ab-cc)]^2 + [c(bc-aa)]^2}

 ≧ 0,

∵ (x+y+z)^2 - 3(xy+yz+zx) = (xx+yy+zz) - (xy+yz+zx) = {(x-y)^2+(y-z)^2+(z-x)^2}/2,
0719132人目の素数さん
垢版 |
2018/09/16(日) 11:16:21.07ID:dbi3zUMB
[三角形の辺長 a,b,c に関するアレ]

(1) abc ≧ (a+b-c)(b+c-a)(c+a-b).
(2) (a^a)(b^b)(c^c) ≧ (a+b-c)^a (b+c-a)^b (c+a-b)^c.
(3) (a+b-c)(b+c-a)(c+a-b) + (a+b)(b+c)(c+a) ≧ 9abc.

(1)は、不等式好きなら誰でも知っているレムスの不等式。
(2)は、上の上の不等式ヲタなら やはり常識である不等式。
(3)が、今回ご紹介する商品。

この他に a+b-c、b+c-a、c+a-b がらみの不等式があれば紹介してクリリン。
0721132人目の素数さん
垢版 |
2018/09/17(月) 01:23:24.03ID:iDwWzM3i
>>719
△なのでRavi変換する。
 x = b+c-a,
 y = c+a-b,
 z = a+b-c,
とおくと
 x+y+z = a+b+c,

(1)
AM-GM で
 a = (y+z)/2 ≧ √(yz),
 b = (z+x)/2 ≧ √(zx),
 c = (x+y)/2 ≧ √(xy),
より
 abc = (y+z)(z+x)(x+y)/8 ≧ xyz,

a,b,c ≧ 0 のとき
 abc - (a+b-c)(b+c-a)(c+a-b) = F_1(a,b,c) ≧ 0,

(2)
 log(左辺) = a log(a) + b log(b) + c log(c)
  ≧ y log(a) + z log(b) + x log(c)   (←チェビシェフ)
  ≧ (y/2)log(yz) + (z/2)log(zx) + (x/2)log(xy)
  = (y+z)/2 log(z) + (z+x)/2 log(x) + (x+y)/2 log(y)
  = a log(z) + b log(x) + c log(y)
  = log(右辺),

(3)
 (左辺) = (2x)(2y)(2z) + (2x+y+z)(x+2y+z)(x+y+2z) = F1(x,y,z) ≧ 0,
0722132人目の素数さん
垢版 |
2018/09/17(月) 20:35:16.00ID:iDwWzM3i
>>721 (2)
 チェビシェフは不成立でした。スマソ

 log(a+b-c) = log(a) + log{1 +(b-c)/a} ≦ log(a) + (b-c)/a,
 a log(a+b-c) ≦ a log(a) +b -c,
巡回的にたす。
0723132人目の素数さん
垢版 |
2018/09/28(金) 16:36:49.64ID:gDbOCyp+
自然数 k,n (k<n)に対して、(n/k)^k ≦ nCk ≦ (en/k)^k を示せ。
ここで e はネイピア数。
0724132人目の素数さん
垢版 |
2018/09/30(日) 05:30:25.69ID:60e7kxgM
>>723
左側:
C[n, k] = Π[j=0, k-1] (n-j)/(k-j) > Π[j=0, k-1] (n/k) = (n/k)^k,

右側: 補題より
C[n, k] = n(n-1)…(n-k+1)/k! < (n^k)/k! < e^(k-1)・(n/k)^k,

〔補題〕
k≧2 のとき (k^k)/k! < e^(k-1),

(略証)
(1 +1/j)^j = Σ[L=1, j] C[j, L](1/j)^L = Σ[L=1, j] (1-1/j)(1-2/j)…(1-(L-1)/j)/L!
はjについて単調増加。
∴ {(j+1)/j}^j = (1 + 1/j)^j < e,
j=1,…,k-1 を入れて掛けると
 (k^k)/k! < e^(k-1),

(別法)
マクローリン展開から
 e^x > x^{k-1} /(k-1)! + (x^k)/k! + x^{k+1} /(k+1)!
   = (x^k)/k! {(k/x) + 1 + x/(k+1)},
 e^k > (k^k)/k! {2 + k/(k+1)} > (k^k)/k! e,   (k≧3)
∴ e^{k-1} > (k^k)/k!,
k=2 は直接確かめる。   (終)
0725132人目の素数さん
垢版 |
2018/09/30(日) 07:21:11.36ID:9qTOp1VD
R^n上の対称行列Tが任意のx∈R^nに対して(x,Tx)≧0を満たす時、T≧0と定義する
又、対称行列U,Vに対してU-V≧0の時、U≧Vと定義する
この時、以下について答えよ

(1)R^n上の任意の対称行列T≧0に対し、T=U^2となる対称行列U≧0が一意に存在する事を示せ(尚、この時、U=√Tと定義する)

(2)R^n上の任意の対称行列A,B≧0に対し、A+B≧2√(AB)の真偽を答え、真ならば証明を、偽ならば反例を挙げよ
0726学術
垢版 |
2018/09/30(日) 12:30:06.76ID:L25jHE+s
未踏士気 ☯
0727学術
垢版 |
2018/09/30(日) 12:56:48.99ID:L25jHE+s
ゴルフ行こうよ。永遠の−0テンプルバンカーショット。ナイトゴルフ。SWVPW。
0733132人目の素数さん
垢版 |
2018/10/01(月) 00:50:29.88ID:eM2YcEDk
>>724 の補題を改良

〔補題'〕
k≧2 のとき (k^k)/k! < e^(k-1) < (k^k)/(k-1)!

(略証)
(1 -1/jj)^j > 1 -1/j,   … AM-GM
(1 +1/j)^j = (1 -1/jj)^j /(1 -1/j)^j > 1/(1 -1/j)^(j-1) = {1 +1/(j-1)}^(j-1),
∴ (1 +1/j)^j = {(j+1)/j}^j はjについて単調増加
∴ {(j+1)/j}^j < e,
j=1,2,…,k-1 を入れて掛けると
 (k^k)/k! < e^(k-1),

{jj/(jj-1)}^j > (1 +1/jj)^j > (1 +1/j),   … AM-GM
∴ {j/(j-1)}^j = {jj/(jj-1)}^j・(1 +1/j)^j > (1+1/j)^(j+1)
∴ (1 +1/j)^(j+1) = {(j+1)/j}^(j+1) はjについて単調減少
∴ {(j+1)/j}^(j+1) > e,
j=1,2,…,k-1 を入れて掛けると
 (k^k)/(k-1)! > e^(k-1),

分かスレ447-448
0734132人目の素数さん
垢版 |
2018/10/01(月) 08:28:02.86ID:eM2YcEDk
>>725 (1)

Tのn個の固有値d_j を主対角線に並べた実対角行列を D とし、
対応する固有ベクトルw_j を各列に並べた行列をWとする。
 T w_j = w_j d_j,
 T W = W D,
n個の固有ベクトルw_jが1次独立のとき |W|≠0 で Tは対角化可能。
 T = W D W^(-1),
T≧0 すなわち Tの固有値がすべて非負のとき、Dの対角要素が非負で、√Dも実対角行列。
 T = W D W^(-1) = {W √D W^(-1)}^2 = U^2,

Tが実対称行列のときは、固有ベクトルを適当に選んでWを実直交行列にとれる。
 W^(-1) = W~
0735132人目の素数さん
垢版 |
2018/10/01(月) 08:34:53.76ID:jOqQMyBJ
>>732
訂正
A+B≧√2(AB+BA)は成り立つかでした
0736132人目の素数さん
垢版 |
2018/10/01(月) 08:35:39.23ID:jOqQMyBJ
√{2(AB+BA)}っすな
0737132人目の素数さん
垢版 |
2018/10/02(火) 21:43:18.74ID:6WqYSECx
a, b, c >0 に対して、
a/{b(b+c)^2} + b/{c(c+a)^2} + c/{a(a+b)^2} ≧ 9/{4(ab+bc+ca)}

今年も不等式の秋が来ましたな。
9/{4(ab+bc+ca)} の出てくる不等式は過去スレで扱ったな。
0738132人目の素数さん
垢版 |
2018/10/02(火) 22:15:34.85ID:6WqYSECx
x, y ∈ R に対して、

(1) 1/(x+1)^2 + 1/(y+1)^2 ≧ 1/(xy+1)

(2) a^3 + b^3 + c^3 - 3abc ≦ (a^2 + b^2 + c^2)^(3/2)
0739132人目の素数さん
垢版 |
2018/10/02(火) 22:20:09.91ID:6WqYSECx
a, b, c > 0 に対して、

(1) 3 + √{(a^2 + b^2 + c^2)(1/a^2 + 1/b^2 + 1/c^2)} ≧ (2/3)(a+b+c)(1/a + 1/b + 1/c)

(2) √{(a^4 + b^4 + c^4)(1/a^4 + 1/b^4 + 1/c^4)} ≧ 1 + √[1 + √{(a^5 + b^5 + c^5)(1/a^5 + 1/b^5 + 1/c^5)}]

(3) a^4/(a^3 + b^3) + b^4/(b^3 + c^3) + c^4/(c^3 + a^3) ≧ (a+b+c)/3

(4) {(a-b)/c}^2 + {(b-c)/a}^2 + {(c-a)/b}^2 ≧ (2√2)*{(a-b)/c + (b-c)/a + (c-a)/b}

(5) a/{√(2b^2+2c^2)} + b/(c+a) + c/(a+b) ≧ 3/2

(6) a+b+c=3 のとき、44 ≧ (a^2+2)(b^2+2)(c^2+2) ≧ 27



参考 (2) https://artofproblemsolving.com/community/q1h1328831p7152622
0740132人目の素数さん
垢版 |
2018/10/02(火) 22:39:23.54ID:6WqYSECx
むかし立ち読みした本に、不等式の証明を行列を使ってやっていたんだけど、どんな本を検索したら見つかりますかね?
0741132人目の素数さん
垢版 |
2018/10/03(水) 03:42:16.06ID:7h2ip4rW
>>737
9/{4(ab+bc+ca)} の出てくる不等式…

〔問題〕
a,b,c > 0 に対して
 1/(b+c)^2 + 1/(c+a)^2 + 1/(a+b)^2 ≧ 9/{4(ab+bc+ca)},

イランMO-1996
Inequalitybot [148]

>>738
(1)
 (x, y) = (2 -1/n, -1/2),
 1/(xy+1) = 2n,

(2)
 a+b+c = s, ab+bc+ca = t とおく。
 |a^3+b^3+c^3-3abc| = |a+b+c| (aa+bb+cc-ab-bc-ca)
 = |s| (ss-3t)
 ≦ (ss-2t)^(3/2)      (← GM-AM)
 = (aa+bb+cc)^(3/2),

*) ss≧0, ss-3t≧0 より、AM-GM で
 (ss-2t)^3 - ss(ss-3t)^2 = (3ss -8t)tt = (1/3){8(ss-3t) +ss}tt ≧ 0,
0742132人目の素数さん
垢版 |
2018/10/03(水) 16:46:30.48ID:7h2ip4rW
>>738 (2) を改造^^

a,b,c∈R に対して
 | a^3+b^3+c^3 - 3abc |^2 ≦ (aa+bb+cc)^3 - (ab+bc+ca)^3,

(略証)
 s = a+b+c, t = ab+bc+ca とおく。
 (ss-2t)^3 - t^3 - ss(ss-3t)^2 = 3(ss-3t)tt ≧ 0,
 (左辺) = ss(ss-3t)^2 ≦ (ss-2t)^3 - t^3 = (右辺),
0743132人目の素数さん
垢版 |
2018/10/03(水) 16:56:34.73ID:7h2ip4rW
>>738 (2) を改造^^

a,b,c∈R に対して
 | a^3+b^3+c^3 - 3abc |^2 ≦ (aa+bb+cc)^3 + 8(ab+bc+ca)^3,

(略証)
 s = a+b+c, t = ab+bc+ca とおく。
 (ss-2t)^3 + (2t)^3 - ss(ss-3t)^2 = 3sstt ≧ 0,
 (左辺) = ss(ss-3t)^2 ≦ (ss-2t)^3 + (2t)^3 = (右辺),
0746132人目の素数さん
垢版 |
2018/10/04(木) 02:15:28.34ID:wFWA09/F
>>739
(3)
a^4 - (a^3+b^3)(a-kb) = {k(a^3+b^3) -abb} b
 = {k[a^3 +(1/2)b^3 +(1/2)b^3] -abb} b
 ≧ {3k/(2^(2/3)) -1} ab^3,    (AM-GM)
(係数) ≧0 より
 k = (1/3)・2^(2/3) = 0.529133684

 a^4/(a^3 + b^3) ≧ a - kb,
循環的にたす。
 (左辺) ≧ (1-k)(a+b+c) = 0.470866316 (a+b+c).
0747132人目の素数さん
垢版 |
2018/10/04(木) 05:59:28.01ID:wFWA09/F
>>742>>609 (2), >>612 にござる。

>>739 (6) 右側 は >>616 >>618

 (aa+2)(bb+2)(cc+2) = uu + 2(tt-2su) + 4(ss-2t) + 8
 = (uu+1+1) + (2/3)(t-3)^2 + (4/3)(tt-3su) + (ss-4t) + 3ss
 ≧ 3ss,

※ (uu+1+1) + (ss-4t) ≧ 3u^(2/3) + {F1(a,b,c)-9u}/s
 = 3{u^(2/3) -3u/s} + F1(a,b,c)/s
 ≧ 0,
0748132人目の素数さん
垢版 |
2018/10/05(金) 00:10:36.73ID:7iOX1iCn
>>618 >>739 (6) >>747

a,b,c ≧ 0, a+b+c ≦ √(8k) のとき
 kk{(a+b+c)^2 +k} ≧ (aa+k)(bb+k)(cc+k) ≧ (3kk/4)(a+b+c)^2,

左側は
 a+b+c ≦ √(8k) より
 ab ≦ (1/4)(a+b)^2 ≦ 2k,
 (a+b)c ≦ (1/4)(a+b+c)^2 ≦ 2k,

 (aa+k)(bb+k) = k{(a+b)^2 +k} - ab(2k-ab) ≦ k{(a+b)^2 +k},

∴ (aa+k)(bb+k)(cc+k) ≦ k{(a+b)^2 +k}(cc+k) = kk(ss+k) -k(a+b)c{2k-(a+b)c} ≦ kk(ss+k),
0749132人目の素数さん
垢版 |
2018/10/05(金) 06:07:13.55ID:7iOX1iCn
>>741 (上)

4(ab+bc+ca){1/(b+c)^2 + 1/(c+a)^2 + 1/(a+b)^2} - 9
= {ab(4aa+7ab+4bb)(a-b)^2 + bc(4bb+7bc+4cc)(b-c)^2 + ca(4cc+7ca+4aa)(c-a)^2 + (2abc)F_1(a,b,c)}/{(a+b)(b+c)(c+a)}^2
= {4t・F_2+(3tt/s)F_1+(9tu/s)F_0+(st-9u)u} / (st-u)^2
≧ 0,

F_n (a,b,c) = (a^n)(a-b)(a-c) + (b^n)(b-c)(b-a) + (c^n)(c-a)(c-b) ≧ 0,
0751132人目の素数さん
垢版 |
2018/10/07(日) 09:38:24.95ID:0iw9KjaT
さあ、はじめようか?
>>737の左辺は、どこに挟まるのでござるかな?

{a/(2bc)}^2 + {b/(2ca)}^2 + {c/(2ab)}^2
≧ 1/(4a^2) + 1/(4b^2) + 1/(4c^2)
≧ 1/(4ab) + 1/(4bc) + 1/(4ca)
≧ 1/(b+c)^2 + 1/(c+a)^2 + 1/(a+b)^2 ← (>>741, >>749)
≧ 9/{4(ab+bc+ca)}
≧ 1/{(a+b)(b+c)} + 1/{(b+c)(c+a)} + 1/{(c+a)(a+b)}
≧ 9/{(a+b)(b+c) + (b+c)(c+a) + (c+a)(a+b)}
≧ 27/{4(a+b+c)^2}
≧ 9/{(a+b)^2 + (b+c)^2 + (c+a)(a+b)^2}
≧ 9/{4(a^2 + b^2 + c^2)}

 " ;ヾ ; ;";ヾ; ;"/" ; ;ヾ ;ヾ;ヾ ; ;ヾ ; ; ヾ ;ゞ  " ;ヾ ; ;";ヾゝゝ" ;ヾゞ           ヽ            /
,." ;ヾ ; ;";ヾ; ;"/" ; ;ヾ ;ヾ;ヾ ; ;ヾ ; ; ヾ ;ゞ  " ;ヾ ; ;";ヾゝゝ" ;ヾ ; ; ヾ ;ゞ;        \        /
 ゞヾ ; ;" ; ; ;; ;"iiiiii;;;;;::::: :)_/ヽ,.ゞ:,,ヾゞヾゞ__;::/        `      `        `   ー ─ ' `
   ゞヾゞ;\\iiiiii;;;;::::: :|;:/ヾ; ;ゞ "ゝゞ ; ;`
 " ;゛ ; ;" ; ;ゞ "|iiiiii;;;;::: : |:/ ヾゞ        `         `      ` `
  `      ,|i;iiiiiii;;;;;;::: :| `    `         `     `      ` `   `
        ,|iiii;iiii;;;;:;_ _: :| ___  不等式の秋    `        `        `,
   `    |iiiiiii;;;;;;((,,,):::|/  ≧ \                    ヾ从//"
    `   |iiiiiiii;;;;ii;;;;;;;;::|::::: (● (● |           `  ゙  `    ヾ'./"
         |iiiiii;iii;;;;i;;:: ::::|ヽ::::......ワ...ノ                 ○     .||.       ,
    `   |iii;;iiiii;::;:;;;;::::::| ( つ且 ~      `              ○○   | |
  , , .,.. ,..M|M|iMii;;ii:i;;i:i;:; ゝ つつ.,.. ,...... ,.... ,,,.,.. ,.... ,,,.,.. ,..,,,,.,...,..,.,| ̄ ̄|,.,..(  ).. ,,,..,,.. ,.... ,,,.,...,.. .. ,.... ,,,.,.. ,.... ,,,
0752132人目の素数さん
垢版 |
2018/10/07(日) 10:33:46.31ID:0iw9KjaT
a, b, c > 0に対して、
(a^2 + 3b^2)(b^2 + 3c^2)(c^2 + 3a^2) ≧ {(a-b)(b-c)(c-a)}^2 ←(>>709-710)

a, b, c > 0に対して、
(a^2 + ab + b^2)(b^2 + bc + c^2)(c^2 + ca + a^2) ≧ {(a-b)(b-c)(c-a)}^2.

ところで
(a^2 + 3b^2)(b^2 + 3c^2)(c^2 + 3a^2) と (a^2 + ab + b^2)(b^2 + bc + c^2)(c^2 + ca + a^2)
の大小は定まりそうにないですが、どうですか?
0753132人目の素数さん
垢版 |
2018/10/07(日) 17:27:07.24ID:0iw9KjaT
>>748
神掛かってる!
大量投下したやつを今ごろ確認しているところでござるが、関連する昨夏の不等式を再掲。
(自分のmemoから抜き出したので、未紹介のものもあるかもしれない。)

a、b、c∈R、k≧0、4≧λ≧0 に対して、
(1) (aa+k)(bb+k)(cc+k) ≧ (3kk/4)*(a+b+c)^2
(2) (aa+k)(bb+k)(cc+k) ≧ {(4k/3)^(3/2)}*(a-b)(b-c)(c-a)
(3) (aa+k)(bb+k)(cc+k) ≧ (kk/4)*{λ(aa+bb+cc) + (9-λ)(ab+bc+ca)}
(4) {aa+ (k+1)/3}{bb+ (k+1)/3}{cc+ (k+1)/3} ≧ {(k+4)/3}^2*{ab+bc+ca+ (k-5)/3}

a、b、c∈R、k≧1 に対して、
(5) (aa+k)(bb+k)(cc+k) ≧ (k+1)^2*(ab+bc+ca+k-2) + (abc-1)^2

a、b、c∈R、k≧2 に対して、
(6) (aa+k)(bb+k)(cc+k) ≧ (k+1)(ab+bc+ca+k-2)^2

a、b、c∈R、k≧(√2)-1 に対して、
(7) (aa+k)(bb+k)(cc+k) ≧ (k+1)^2*{(a+b+c)^2/3 + k-2}
0755132人目の素数さん
垢版 |
2018/10/07(日) 18:16:09.56ID:0iw9KjaT
>>748
> a,b,c ≧ 0, a+b+c ≦ √(8k) のとき
>  kk{(a+b+c)^2 +k} ≧ (aa+k)(bb+k)(cc+k) ≧ (3kk/4)(a+b+c)^2,

左側の等号成立条件は a=b=c=k=0 以外にありますか?
0757132人目の素数さん
垢版 |
2018/10/07(日) 18:42:14.42ID:0iw9KjaT
連投すまぬ。
a,b,cのうちの2つが0なら成り立ちますね。他にないかな?
0758132人目の素数さん
垢版 |
2018/10/07(日) 19:18:13.82ID:0iw9KjaT
>>755
a,b,cのうちの少なくとも2つが0、
a,b,cのうちの一つが0で、2つが√(2k)のとき

これだけかな?
0759132人目の素数さん
垢版 |
2018/10/08(月) 02:39:14.27ID:OXto1iGE
>>738 (1)
x, y >0 として証明。
lhs - rhs = {xy(x-y)^2 + (xy-1)^2}/{(x+1)^2 (y+1)^2 (xy+1)} ≧0.

一般化できるかな?つまり、
x,y,z>0 のときに、1/(x+1)^2 + 1/(y+1)^2 + 1/(z+1)^2 ≧ 1/(xyz+1) は成り立つ?
0760132人目の素数さん
垢版 |
2018/10/08(月) 03:25:37.55ID:OXto1iGE
4文字なら、a,b,c,d>0に対して、

1/(1+a)^2 + 1/(1+b)^2 + 1/(1+c)^2 + 1/(1+d)^2
≧ 1/(1+ab) + 1/(1+cd)
> 1/(1+abcd).
0761132人目の素数さん
垢版 |
2018/10/08(月) 03:40:58.25ID:moWJj/Va
〔補題〕
(1) 4(2-√3) > (√6 -√2),
(2) 12(2-√3) > 4(2-√3) + 2(√6 -√2) > 3(√6 -√2),
(3) (√2 +√3) > 2(√6 -√2) + 4(2-√3),
(4) 22/7 > 2(√6 -√2) + 4(2-√3),
(5) 6 + (√6 -√2) > (√5)(√2 +√3),
0762132人目の素数さん
垢版 |
2018/10/08(月) 04:06:45.56ID:moWJj/Va
>>761
(1)
 √3 -1 ≒ 0.7320508   1/√2 ≒ 0.70710678
 (左辺) - (右辺) = 2(√3 -1)(√3 -1 -1/√2) > 0,
(2)
 (1) から直ちに出る。
(3)
 (左辺) - (右辺) = (1/4)(√2 -1)^2・(√3 -1)^4・(√3 -√2) > 0,
(4)
 (左辺) - (右辺) = (1/14)(√2 -1)^3・(√3 -1)^4・(3√6 -7) > 0,
(5)
 さてどうするか…

なお、Snellius-Huygens から、2(√6 -√2) + 4(2-√3) > π が分かる。
0764132人目の素数さん
垢版 |
2018/10/08(月) 17:08:48.13ID:moWJj/Va
>>759
s = x+y+z, t = xy+yz+zx, u = xyz とおく。

lhs - rhs = {3+4s+2ss+2(st-3u)+(tt-2su)}/(u+t+s+1)^2 - 1/(u+1)
 = {2+2s+(ss-2t)-5u+2(ss-t)u+2(st-9u)u+11uu+(tt-2su)u}/{(u+t+s+1)^2・(u+1)},
 ≧0.        (← x,y,z≧0)

* 2 -5u +11uu = 63/44 + 11(5/22 -u)^2 ≧ 63/44,
0766132人目の素数さん
垢版 |
2018/10/09(火) 04:15:31.03ID:CC0qY+GR
n変数にして証明できますかね?
a_k >0 (k=1,2,…n) に対して、Σ1/(1+a_k)^2 ≧ 1/(1+Πa_k).
0767132人目の素数さん
垢版 |
2018/10/09(火) 05:55:14.05ID:CC0qY+GR
x>0に対して、9x^{10} + 2 ≧ 9x^8 + 2x^9 をAM-GMで示せ。
(蛇足だが、この不等式は任意の実数で成り立つ)
0768132人目の素数さん
垢版 |
2018/10/09(火) 06:15:35.95ID:jtiWu+AA
>>766
nについての帰納法でやってみた。

n=2 は >>759 より成立。

n≧3 のとき
(1) x_j ≧ 1 があるとき、帰納法の仮定により
 Σ[k=1,n] 1/(1+a_k)^2 > Σ[k≠j] 1/(1+a_k)^2
 ≧ 1/(1+Π[k≠j] a_k)
 ≧ 1/(1+Π[k=1,n] a_k),

(2) x_1〜x_n がすべて1以下のとき、右辺は増加する。
・n=3 の場合がチョト面倒。
 (右辺) = 1/(xyz+1) - 1/(xy+1)
 = xy(1-z)/{(xyz+1)(xy+1)}
 ≦ xy(1-z)/{xy(z+1)}       (← xy(1-z)≧0)
 = (1-z)/(z+1),
 (左辺) - (右辺) ≧ 1/(xy+1) + 1/(z+1)^2 -1/(xyz+1)  (←帰納法の仮定)
 ≧ 1/(z+1)^2 - (1-z)/(z+1)
 = {z/(z+1)}^2
 ≧ 0,

・n≧4 ならば
 Σ[k=1,n] 1/(1+a_k)^2 ≧ Σ[k=1,n] 1/4   (← a_k≦1)
 = n/4
 ≧ 1
 > 1/(1+Π[k=1,n] a_k),
0769132人目の素数さん
垢版 |
2018/10/09(火) 06:56:44.02ID:jtiWu+AA
>>767

AM-GM より
9x^10 -10x^9 + 1
 = (x-1) (9x^9 -x^8 -x^7 -x^6 -x^5 -x^4 -x^3 -x^2 -x -1)
 = (x-1)^2 (9x^8 +8x^7 +7x^6 +6x^5 +5x^4 +4x^3 +3x^2 +2x +1)
 = (x-1)^2 {5x^8 + (x+1)^2 (4x^6 +3x^4 +2x^2 +1)}
 ≧ 0,

AM-GMより
4x^10 -5x^8 + 1
 = (x^2 -1) (4x^8 -x^6 -x^4 -x^2 -1)
 = (x^2 -1)^2 (4x^6 +3x^4 +2x^2 +1)
≧ 0,

(与式) = {(上) + (下)・9}/5
0771132人目の素数さん
垢版 |
2018/10/09(火) 14:38:57.52ID:jtiWu+AA
>>767

(左辺) - (右辺) = 2(4x^10 -5x^8 +1) + {(x-1)x^4}^2

 ≧ 2(4x^10 -5x^8 +1)

 = 2{(X^5 + X^5 + X^5 + X^5 + 1) - 5 X^4}   (← X=x^2≧0)

 ≧ 0,

最後のところで AM-GM を使いました。
0772132人目の素数さん
垢版 |
2018/10/09(火) 16:36:47.68ID:CC0qY+GR
>>767
AM-GMより、
x^{10} + x^9 ≧ 2x^9,
8x^{10} + 2 ≧ 10x^8. (x^8 が8個と 1が2個)

辺々加えて、
9x^{10} + 2 + x^8 ≧ 10x^8 + 2x^9.

( ゚∀゚) ウヒョッ!
0773132人目の素数さん
垢版 |
2018/10/09(火) 18:11:28.36ID:CC0qY+GR
>>738(1) >>759 >>764 >>768
> x,y,z>0 のとき、1/(x+1)^2 + 1/(y+1)^2 + 1/(z+1)^2 ≧ 1/(xyz+1).

右辺を見て次の不等式を思い出したが、繋がるかな?

x,y,z>0 のとき、1/{x(1+y)} + 1/{y(1+z)} + 1/{z(1+x)} ≧ 3/(1+xyz).
0774132人目の素数さん
垢版 |
2018/10/09(火) 18:46:04.69ID:jtiWu+AA
>>767

p_0 = 9,
p_1(x) = 6.19544630295     + (x-0.03352960039751934)^2 p_0 > 0,
p_2(x) = 3.8953637526451576 + (x-0.003121543171869486)^2 p_1(x) > 0,
p_3(x) = 2.0721715662084579 + (x+0.08618793580133872)^2 p_2(x) > 0,
p_4(x) = x^8 + 2(x+1)^2 (4x^6 +3x^4 +2x^2 +1),
    = 0.5197441948878409 + (x+0.8393520966569508138)^2 p_3(x) > 0,
p_5(x) = 9x^10 -2x^9 -9x^8 +2 = (x-1)^2 p_4(x) > 0,

( ゚∀゚) ウヒョッ!
0776132人目の素数さん
垢版 |
2018/10/09(火) 20:03:46.82ID:CC0qY+GR
>>768
> n≧3 のとき
> (1) x_j ≧ 1 があるとき、帰納法の仮定により
>  Σ[k=1,n] 1/(1+a_k)^2 > Σ[k≠j] 1/(1+a_k)^2

不等号が逆向きになりませんか?

  Σ[k=1,n] 1/(1+a_k)^2 < Σ[k≠j] 1/(1+a_k)^2
0778132人目の素数さん
垢版 |
2018/10/09(火) 21:22:11.48ID:CC0qY+GR
>>737
(問題再掲)
> a, b, c >0 に対して、
> a/{b(b+c)^2} + b/{c(c+a)^2} + c/{a(a+b)^2} ≧ 9/{4(ab+bc+ca)}

(証明)
(ab+bc+ca)*[a/{b(b+c)^2} + b/{c(c+a)^2} + c/{a(a+b)^2}]
≧ [ √(ab*a/{b(b+c)^2}) + √(bc*b/{c(c+a)^2}) + √(ca*c/{a(a+b)^2}) ]^2
= [ a/(b+c) + b/(c+a) + c/(a+b) ]^2
≧ (3/2)^2.

  ∧_∧
  ( ;´∀`) < シコシコ、ネビットの順に使うナリ。
  人 Y /
 ( ヽ し
 (_)_)
0779132人目の素数さん
垢版 |
2018/10/09(火) 23:28:00.13ID:jtiWu+AA
>>759 >>766 >>768

n≧3 のとき
p = Π[k=1,n-1] a_k, z = a_n とおく。

(右辺) = 1/(p・z+1) - 1/(p+1)
 = p(1-z)/{(p・z+1)(p+1)}
 = Max{ p(1-z)/{(p・z+1)(p+1)}, 0}
 ≦ Max{ (1-z)/(z+1), 0}
 ≦ 1/(z+1)^2,
∴ (左辺) - (右辺) ≧ 0,

>>775
 p_k(x) は 2k次の多項式。
 p_5(x) = (左辺) - (右辺) = 9x^10 -2x^9 -9x^8 +2,
 p_k(x) の最小値を b_k とし、そのときのxを a_k とする。
 p_{k-1}(x) = {p_k(x) - b_k}/(x-a_k)^2,
0780132人目の素数さん
垢版 |
2018/10/10(水) 05:28:35.94ID:hapOoDe1
>>746
するってぇと、こういうことかい?

k = (1/n)*(n-1)^{(n-1)/n} とおくとき、a,b,c>0 に対して、
a^{n+1}/(a^n + b^n) + b^{n+1}/(b^n + c^n) + c^{n+1}/(c^n + a^n) ≧ (1-k)(a+b+c).
0781132人目の素数さん
垢版 |
2018/10/10(水) 10:06:50.14ID:hapOoDe1
>>710
一般の自然数nの場合に右辺はどうなるのでせうか? 次式は成り立ちますか?

a,b,c>0に対して、
(a-b)(a-c)a^n + (b-c)(b-a)b^n + (c-a)(c-b)c^n ≧ (n+1){(a-b)(b-c)(c-a)}^2.
0783132人目の素数さん
垢版 |
2018/10/10(水) 19:47:02.29ID:hapOoDe1
a,b,c>0とし、Δ= (a-b)(b-c)(c-a)とおく。昨夏にやった不等式について。

(1) (27/8)*(a^2+b^2)(b^2+c^2)(c^2+a^2) ≧ (a^2+ab+b^2)(b^2+bc+c^2)(c^2+ca+a^2) ≧ (a^2+b^2)(b^2+c^2)(c^2+a^2) ≧ Δ^2
(2) k*Δ^2 ≧ (a^2-ab+b^2)(b^2-bc+c^2)(c^2-ca+a^2) ≧ Δ^2
(3) m*Δ^2 ≧ (a-b)(a-c)a^4 + (b-c)(b-a)b^4 + (c-a)(c-b)c^4 ≧ 5Δ^2

(疑問1) k、mの値を知りたい。
(疑問2) (1)もΔ^2の定数倍で挟みたい。
0784132人目の素数さん
垢版 |
2018/10/11(木) 03:47:19.96ID:XBFA4KXK
>>760
 1/(1+ab) + 1/(1+cd) > 1/(1+ab/2)^2 + 1/(1+cd/2)^2 > 1/(1+abcd/4),
 >>738(1) >>759

>>773 (下)
 1/{x(1+y)} + 1/{y(1+z)} + 1/{z(1+x)} ≧ 3/{G(1+G)} ≧ 3/(1+xyz),
 G = (xyz)^(1/3),

 バルカンMO-2006
 [8] 安藤哲哉 (2014) 例題3.1.7(4)
 [9] 佐藤淳郎[訳] (2013) 問題3.93
 Inequalitybot [77]

>>783
 例えば a=b≠c ⇒ =0
0785132人目の素数さん
垢版 |
2018/10/11(木) 17:32:09.43ID:XBFA4KXK
・n=2
 1/(1+a)^2 + 1/(1+b)^2 > 1/(1+ab),   >>759(上)

・n=3
 1/(1+a)^2 + 1/(1+b)^2 + 1/(1+c)^2 > 1/(1+abc/2),   >>759(下) >>773(上)

・n=4
 1/(1+a)^2 + 1/(1+b)^2 + 1/(1+c)^2 + 1/(1+d)^2 > 1/(1 + abcd/4),   >>760 >>784

・nについての帰納法で >>784
 Σ1/(1+a_k)^2 ≧ 1/{1 + 4Π(a_k /2)},
0786132人目の素数さん
垢版 |
2018/10/12(金) 02:23:37.06ID:UbVFOF8C
>>785 念のため…

〔補題〕
 n≧2, a_k≧0 (k=1〜n) のとき
 Σ[k=1, n] 1/(1+a_k)^2 ≧ 1/{ 1 + (Π[k=1, n] a_k) /2^(n-2) },

(略証)
nについての帰納法による。
・n=2 のとき
 >>759 (上)
・n≧3 のとき
(左辺) = Σ[k=1, n] 1/(1+a_k)^2
 ≧ 1/{1 + (Π[k=1, n-1] a_k) /2^(n-3) } + 1/(1+a_n)^2   (←帰納法の仮定)
 ≧ 1/{1 + (Π[k=1, n-1] a_k) /2^(n-2) }^2 + 1/(1+a_n)^2
 ≧ 1/{1 + (Π[k=1, n] a_k) /2^(n-2) }           ( >>759 上)
 = (右辺).
0787132人目の素数さん
垢版 |
2018/10/12(金) 04:07:57.14ID:f607XAs3
(a^2-ab+b^2)(b^2-bc+c^2)(c^2-ca+a^2) を同じ式で挟むとしたら、こんなもん?

(a^2+b^2)(b^2+c^2)(c^2+a^2)
≧ (a^2-ab+b^2)(b^2-bc+c^2)(c^2-ca+a^2)
≧ (8/27)*(a^2+b^2)(b^2+c^2)(c^2+a^2)


>>784
成程、a=bのときを考えれば凾ナ挟めないのは明らかですね。

>>786
ちょうど悩んでいたところで助かりますた。
直近でやった不等式が使えるとは、偶然以上の何かを感じる…
0788132人目の素数さん
垢版 |
2018/10/12(金) 05:35:02.53ID:UbVFOF8C
>>753

(1)  (aa+k)(bb+k)(cc+k) ≧ (3kk/4)ss + (u-K)^2,   ただし K = (k/2)^(3/2),
   [前スレ.456] [前スレ.469] >>4 [3]

(略証)
 (aa+k)(bb+k)(cc+k) = uu + k(tt-2su) + kk(ss-2t) + k^3
 = {uu + 2(k/2)^3} + (2k/3)(tt-3su) + (k/3)(t-3k/2)^2 + kk(ss-t) + (3kk/4)ss
 ≧ (u-K)^2 + (k/3)(t-3k/2)^2 + (2k/3)(tt-3su) + (kk/4){ss-4t+3u^(2/3)} + (3kk/4)ss
 = (u-K)^2 + (k/3)(t-3k/2)^2 + (2k/3)(tt-3su) + (kk/4s)F1(a,b,c) + (3kk/4)ss,

※ uu + 2(k/2)^3 = uu + 2KK = (u-K)^2 + K(u+u+K) ≧ (u-K)^2 + (3kk/4)u^(2/3),
  ただし K = (k/2)^(3/2),
  ss -4t +3u^(2/3) ≧ ss -4t +9u/s = F1(a,b,c)/s,

(3) はλ=4 が最良で、
  (aa+k)(bb+k)(cc+k) ≧ (kk/4)(4ss-3t) + (u-K)^2,   但し K = (k/2)^(3/2),
  [前スレ.469] >>4 [4] >>36
0789132人目の素数さん
垢版 |
2018/10/12(金) 06:10:22.02ID:f607XAs3
去年、アイゼンシュタイン整数を使って、a,b,c>0に対して、

(1) (a^2+ab+b^2)(b^2+bc+c^2)(c^2+ca+a^2) ≧ (3√3/8)*|(a^2-bc)(b^2-ca)(c^2-ab)|,
(2) (a^2+ab+b^2)(b^2+bc+c^2)(c^2+ca+a^2) ≧ (3√3/8)*(ab+bc+ca)^3

が出て、でも(2)は次より弱いから無視。

(3) (a^2+ab+b^2)(b^2+bc+c^2)(c^2+ca+a^2) ≧ (ab+bc+ca)^3

もっと細かく書くと、

(a^2+ab+b^2)(b^2+bc+c^2)(c^2+ca+a^2)
≧ (27/64)*(a+b)^2 (b+c)^2 (c+a)^2
≧ (1/3)*(a+b+c)^2 (ab+bc+ca)^2
≧ (ab+bc+ca)^3.

------------------------------------------------
(疑問1) 同様にやったら、次が成り立つと思うんですが、計算合ってます蟹?

(1)’ (a^2-ab+b^2)(b^2-bc+c^2)(c^2-ca+a^2) ≧ (3√3/8)*|(a^2-bc)(b^2-ca)(c^2-ab)|,
(2)’ (a^2-ab+b^2)(b^2-bc+c^2)(c^2-ca+a^2) ≧ (3√3/8)*|ab-bc+ca|^3

------------------------------------------------
(疑問2) (2)より強い(3)があったように、(2)’より強い次式って成り立ちますか?
2乗の差をとって計算していたのですが、挫折しますた。

(3)’(a^2-ab+b^2)(b^2-bc+c^2)(c^2-ca+a^2) ≧|ab-bc+ca|^3
0791132人目の素数さん
垢版 |
2018/10/12(金) 09:29:09.68ID:f607XAs3
------------------------------------------------
(疑問3) a,b,c>0 に対して、
4(a^2 + b^2 + c^2)^3 ≧ (a^2-ab+b^2)(b^2-bc+c^2)(c^2-ca+a^2)

が成り立つけど、左辺の係数の4をもっと小さくできないだろうか?

(左-右 = 12t(F_0)^2 + 12t^2 F_0 + 4t^3 + (2F_1 - st + 9u)^2 ≧0)

------------------------------------------------
(疑問4) 以前やった2つの不等式
a,b,c>0 に対して、(a^2+ab+b^2)(b^2+bc+c^2)(c^2+ca+a^2) ≧ {(a-b)(b-c)(c-a)}^2,
a,b,c∈Rに対して、(a^2+b^2+c^2)^3 ≧ 2{(a-b)(b-c)(c-a)}^2

の左辺について、a,b,c>0 に対して何か不等式は作れないだろうか? 👀
Rock54: Caution(BBR-MD5:1341adc37120578f18dba9451e6c8c3b)
0793132人目の素数さん
垢版 |
2018/10/12(金) 21:08:11.69ID:f607XAs3
(1) a,b,c∈R に対して、8(a^2+b^2)(b^2+c^2)(c^2+a^2) ≧ (a+b)^2 (b+c)^2 (c+a)^2.
(2) a,b,c∈R に対して、2(a^2+b^2)(b^2+c^2)(c^2+a^2) ≧ (a-b)^2 (b-c)^2 (c-a)^2.
(3) a,b,c>0 に対して、(a^2+b^2)(b^2+c^2)(c^2+a^2) ≧ (a-b)^2 (b-c)^2 (c-a)^2.

(2),(3) に比べて (1)の左辺の係数8が大きいですが、これが限界?
(1)の条件を a,b,c>0 に変えたら、係数は小さくできるかな?
最良値かどうかを判断する考え方がイマイチ分かりませぬ… ('A`)
0794132人目の素数さん
垢版 |
2018/10/13(土) 03:26:19.86ID:OSa6VDkO
>>789

(3) (aa+ab+bb)(bb+bc+cc)(cc+ca+aa) ≧ (1/3)sstt ≧ |t|^3,

(略証)
 ss±3t = {(a±b)^2 + (b±c)^2 + (c±a)^2}/2 ≧ 0,  (複号同順)
∴ |t| ≦ ss/3,

(疑問1)
(1)' … 1
 (aa-ab+bb)(bb-bc+cc)(cc-ca+aa) - (aa-bc)(bb-ca)(cc-ab)
= {(a-b)cc}^2 + {(b-c)aa}^2 + {(c-a)bb}^2 + (abc)^2
 ≧ 0,
 (aa-ab+bb)(bb-bc+cc)(cc-ca+aa) + (aa-bc)(bb-ca)(cc-ab)
 = {(a-b)ab}^2 + {(b-c)bc}^2 + {(c-a)ca}^2 + (abc)^2
 ≧ 0,

(2)' … 1/27
(疑問2) … 1/27
(3)’(aa-ab+bb)(bb-bc+cc)(cc-ca+aa) ≧ (1/81)sstt ≧ (1/27)|t|^3,
  (3) と同様に出ます。(*) 右辺はtのままです。
0795132人目の素数さん
垢版 |
2018/10/13(土) 03:29:35.03ID:OSa6VDkO
>>791 >>792

(疑問3) … 3/8
 (aa-ab+bb)(bb-bc+cc)(cc-ca+aa) = (ss-3t)(tt-3su) + stu -8uu,

(左辺) - (右辺)
 = (3/8)(aa+bb+cc)^3 - (aa-ab+bb)(bb-bc+cc)(cc-ca+aa)
 = (3/8)(ss-2t)^3 - (ss-3t)(tt-3su) -stu +8uu
 = (1/32)(3s^3 -10st +16u)^2 + (3/32){s(ss-2t)}^2  (←uで平方完成)
 ≧ 0,
 等号成立は (a, 0, -a) etc.
0796132人目の素数さん
垢版 |
2018/10/13(土) 03:48:21.26ID:OSa6VDkO
>>794
 絶対値は間違いです...orz

(3) (aa+ab+bb)(bb+bc+cc)(cc+ca+aa) ≧ (1/3)sstt,

(3)’(aa-ab+bb)(bb-bc+cc)(cc-ca+aa) ≧ (1/81)sstt,
0797132人目の素数さん
垢版 |
2018/10/13(土) 07:40:05.27ID:OSa6VDkO
>>794 >>795 >>796

aa+ab+bb = (3/4)(a+b)^2 + (1/4)(a-b)^2,
aa-ab+bb = (1/4)(a+b)^2 + (3/4)(a-b)^2,
(a+b)(b+c)(c+a) = st-u ≧ 8st/9,

(3)
 (aa+bb+cc)^3 ≧ (27/8)(aa+bb)(bb+cc)(cc+aa)     AM-GM
 ≧ (aa+ab+bb)(bb+bc+cc)(cc+ca+aa)
 ≧ (1/3){9(a+b)(b+c)(c+a)/8}^2
 ≧ (1/3)sstt,

(3')
 (3/8)(aa+bb+cc)^3 ≧ (aa-ab+bb)(bb-bc+cc)(cc-ca+aa)
 ≧ {(a+b)(b+c)(c+a)/8}^2
 ≧ (1/81)sstt,

>>794 (1)' から >>611 (6)
0798132人目の素数さん
垢版 |
2018/10/13(土) 07:46:23.21ID:OSa6VDkO
>>791
(疑問4)
 (上) >>752 (中)
 (下) >>609 (3)

>>793

(1) これが限界。 a=b=c で等号が成立するなら、a,b,c>0 に変えても同じぢゃね?

(2) (1-i)(a+ib)(b+ic)(c+ia) = -(a-b)(b-c)(c-a) + i{(a+b)(b+c)(c+a) - 4abc},
   >>609 (4) >>615
0800132人目の素数さん
垢版 |
2018/10/14(日) 05:52:44.82ID:Rg/i5zok
>>741
> |s| (ss-3t) ≦ (ss-2t)^(3/2)      (← GM-AM)

どのように相加相乗を使っているのですか?
0801132人目の素数さん
垢版 |
2018/10/14(日) 06:14:49.66ID:Rg/i5zok
742-743を見て、探してみたが、意外と少なかった… ('A`)ヴォエァ!

------------------------------------------
不等式スレ内を検索して

a^3+b^3+c^3-3abc : >>29>>738>>742-743
a^3+b^3+c^3+3abc : 第5章>>269、第2章>>372

------------------------------------------
My Collections から (出典不明)

(1) a,b,c∈Rに対して、(a^2+b^2+c^2)^3 ≧ (a^3+b^3+c^3-3abc)^2 + (ab+bc+ca)^3
(2) a,b,c≧に対して、a^3+b^3+c^3-3abc ≧ (1/4)*(a+b-2c)^3
0802132人目の素数さん
垢版 |
2018/10/14(日) 19:32:20.99ID:0CPQSloM
>>28 (2), >>29 (1), [前スレ.262], [初代スレ.836-869]
「楠瀬の不等式」
出典: 数学セミナー、出題:1992年4月、解説:1992年7月

a,b,c ∈ R に対して
 aa+bb+cc ≧ |ab| + |bc| + |ca| ≧ |ab+bc+ca|,
>>742 >>743 から
 | a^3+b^3+c^3 - 3abc |^2 ≦ (aa+bb+cc)^3 - |ab+bc+ca|^3.

>>800
 {ss, ss-3t, ss-3t} はいずれも非負。
 AM = ss-2t,  GM = {s(ss-3t)}^(2/3).
0803132人目の素数さん
垢版 |
2018/10/14(日) 19:47:58.72ID:Q7SdFY73
(x_1+…+x_n)/n=xとするとき、
(Σ(x_k-x)^3)^2 と (Σ(x_k-x)^2)^3 の大小について何か言えますか?
Σはk=1からnまでの和です。
0804132人目の素数さん
垢版 |
2018/10/15(月) 01:00:34.63ID:TBaDGY4B
>>803

(x_1 + x_2 + … + x_n) /n = A とおくとき、

( Σ[j=1,n] (x_j - A)^3 )^2 / ( Σ[k=1,n] (x_k - A)^2 )^3 ≦ (n-2)^2 /n(n-1) < 1,

等号成立は {a,…,a, b} など。
0805132人目の素数さん
垢版 |
2018/10/15(月) 08:43:04.04ID:TBaDGY4B
>>801 (1) は >>609 (2), >>742 と同じでつね。
>>802 (中) の方がチョト強い。

>>803
n=3 のとき
{(x-A)^2 + (y-A)^2 + (z-A)^2}^3 - 6{(x-A)^3 + (y-A)^3 + (z-A)^3}
= 2{(x-y)(y-z)(z-x)}^2 + (x+y+z-3A){……}

(略証)
(x-A)^3 + (y-A)^3 + (z-A)^3 = 3(x-A)(y-A)(z-A) + (x+y+z-3A){……} = 3(x-A)(y-A)(z-A),
より
{(x-A)^2 + (y-A)^2 + (z-A)^2}^3 - 6{(x-A)^3 + (y-A)^3 + (z-A)^3}
= 2{(x-y)(y-z)(z-x)}^2 + (x+y+z-3A){ …… }
≧ 0,
0806132人目の素数さん
垢版 |
2018/10/15(月) 09:33:15.98ID:r7ZX+DeY
>>805
むむむ…


ところで、ちょっと作ったんだけど、係数はこれが最善かな?
a,b,c∈R に対して、(a^2+b^2+c^2)^3 ≧ (27/16)*{(a-b)(b-c)(c-a)}^2.
0807132人目の素数さん
垢版 |
2018/10/15(月) 11:27:35.52ID:TBaDGY4B
>>806 … 2 ぢゃね?
 >>791 (疑問4・下) >>609 (3)


>>802
〔楠瀬の不等式〕
x,y,z ≧ 0 のとき
 x^3 + y^3 + z^3 -3xyz = (x+y+z)(xx+yy+zz-xy-yz-zx) ≧ A|(x-y)(y-z)(z-x)|,
ここに A = √(9+6√3) = √{(3/2)√3}(1+√3) = 4.403669475

(略証)
 (左辺) - (右辺) = (x^3 +y^3 +z^3 -3xyz) - A|(x-y)(y-z)(z-x)|
 = (1/2)(x+y+z){(x-y)^2 +(y-z)^2 +(z-x)^2} - A|(x-y)(y-z)(z-x)|,

 x,y,z の間隔を固定して一斉に動かしても、{ … } 内と右辺は変わらない。
 最小元が 0 のときに成り立てばよい。 以下 z=0 とする。

 (左辺) - (右辺) = x^3 -A xy|x-y| +y^3,

・0≦x≦y のとき
 x^3 + A xy(x-y) + y^3 = (x + y/αα)(x-αy)^2,
  α = {(1+√3) - √(2√3)}/2 = 0.43542054468234
  1/αα = (1+√3) + A/√3 = 5.27451056440629

・0≦y≦x のとき
 x^3 - Axy(x-y) + y^3 = (x + y/ββ)(x-βy)^2
  β = {(1+√3) + √(2√3)}/2 = 2.29663026289
  1/ββ = (1+√3) - A/√3 = 0.18959105073

 αβ = 1,
0809132人目の素数さん
垢版 |
2018/10/15(月) 12:36:41.62ID:r7ZX+DeY
a,b,c∈R に対して、(a^2+b^2+c^2)^3 ≧ k(a^2+b^2)(b^2+c^2)(c^2+a^2)

k = 27/8 が限界かと思うけど、2になりますかね?
0810132人目の素数さん
垢版 |
2018/10/15(月) 12:53:20.17ID:TBaDGY4B
>>806 >>807

(略証)
 bはaとcの中間にあるとしてよい。
 0 ≦ (a-b)(b-c) ≦ (1/4)(a-c)^2,
∴ aa+cc = (1/2)(a+c)^2 + (1/2)(a-c)^2 ≧ (1/2)(a-c)^2,
∴ (aa+cc)^3 ≧ (1/8)(a-c)^6 ≧ 2(a-c)^2 {(a-b)(b-c)}^2 = 2刧,
 >>609 (3), >>612 より再録
0811132人目の素数さん
垢版 |
2018/10/16(火) 00:51:28.34ID:5DYkLdwz
>>805

 (x-A) + (y-A) + (z-A) = 0,
x-A と y-A が同符号のとき
 (z-A)^2 = {(x-A) + (y-A)}^2 ≧ 4|(x-A)(y-A)|,
より
 (x-A)^2 + (y-A)^2 + (z-A)^2 = (1/2)(x+y-2A)^2 + (1/2)(x-y)^2 + (z-A)^2
 = (3/2)(z-A)^2 + (1/2)(x-y)^2
 ≧ (3/2)(z-A)^2,

{(x-A)^2 + (y-A)^2 + (z-A)^2}^3 ≧ (27/8)(z-A)^6 ≧ 6{3(x-A)(y-A)(z-A)}^2


>>809
 k = 27/8 ですね。 A,B,C≧0 より
(左辺) - (右辺) = (A+B+C)^3 -(27/8)(A+B)(B+C)(C+A)
 = S^3 - (27/8)(ST-U)
 = (S^3 -4ST +9U) + (5/8)(ST-9U)
 ≧ 0,
0812132人目の素数さん
垢版 |
2018/10/16(火) 03:14:15.05ID:K3TfA7ci
>>802
> a,b,c ∈ R に対して
>  aa+bb+cc ≧ |ab| + |bc| + |ca| ≧ |ab+bc+ca|,
> >>742 >>743 から
>  | a^3+b^3+c^3 - 3abc |^2 ≦ (aa+bb+cc)^3 - |ab+bc+ca|^3.

下の2行が分かりませぬ…。
0816132人目の素数さん
垢版 |
2018/10/17(水) 02:05:16.29ID:CNsWZSmr
>>815

SP.172
Prove that for any real numbers x,y,z:
(x+y+z)(y+z-x)(z+x-y)(x+y-z) ≦ (2yz)^2.

SP.173
 Prove that for any positive real numbers x,y,z:
 {xx√(yy+zz) + yy√(zz+xx) + zz√(xx+yy)} / (x^3+y^3+z^3) ≦ √2.

SP.174
Prove that for any positive real numbers a,b,c,x,y,z:
 (a^3+x^3+x^3+x^3)(y^3+b^3+y^3+y^3)(z^3+z^3+c^3+z^3) ≧ (ayz+bzx+cxy+xyz)^3.

SP.179 (改)
 If x ∈ [0,1) then:
1/2 < cos(x) ≦ 1 ≦ arcsin(x) + e^(-x).

UP.177
 If x,y,z,t >1 then:
 {log(x)/log(ztx)} {log(y)/log(txy)} {log(z)/log(xyz)} {log(t)/log(yzt)} < 1/16.
0817132人目の素数さん
垢版 |
2018/10/17(水) 03:59:20.74ID:CNsWZSmr
>>815
解答作りますた。

SP.172
 (x+y+z)(y+z-x) = (y+z)^2 -xx = 2yz - (xx-yy-zz),
 (z+x-y)(x+y-z) = xx - (y-z)^2 = 2yz + (xx-yy-zz),
辺々掛ける。
 (左辺) = (2yz)^2 - (xx-yy-zz)^2 ≦ (2yz)^2,

(*) x,y,z がΔの3辺の場合は、Δの面積が2辺の積の半分以下であることを表わす。

SP.173
 (左辺)^2 ≦ 3x^4・(yy+zz) + 3y^4・(zz+xx) + 3z^4・(xx+yy)
  = x^3・{3(xyy + xzz)} + y^3・{3(yzz + yxx)} + z^3・{3(zxx + zyy)}
  ≦ x^3・{(x^3+y^3+y^3) + (x^3+z^3+z^3)} + y^3・{(y^3+z^3+z^3) + (y^3+x^3+x^3)} + z^3・{(z^3+x^3+x^3) + (z^3+y^3+y^3)}
  = 2(x^3+y^3+z^3)^2,
  
SP.174
 コーシーそのもの。

SP.179
 arcsin(x) ≧ x, (0≦x<1)
 e^(-x) ≧ 1 - x,
辺々たす。

UP.177
 X=log(x), Y=log(y), Z=log(z), T=log(t) はすべて正だから AM-GM で
 Z+T+X ≧ 3(ZTX)^(1/3),
 T+X+Y ≧ 3(TXY)^(1/3),
 X+Y+Z ≧ 3(XYZ)^(1/3),
 Y+Z+T ≧ 3(YZT)^(1/3),
辺々掛けて
 (Z+T+X)(T+X+Y)(X+Y+Z)(Y+Z+T) ≧ 81 XYZT,

(左辺) = X/(Z+T+X)・Y/(T+X+Y)・Z/(X+Y+Z)・T/(Y+Z+T) ≦ 1/81,
0819132人目の素数さん
垢版 |
2018/10/17(水) 06:46:49.96ID:TTcOhpLB
ROMANIAN MATHEMATICAL MAGAZINE (RMM 11)、解答なし
http://www.ssmrmh.ro/wp-content/uploads/2018/09/11-RMM-WINTER-EDITION-2018-1.pdf
JP158、JP165、SP164、SP165など、いかがでござるか?

JP165の右辺を見て、毒電波を受信した。

a,b,c∈R に対して、
√{6(a^2+b^2+c^2)} ≧ √(a^2+b^2) + √(b^2+c^2) + √(c^2+a^2) ≧ 2√(a^2+b^2+c^2)


---------------------------------------------
ROMANIAN MATHEMATICAL MAGAZINE って、不等式専門雑誌なん?
最新2回分には解答が公開されないっぽい。
http://www.ssmrmh.ro/category/current-issue/
---------------------------------------------
0821132人目の素数さん
垢版 |
2018/10/17(水) 06:51:20.00ID:TTcOhpLB
解答のない号で、三角形がらみ(a,b,c,R,r,S,A,B,Cのみ)、シンプル、既出でないものを抽出。

ROMANIAN MATHEMATICAL MAGAZINE (RMM 12)
http://www.ssmrmh.ro/wp-content/uploads/2018/09/12-RMM-SPRING-EDITION-2019-2.pdf
JP173、JP179、UP171、UP175

ROMANIAN MATHEMATICAL MAGAZINE (RMM 11)
http://www.ssmrmh.ro/wp-content/uploads/2018/09/11-RMM-WINTER-EDITION-2018-1.pdf
JP157、UP155


ところで、JP171、JP174、JP153 などで説明なしに使われている h_a、m_a、l_a などは何を意味するのだろう? 垂線、中線、二等分線かな?
定義が分からないので、見た目がシンプルでも上のリストから外してしまったが…
0822132人目の素数さん
垢版 |
2018/10/17(水) 08:05:48.87ID:TTcOhpLB
>>815-817
SP.173の分母を払った式

(√2)(x^3+y^3+z^3) ≧ x^2√(y^2+z^2) + y^2√(z^2+x^2) +z^2√(x^2+y^2)

を見て、右辺にCSを使えば片付きそうな気がしたが、大きくなり過ぎた。

√{2(x^4+y^4+z^4)(x^2+y^2+z^2)}
≧ (√2)(x^3+y^3+z^3)
≧ x^2√(y^2+z^2) + y^2√(z^2+x^2) +z^2√(x^2+y^2)
0823132人目の素数さん
垢版 |
2018/10/17(水) 08:47:58.61ID:CNsWZSmr
>>819

JP.158
 Let a,b,c>0. Prove that:
  (1/a + 1/b + 1/c) + a/(bb+cc) + b/(cc+aa) + c/(aa+bb) ≧ 1/(a+b) + 1/(b+c) + 1/(c+a),

JP.165 (改)
 If a,b,c≧0 then:
 4(a+b+c) ≦ (2√2){√(aa+bb) + √(bb+cc) + √(cc+aa)},

SP.164 (改)
 If a,b,c > 0 then:
 (a+b)√(aa-ab+bb) + (b+c)√(bb-bc+cc) + (c+a)√(cc-ca+aa) ≧ 2(aa+bb+cc),

SP.165 (改)
 If a,b,c ≧0 then:
 (a+b)√(aa+bb) + (b+c)√(bb+cc) + (c+a)√(cc+aa) ≧ (1/√2){(aa+bb+cc) + (a+b+c)^2},
0824132人目の素数さん
垢版 |
2018/10/17(水) 09:14:42.86ID:CNsWZSmr
>>819 >>823

JP.165 (改)
 x+y ≦ √{2(xx+yy)} より
 4(a+b+c) ≦ (2√2){√(aa+bb) + √(bb+cc) + √(cc+aa)},

SP.164 (改)
コーシーより
 (x+y)√(xx-xy+yy) = √{(x+y)(x^3+y^3)} ≧ xx + yy,
(略証)
 (x+y)^2・(xx-xy+yy) - (xx+yy)^2 = (x+y)(x^3+y^3) - (xx+yy)^2 = xy(x-y)^2 ≧0,

SP.165 (改)
 √(xx+yy) ≧ (x+y)/√2, etc.
 (左辺) ≧ {(a+b)^2 + (b+c)^2 + (c+a)^2}/(√2) ≧ (√2)(aa+bb+cc+t),
 t = ab+bc+ca,

>>819
a,b,c∈R に対して、
√{6(aa+bb+cc)} ≧ √(aa+bb) + √(bb+cc) + √(cc+aa)
 ≧ √{4(aa+bb+cc) + 2t}
 ≧ (√2)s,
 s = a+b+c, t = ab+bc+ca,

(略証)
左側はコーシー
中は √(xx+yy)√(xx+zz) ≧ xx+xy, etc.
 ∵ (xx+yy)(xx+zz) - (xx+yz)^2 = {x(y-z)}^2 ≧ 0, (コーシー)
右側は aa+bb+cc ≧ t.
0825132人目の素数さん
垢版 |
2018/10/17(水) 10:31:10.77ID:CNsWZSmr
>>820

SP.140
 Let a,b,c be positive real numbers. Prove that:
 (b+c)/a + (c+a)/b + (a+b)/c ≧ 4(aa+bb+cc)/(ab+bc+ca) + 2(ab+bc+ca)/(aa+bb+cc) ≧ 6,
0826132人目の素数さん
垢版 |
2018/10/18(木) 03:45:39.48ID:Dw4OfxmO
>>820 >>825

SP.140 (改)
 (b+c)/a + (c+a)/b + (a+b)/c ≧ 4(aa+bb+cc)/(ab+bc+ca) + 2
 ≧ 3(aa+bb+cc)/(ab+bc+ca) + 3(ab+bc+ca)/(aa+bb+cc) ≧ 6,

(略証)
 s = a+b+c, t = ab+bc+ca, u = abc とおく。
(左辺) - 4(ss-2t)/t - 2
 = (st-3u)/u - 4(ss-2t)/t - 2
 = st/u - 4ss/t + 3
 = (s/ttu)(t^3 -4stu +9uu) + (3/tt)(tt-3su)
 ≧ 0,


>>819 >>823 >>824
SP.164 (改) より
 √(xx-xy+yy) > {(xx+yy)/(x+y), M_4} > M_3 > √{(xx+yy)/2} > (x+y)/2 > √(xy) > 2xy/(x+y),

ここに M_r = {(x^r+y^r)/2}^(1/r) はr乗平均, M_1 = (x+y)/2, M_2 = √{(xx+yy)/2},
0827132人目の素数さん
垢版 |
2018/10/18(木) 06:51:05.46ID:Dw4OfxmO
>>821

RMM 12 (Spring2019)

JP.173
 Prove that in any triangle ABC,
  1/a + 1/b + 1/c ≧ √{3/(2Rr)} ≧ (√3)/R.

JP.179
 In acute triangle ABC the following relationship hplds:
 3 ≦ sin(2A)/sin(2B) + sin(2B)/sin(2C) + sin(2C)/sin(2A) ≦ 3/{8cos(A)cos(B)cos(C)},

UP.171
 Find that in any acute-angled triangle ABC the following inequality holds:
 min{a/(b+c), b/(c+a), c/(a+b)} ≦ {cos(A) + cos(B) + cos(C)}/3 ≦ Max{a/(b+c), b/(c+a), c/(a+b)},

UP.175 (改)
 In acute triangle ABC the following relationship holds:
 (b+c)^2/(bb+cc-aa) + (c+a)^2/(cc+aa-bb) + (a+b)^2/(aa+bb-cc) ≧ 12,

等号成立は正△のとき、だろうな…
0828132人目の素数さん
垢版 |
2018/10/18(木) 07:52:45.35ID:Dw4OfxmO
>>819 >>823

RMM 11 (Winter2018)

JP.158 (訂正)
 Let a,b,c>0. Prove that:
  (1/a + 1/b + 1/c) + a/(bb+cc) + b/(cc+aa) + c/(aa+bb) ≧ 3/(a+b) + 3/(b+c) + 3/(c+a),

(略証) チェビシェフしたあと、
 (1/x + 1/y)/2 + (x+y)/{2(xx+yy)} - 3/(x+y)
 = (x+y)/(2xy) + (x+y)/{2(xx+yy)} - 3/(x+y)
 = (x+y)(xx+xy+yy)/{2xy(xx+yy)} - 3/(x+y)
 = (x-y)^2 (xx-xy+yy)/{2xy(xx+yy)(x+y)}
 ≧ 0,
0829132人目の素数さん
垢版 |
2018/10/18(木) 15:11:32.69ID:k/D5nzuI
>>822
√{2(x^4+y^4+z^4)(x^2+y^2+z^2)} にCSを使うと、使い方次第で
≧ (√2)(x^3+y^3+z^3) にも
≧ x^2√(y^2+z^2) + y^2√(z^2+x^2) +z^2√(x^2+y^2) にもなるんだな。
0830132人目の素数さん
垢版 |
2018/10/19(金) 11:50:11.00ID:UmCMoNsS
>>822 >>829
 コーシーとチェビシェフの合わせ技(?)

〔補題〕
 (a,b,c) と (p,q,r) が同順序のとき
 √(aa+bb+cc) √(pp+qq+rr) ≧ (ap+bq+cr) ≧ (a+b+c)(p+q+r)/3 ≧ (aq+ar+bp+bq+cp+cq)/2,

 (a,b,c) と (p,q,r) が逆順序のとき
 √(aa+bb+cc) √(pp+qq+rr) ≧ (aq+qr+bp+br+cp+cq)/2 ≧ (a+b+c)(p+q+r)/3 ≧ (ap+bq+cr),
0831132人目の素数さん
垢版 |
2018/10/23(火) 15:23:49.80ID:QCR0wRAh
z∈C が |z + 1/2| < 1/2 をみたすとき、
任意の n∈N に対して |1 + z + z^2 + … + z^n|^2 < 1.
0832132人目の素数さん
垢版 |
2018/10/23(火) 16:45:18.22ID:foOj88Cn
>>831

題意より、
|z| ≦ |z+1/2| + (1/2) < 1,
∴ |1-z|^2 = (1-z)(1-z~)
 = (3/2) - 2|z+1/2|^2 + 3|z|^2
 > 1 + 3|zz|
 > 1 + 2|zz| + |zz|^2
 = (1+|zz|)^2,
∴ |1-z| > 1 + |zz| > 1 + |z|^(n+1) ≧ |1 - z^(n+1)|.

東工大-2000 前期 Q.2
[第7章.114,116,160]
Inequalitybot [183]
0834132人目の素数さん
垢版 |
2018/10/24(水) 04:35:15.90ID:RiX0WTIF
>>831
この問題の結論の不等式って、|1 + z + z^2 + … + z^n| < 1 と書かずに、
あえて2乗にしているのは、何か意味があるのかな?
0835132人目の素数さん
垢版 |
2018/10/24(水) 22:16:00.49ID:LB37fX3V
>>627 (Nesbitt-Igarashi)

(略証)
各辺に ab+bc+ca を掛けると コーシー型になる:

{a(bb+bc+cc) + b(cc+ca+aa) + c(aa+ab+bb)} {a/(bb+bc+cc) + b/(cc+ca+aa) + c/(aa+ab+bb)}
 ≧ {a(b+c) + b(c+a) + c(a+b)} {a/(b+c) + b/(c+a) + c/(a+b)}
 ≧ (a+b+c)^2,

そこで ラグランジュの恒等式
 (ax + by + cz)(a/x + b/y + c/z) - (a+b+c)^2 = (ab/xy)(x-y)^2 + (bc/yz)(y-z)^2 + (ca/zx)(z-x)^2,
を使う。

・左辺は
 x = bb + bc + cc,
 y = cc + ca + aa,
 z = aa + ab + bb,
 ax + by + cz = (a+b+c)(ab+bc+ca),     >>621
 (左辺) - (a+b+c)^2 = {a(a+b+c)/(bb+bc+cc)}{b(a+b+c)/(cc+ca+aa)}(a-b)^2 + …

・中辺は
 x = b + c,
 y = c + a,
 z = a + b,
 ax + by + cz = 2(ab+bc+ca),
 (中辺) - (a+b+c)^2 = {a/(b+c)}{b/(c+a)}(a-b)^2 +{b/(c+a)}{c/(a+b)}(b-c)^2 + {c/(a+b)}{a/(b+c)}(c-a)^2,

ここで、
 (a+b+c)/(bb+bc+cc) > (b+c)/(bb+bc+cc) > 1/(b+c),
 (a+b+c)/(cc+ca+aa) > (c+a)/(cc+ca+aa) > 1/(c+a),
 (a+b+c)/(aa+ab+bb) > (a+b)/(aa+ab+bb) > 1/(a+b),
だから
 (左辺) ≧ (中辺).

* (x,y,z) はもっと改良できるかも…
0836132人目の素数さん
垢版 |
2018/10/25(木) 12:43:20.62ID:0sa6guuR
>>835 *

 x = (b^n - c^n)/(b-c),
 y = (c^n - a^n)/(c-a),
 z = (a^n - b^n)/(a-b),
とすると
 x-y = -(a-b) D_n /,
 y-z = -(b-c) D_n /,
 z-x = -(c-a) D_n /,
ここに
 D_n = det{ [1,1,1] [a,b,c] [a^n,b^n,c^n] }
  = (a-b)(b-c)(c-a) = D_2,   … Vandermonde の行列式
0837132人目の素数さん
垢版 |
2018/10/26(金) 11:32:25.02ID:QcEpehDd
>>836

3文字のとき
 D_n = det{ [1,1,1] [a,b,c] [a^n,b^n,c^n] }
   = (c-b)a^n + (a-c)b^n + (b-a)c^n,

特性多項式
 (λ-a)(λ-b)(λ-c) = λ^3 -s・λ^2 + tλ -u,
 ただし s = a+b+c,t = ab+bc+ca,u = abc,

漸化式
 D_n = s・D_{n-1} - t・D_{n-2} + u・D_{n-3},

D_n/ = Σ {すべての(n-2)次積}
  … (n-2)個の重複組み合わせに対応

D_0 / = 0,
D_1 / = 0,
D_2 / = 1,
D_3 / = a+b+c = s,
D_4 / = aa+ab+ac+bb+bc+cc = ss-t,
D_5 / = s^3 -2st +u,
D_6 / = s^4 -3sst +tt +2su,
D_7 / = s^5 -4s^3・t +3stt +3ssu -2tu,
0838132人目の素数さん
垢版 |
2018/10/26(金) 17:18:14.29ID:QcEpehDd
>>837

まづ
 x_1 = y_1 = z_1 = 1,
 x_2 = b+c,y_2 = c+a,z_2 = a+b,
 x_3 = bb+bc+cc,y_3 = cc+ca+aa,z_3 = aa+ab+bb,
 ……
 x_n = b^(n-1) + b^(n-2)c + …… + c^(n-1),
とおく。

ラグランジュの恒等式から
 (ax+by+cz)(a/x + b/y + c/z) - (a+b+c)^2
 = (a/x)(b/y)(x-y)^2 + (b/y)(c/z)(y-z)^2 + (c/z)(a/x)(z-x)^2
 = (D_n/)^2 {(a/x_n)(b/y_n)(a-b)^2 + (b/y_n)(c/z_n)(b-c)^2 + (c/z_n)(a/x_n)(c-a)^2},  >>835

そこで
 (D_n/)/x_n,(D_n/)/y_n,(D_n/)/z_n
がnについて単調増加であることを示そう。

F_n = x_n (D_{n+1}/) - x_{n+1} (D_n/)
= {(b-a)(ab)^2 + (c-b)(bc)^2 + (a-c)(ca)^2} /
= (D_{-n}/)u^n
= Σ {ab,bc,ca の (n-1)次積}
≧ 0,
∴ nについて単調増加。
 (D_{n+1}/) / x_{n+1} ≧ (D_n/) / x_n ≧ …… ≧ (D_2/) / x_2 = 1/(b+c),

これを Nesbitt-Igarashi 列とか呼ぼう。

 F_0 = 0,
 F_1 = 1,
 F_2 = t,
 F_3 = tt -su,
 F_4 = t^3 -2stu +uu,
漸化式
 F_n = t F_{n-1} - su F_{n-2} + uu F_{n-3},
0842132人目の素数さん
垢版 |
2018/10/27(土) 02:35:22.24ID:4A6u4AJ8
>>841
うーむ。
Nesbitt's inequality の英語のwikiを見てきたが、どこの国の人か分からんなあ。

ところで Nesbitt's inequality の一般化について、このスレでやったことあったっけ?
0843132人目の素数さん
垢版 |
2018/10/29(月) 23:18:40.58ID:YnG/8gLh
不等式ぢゃないが、次の等式を手計算で証明するのはキツそうでござるかな?

(6a^2 - 4ab + 4b^2)^3 + (3b^2 + 5ab - 5a^2)^3
= (6b^2 - 4ab + 4a^2)^3 + (3a^2 + 5ab - 5b^2)^3
0844132人目の素数さん
垢版 |
2018/10/30(火) 03:45:36.60ID:bXAGzjkG
>>843
(6aa-4ab+4bb)^3 - (6bb-4ab+4aa)^3 = (3aa+5ab-5bb)^3 - (3bb+5ab-5aa)^3,

(略証)
x^3 - y^3 = (x-y)(xx+xy+yy),
から
(maa-nab+nbb)^3 - (naa-nab+mbb)^3
= (m-n)(a-b)(a^3+b^3) {(mm+mn+nn)(aa+ab+bb) -3(m+n)n・ab}
= (a-b)(a^3+b^3) {(m^3 - n^3)(aa+ab+bb) - 3(m-n)(m+n)n・ab},

(m,n) = (6,4) (3,-5) のときは
 m^3 - n^3 = 152,
 (m-n)(m+n)n = 80,
となり、相等しい。
0845132人目の素数さん
垢版 |
2018/10/30(火) 04:35:23.11ID:XhFYWByL
>>844
おぉ有難い。上手にやりましたね。
それにしても、この等式を見つけ出したラマヌジャンは変態ジャン。
0846132人目の素数さん
垢版 |
2018/10/30(火) 06:59:54.63ID:XhFYWByL
>>844
> = (a-b)(a^3+b^3) {(m^3 - n^3)(aa+ab+bb) - 3(m-n)(m+n)n・ab},

ここは
= (a-b)(a^3+b^3) {(m^3 - n^3)(aa+ab+bb)}
じゃないですか?
0847132人目の素数さん
垢版 |
2018/11/02(金) 01:22:59.67ID:im1SI6w9
>>843

 6^3 + (-4)^3 + (-3)^3 + (-5)^3 = 0,
 4^3 + (-6)^3 + 5^3 + 3^3 = 0,
から推して
 (6aa+pab+4bb)^3 + (-4aa-pab-6bb)^3 + (-3aa+qab+5bb)^3 + (-5aa-qab+3bb)^3 = 0,
と予想する。(p,q は或る定数)
ab=0 のときは明らか。

6ab(aa-bb){2(5p-4q)(aa+bb) + (84+pp-4qq)ab} = 0,

5p -4q = 0, 84 +pp -4qq = 0,

p = ±4, q=±5  (複号同順)

(例)
a = ±1,b = ±2,p=±4,q=±5 (複号同順)のとき
 ±{30,-36,27,-3} = ±3{10,-12,9,-1}
0848132人目の素数さん
垢版 |
2018/11/02(金) 02:51:41.47ID:im1SI6w9
>>847

12^3 - 10^3 = 9^3 - 1^3 = 8^3 - (-6)^3 = 728,
のような珍例を「ナニワ数」と云う。…っちゅうのは冗談やけどな。

・系列解は他にもある。
 {7aa-16ab-3bb,14aa+4ab+6bb,-14aa+4ab-6bb,-7aa-16ab+3bb} (Dickson)
 (maa-pab-nbb)^3 + (-maa-pab+nbb)^3 = -6pab(maa-nbb)^2 -2ppp(ab)^3
 m → km,n' → -kn,p' → -p/kk とすれば 6pab(maa+nbb)^2 + 2(p/kk)^3 (ab)^3
 辺々たすと 2p{12mn - (1 - 1/k^6)pp}(ab)^3,
 12mn - (1 - 1/k^6)pp = 0 ならば成立。

 {aa-7ab+63bb,8aa-20ab-42bb,6aa+20ab-56bb,-9aa+7ab-7bb}

http://www.maroon.dti.ne.jp/fermat/dioph1.html

・Fermat cubic surface とか云うらしい。
http://www.math.harvard.edu/~elkies/4cubes.html
0849132人目の素数さん
垢版 |
2018/11/03(土) 05:17:44.99ID:/E6xXixt
m^3 - n^3 = m’^3 - n’^3 のとき、 ラマヌジャン系列

(maa+pab+nbb)^3 - (naa+pab+mbb)^3
= (m-n)(a^2-b^2){(mm+mn+nn)(a^4+aabb+b^4) + 3(m+n)p ab(a^2+b^2) + 3(pp+mn) aabb}
= (m^3 - n^3) (a^6 - b^6) + 3(m^2-n^2)p ab(a^4 - b^4) + 3(m-n)(pp+mn) aabb(a^2-b^2),

→ m^3 - n^3,(m^2-n^2)p,(m-n)(pp+mn) が等しいとき、相等しい。

(maa+qab-nbb)^3 - (naa+qab-mbb)^3
= (m-n) (a^2+b^2){(mm+mn+nn)(a^4-aabb+b^4) + 3(m+n)q b(a^2-b^2) + 3(qq-mn) aabb}
= (m^3 - n^3) (a^6 + b^6) + 3(m^2-n^2)q ab(a^4 - b^4) + 3(m-n)(qq-mn) aabb(a^2 + b^2),

→ m^3 - n^3,(m^2-n^2)q,(m-n)(qq-mn) が等しいとき、相等しい。
0852132人目の素数さん
垢版 |
2018/11/05(月) 04:57:42.95ID:mrtyuQkn
>>851
この出題者が出していた大量の不等式の問題は、もう削除されて見れないんだよな。
実に惜しいことをした。
0853132人目の素数さん
垢版 |
2018/11/08(木) 02:11:08.99ID:PxlSRNgU
bot.62
x,y,z∈[0,1] のとき、sqrt|x-y| + sqrt|y-z| + sqrt|z-x| の最大値

どぉやるんでせうか?
0854132人目の素数さん
垢版 |
2018/11/09(金) 03:38:15.13ID:UXVKU4RE
(3/4)*(1 + A/H)^2 ≧ (A/G)^3 + (G/H)^3

この手の不等式が胸やけ起こしそうなくらい沢山載ってる本ないかな?
0855132人目の素数さん
垢版 |
2018/11/09(金) 06:52:20.28ID:pvdoV3Z4
>>853 [62]

yはxとzの中間にあるとする。コーシーで
 (√|x-y| + √|y-z|)^2 ≦ (1+1) (|x-y|+|y-z|) = 2|x-z|,
(左辺) ≦ (1+√2)|z-x| ≦ 1+√2,
等号は(0,1/2,1) etc.

中国MO-2012 Round2-A.3
0856132人目の素数さん
垢版 |
2018/11/11(日) 22:10:14.90ID:+dEG2DHU
三角形の辺長 a,b,c に対して、
Σ[cyc] (a+b-c)(b+c-a)/(c+a-b) ≧ 3(aa+bb+cc)/(a+b+c).
0857132人目の素数さん
垢版 |
2018/11/12(月) 08:15:22.35ID:TKDy5P8X
>>856

b+c-a = x, c+a-b = y, a+b-c = z,
とおく。(Ravi変換)
2a = y+z, 2b = z+x, 2c = x+y, a+b+c = x+y+z,

(左辺) = xy/z + yz/x + zx/y = (xxyy+yyzz+zzxx)/xyz,

(右辺) = 3(aa+bb+cc)/(a+b+c)^2 = 6(xx+yy+zz+xy+yz+zx)/{4(x+y+z)},

4(x+y+z)(xxyy+yyzz+zzxx) - 6xyz(xx+yy+zz+xy+yz+zx)
= (3x+y+z)[x(y-z)]^2 + (x+3y+z)[y(z-x)]^2 + (x+y+3z)[z(x-y)]^2 ≧ 0,
かな。
0859132人目の素数さん
垢版 |
2018/11/12(月) 19:53:45.29ID:TKDy5P8X
>>854

(3/4)(1 + A/H)^2 - (A/G)^3 - (G/H)^3 -1 = (1/108){(a-b)(b-c)(c-a)/abc}^2 ≧ 0,

(略証)
s = a+b+c, t = ab+bc+ca, u = abc,  = (a-b)(b-c)(c-a),
とおくと
A = s/3, G = u^(1/3), H = 3u/t,
A/H = st/9u, A/G = s/{3u^(1/3)}, G/H = t/{3u^(2/3)},
ゆえ
(左辺) = (3/4)(1+st/9u)^2 - s^3/27u - t^3/27uu -1
 = (1/108uu){(st+9u)^2 -4s^3u -4t^3 -108uu}
 = (1/108uu)竸2,
0860132人目の素数さん
垢版 |
2018/11/13(火) 09:14:18.15ID:btM1CEFR
>>859
(3/4)*(1 + A/H)^2 ≧ (A/G)^3 + (G/H)^3 + 1

3変数の場合に上式を証明しているけど、これは一般の場合にも成り立つのかな?
>>854では、右辺に +1がないのには意味があるのかな?
0863132人目の素数さん
垢版 |
2018/11/13(火) 22:03:59.25ID:btM1CEFR
[2] かんどころ P.121定理6.7 は、証明ついてないようだけど、どうやればいいか分かりますか?
0864132人目の素数さん
垢版 |
2018/11/14(水) 00:54:00.60ID:uakH23jG
>>860
右辺に +1 が無いと緩くなります。

2変数の場合は
 (3/4)(1 + A/H)^2 - (A/G)^2 - (G/H)^2 -1 = (3a+b)(a+3b){(a-b)/8ab}^2 ≧ 0,

∵ (A/G)^2 = (G/H)^2 = A/H = (a+b)^2 /4ab,
0865132人目の素数さん
垢版 |
2018/11/14(水) 03:43:20.66ID:uakH23jG
余談ですが、n変数の (A-G)/(G-H) の下限は
 n=2  1.0
 n=3  0.90096030150908885
 n=4  0.7761577683742073233
 n=5  0.67617485
 n=6  0.59845640
 n=7  0.53716474
 n=8  0.48781223
 n=9  0.44727765
 n=10  0.41339822
ぐらいかな。

http://suseum.jp/gq/question/2646, 2948
0868132人目の素数さん
垢版 |
2018/11/14(水) 14:58:31.03ID:fnsD9k3Y
>>867
あまりにもショボすぎるので、改造してみた。

a,b,c,d,x,y,z∈R, a≧d≧0, b≧c≧0, x≧y≧0 に対して、
(ax+cy+z)(bx+dy+z)≧{(a+b)x+z}{(c+d)y+z}.

後ろのzも pz+qw, rz+sw にできぬか?

                l三`ー 、_;:;:;:;:;:;:j;:;:;:;:;:;:_;:;:;_;:-三三三三三l
               l三  r=ミ''‐--‐';二,_ ̄    ,三三三彡彡l_   この感じ・・・・
              lミ′   ̄    ー-'"    '=ミニ彡彡/‐、ヽ
                  l;l  ,_-‐ 、    __,,.. - 、       彡彡彳、.//  
_______∧,、_‖ `之ヽ、, i l´ _,ィ辷ァ-、、   彡彡'r ノ/_ ______
 ̄ ̄ ̄ ̄ ̄ ̄ ̄'`'` ̄ 1     ̄フ/l l::. ヽこ~ ̄     彡彳~´/  ̄ ̄ ̄ ̄ ̄ ̄
                 ヽ   ´ :l .l:::.         彡ィ-‐'′
                ゝ、  / :.  :r-、        彡′
              / ィ:ヘ  `ヽ:__,ィ='´        彡;ヽ、
          _,,..-‐'7 /:::::::ヽ   _: :_    ヽ      ィ´.}::ヽ ヽ、
      _,-‐'´    {  ヽ:::::::::ヘ `'ー===ー-- '   /ノ /::::::ヘ, ヽー、
0869132人目の素数さん
垢版 |
2018/11/14(水) 16:19:40.82ID:fnsD9k3Y
できた ( ゚∀゚) ウヒョッ

a,b,c,d,p,q,r,s,x,y,z.w∈R,
a≧d≧0, b≧c≧0, p≧s≧0, q≧r≧0, x≧y≧0, z≧w≧0 に対して、
(ax+cy+pz+rw)(bx+dy+qz+sw)≧{(a+b)x+(p+q)z}{(c+d)y+(r+s)w}.
0870132人目の素数さん
垢版 |
2018/11/14(水) 16:53:56.03ID:fnsD9k3Y
話を元に戻すと、>>867 を使ったAM-GMの証明 ([2] かんどころP.118)で、
1回目に>>867を使うところは分かる。

(a_1 + a_2 + (a_3+…+a_n))(a_1 + a_2 + (a_3+…+a_n))
≧(2a_1 + (a_3+…+a_n))(2a_2 + (a_3+…+a_n))

2回目に>>867を使うところ、どこが対応しているのか分からんのですが、どうなってるのですか?

(2a_1 + (a_3+…+a_n))(a_1 + a_2 + (a_3+…+a_n))
≧(3a_1 + (a_4+…+a_n))(2a_3 + (a_2+a_4+…+a_n))

以下続けて (k*a_1+ a_{k+1}+…+a_n) と (a_1 + a_2 + (a_3+…+a_n)) に>>867を使って
最終的に n*a_1 と S-a_1+a_k (k=2.3.…,n) になるまで続けるんだけど、そこが分かりませぬ。
0871132人目の素数さん
垢版 |
2018/11/15(木) 01:35:19.26ID:BIkI04V5
>>870

>>867 を使わなくても出せるでござる。
A-S≧0,d≧0 のとき
 (A-d)S - A(S-d) = d(A-S) ≧ 0,

ここで
 S = a_1 + a_2 + … + a_n,
 A = k・a_1 + a_{k+1} + … + a_n, (k=n のとき A=n・a_1)
 d = a_1 - a_k ≧ 0,
とおいて
{(k-1)a_1 +(a_k + … +a_n)}S - (k・a_1 +a_{k+1} + … +a_n)(S -a_1 +a_k) = (a_1 -a_k)(k・a_1 -S) ≧ 0,    (k=2,3,…,n)
0872132人目の素数さん
垢版 |
2018/11/15(木) 01:39:19.56ID:BIkI04V5
>>866
〔Jacobsthalの不等式〕
(n-1)個の正の実数 x_1, x_2, …, x_(n-1) の相加平均をA '、相乗平均をG ' とする。
それに x_n (>0) を追加した n個組の相加平均をA_n、相乗平均をG_n とする。このとき
 n(A_n - G_n) ≧ (n-1)(A '-G '),  …[1]
 (A_n/G_n)^n ≧ (A '/G ')^(n-1),  …[2]

(略証)
A_n, G_n, x_n を A, G, x と略記する。
[1]
 n A - (n-1)A '= x,
 n G - (n-1)G '= G '{n(G/G ') - (n-1)} ≦ G '(G/G ')^n = x, (← Bernoulli)
辺々引く。
[2]
 A '(A/A ')^n ≧ A '{n(A/A ') - (n-1)} = n - (n-1)A '= x, (← Bernoulli)
 G '(G/G ')^n = x,
辺々割る。

[1] または [2] を n=1 まで繰り返すと A ≧ G が出る。

ニコニコ大百科
http://dic.nicovideo.jp/a/jacobsthalの不等式
0873132人目の素数さん
垢版 |
2018/11/15(木) 05:32:47.02ID:Kjq0ut8v
>>871
ありがとうございます。
なるほど、>>867を使わずにできますね。
>>871の不等式を使って、残りも同様にしていけばいいんですね。
つまり prime132氏が新証明(?)をしたわけですな。

Guha が1967年に>>867を繰り返し使ってAM-GMを証明した方法も知りたい。
「Guha 1967 AM-GM」をgoogleで検索して一番上に出る
When Less is More: Visualizing Basic Inequalities
のPP.31-32に n=4のときに、Guha's inequality を繰り返し使った例があり、
それを見ても、2回目以降にどう使っているのか分かりません。

Guha's inequality
a≧0, p≧q≧0, x≧y≧0, then
(px+y+a)(x+qy+a)≧((p+1)x+a)((q+1)y+a).

(4A_4)^4
= (a+b+c+d)(a+b+c+d)(a+b+c+d)(a+b+c+d)
≧ (2a+c+d)(2b+c+d)(a+b+c+d)(a+b+c+d) …(1)
≧ (3a+d)(2b+c+d)(b+2c+d)(a+b+c+d)   …(2)
≧ 4a(2b+c+d)(b+2c+d)(b+c+2d)      …(3)
≧ 4a(3b+d)(3c+d)(b+c+2d)         …(4)
≧ 4a(4b)(3c+d)(c+3d)            …(5)
≧ 4a(4b)(4c)(4d)               …(6)
= (4G_4)^4

と書かれているんですが、(1)は(p,q,x,y,a) = (1,1,a,b,c+d)で理解できる。
(2)〜(6)はどう適用したのか謎。
たとえば(2)で (2a+c+d)(a+b+c+d)≧(3a+d)(b+2c+d) となるには
(p,q,x,y,a)に何を対応させているのか?x=a, y=c 以外が謎。
左辺第1因子の形からp=2,q=1でないといけないけど、第2因子に2がない。
0874132人目の素数さん
垢版 |
2018/11/15(木) 06:40:30.77ID:Kjq0ut8v
(3)→(4)は (p,q,x,y,a) = (2,2,b,c,d)で
(5)→(6)は (p,q,x,y,a) = (3,3,c,d,0)か。

じゃあ、残り3ヵ所は、どう適用したんだろう?
(1)→(2) (2a+c+d)(a+b+c+d)≧(3a+d)(b+2c+d)
(2)→(3) (3a+d)(a+b+c+d)≧(4a)(b+c+2d)
(4)→(5) (3b+d)(b+c+2d)≧(4b)(c+3d)

実は使ってないってオチなのか?
0875132人目の素数さん
垢版 |
2018/11/15(木) 08:38:51.30ID:Kjq0ut8v
本人の論文を探すしかないな。
U.C.Guha, arithmetric mean-geometric mean inequality, Mathematical Gazette, 51(1967),pp.14-146

というのは分かったけど、ネットに転がってないかな
0876132人目の素数さん
垢版 |
2018/11/15(木) 09:20:07.76ID:Kjq0ut8v
Handbook of Means and Their Inequalities, pp.101-102 を見たら、
Guhaの不等式を使った証明の数式部分が、省略している部分も含めて
[2] かんどころ P.118と全く同じだった。
0877132人目の素数さん
垢版 |
2018/11/17(土) 07:54:31.62ID:js5kwOKA
(1)
|x|≦1, |y|≦1 (x,y∈R) に対して、
0≦ xx + yy - 2xxyy + 2xy*√{(1-xx)(1-yy)} ≦1.

(2)
m>n>1 (m,n∈Z) に対して、
(m+n+1)!/(m!*n!) > {(m+n)^{m+n}}/{(m^m)(n^n)} > 2^{2n-1}.
0878132人目の素数さん
垢版 |
2018/11/17(土) 11:12:15.05ID:js5kwOKA
(3)
a,b∈C に対して、
|a+b|/(1+|a+b|) < (|a|+|b|)/(1+|a|+|b|) < |a|/(1+|a|) + |b|/(1+|b|)
0879132人目の素数さん
垢版 |
2018/11/18(日) 02:10:29.27ID:ENzLbcND
>>877
(1)
 x = sinθ, y = sinφ (-π/2≦θ,φ≦π/2) とおく。
 √(1-xx) = cosθ, √(1-yy) = cosφ,
 x√(1-yy) + y√(1-xx) = sinθcosφ + sinφcosθ = sin(θ+φ),
両辺を2乗する。

(2)(左)
 log{(m+n+1)!} -(m+n)log(m+n) > (3/2)log(m+n) -(m+n) +0.8918
 log(m!) - m・log(m) < (1/2)log(m) -m +1,
 log(n!) - n・log(n) < (1/2)log(n) -n +1,
辺々引くと
 log{(m+n+1)!} -log(m!) -log(n!) -(m+n)log(m+n) +m・log(m) +n・log(n)
 > (3/2)log(m+n) - (1/2)log(mn) - 1.1082
 > (1/2)log(m+n) + (1/2)log{(m+n)^2 /4mn} + log(2) - 1.1082
 ≧ (1/2)log(m+n) - 0.41505
 ≧ (1/2)log(3) - 0.41505 (m+n≧3)
 = 0.549306

(2)(右)
 (m+n)^{m+n} = (m+n)^{m-n} (m+n)^{2n}
 ≧ m^{m-n} (4mn)^n
 = m^m (4n)^n,

∴ (m+n)^(m+n)/(m^m・n^n) ≧ 4^n,

>>878
(3)
 x/(1+x) は x≧0 で単調増加 (x∈R)
 |a+b| ≦ |a| + |b|
∴ φ(|a+b|) ≦ φ(|a|+|b|)
 = |a|/(1+|a|+|b|) + |b|/(1+|a|+|b|)
 ≦ φ(|a|) + φ(|b|),
0880132人目の素数さん
垢版 |
2018/11/18(日) 02:17:09.96ID:ENzLbcND
>>879 (2) (左)

〔補題〕
log(m!) < (m+1/2) log(m) -m+1,
(略証)
{log(k-1) + log(k)}/2 < ∫[k-1,k] log(x)dx より
log(m!) = Σ[k=2,m] log(k)
< ∫[1,m] log(x)dx + (1/2)log(m)
= [ x・log(x) -x ](x=1,m) + (1/2)log(m)
= (m+1/2)log(m) -m +1,

log(n!) < (n +1/2) log(n) -n+1,
(略証)
{log(k-1) + log(k)}/2 < ∫[k-1,k] log(x)dx より
log(n!) = Σ[k=2,n] log(k)
< ∫[1,n] log(x)dx + (1/2)log(n)
= [ x・log(x) -x ](x=1,n) + (1/2)log(n)
= (n+1/2)log(n) -n +1,

log{(m+n+1)!} > (m+n+3/2) log{(m+n+1)!}
= Σ[k=2,m+n+1] log(k)
> ∫[3/2,m+n+3/2] log(x)dx
= [ x・log(x) -x ](x=3/2,m+n+3/2)
= (m+n+3/2) log(m+n+3/2) -(m+n) -(3/2)log(3/2)
> (m+n+3/2) log(m+n) +(3/2) - (m+n) - (3/2)log(3/2)
= (m+n+3/2) log(m+n) - (m+n) + 0.8918023378
0881132人目の素数さん
垢版 |
2018/11/18(日) 02:23:36.94ID:ENzLbcND
>>879 (2)(左)

log {(m+n+1)!} > (m+n+3/2) log(m+n) - (m+n) + 0.8918
(略証)
log(k) > ∫[k-1/2,k+1/2] log(x)dx より
log{(m+n+1)!} = Σ[k=2,m+n+1] log(k)
> ∫[3/2,m+n+3/2] log(x)dx
= [ x・log(x) -x ](x=3/2,m+n+3/2)
= (m+n+3/2) log(m+n+3/2) -(m+n) -(3/2)log(3/2)
> (m+n+3/2) log(m+n) +(3/2) - (m+n) - (3/2)log(3/2)
= (m+n+3/2) log(m+n) - (m+n) + 0.8918023378
0882132人目の素数さん
垢版 |
2018/11/18(日) 20:34:10.43ID:Fcj0HO3Z
>>877
(1)
右辺-中辺 = [xy - √{(1-xx)(1-yy)}]^2 ≧0,
中辺-左辺 = {x√(1-yy) + y√(1-xx)}^2 ≧0.

 (゚∀゚ )
  ノヽノ) =3 プゥ
  くく
0886132人目の素数さん
垢版 |
2018/11/18(日) 23:25:53.36ID:Fcj0HO3Z
a,b,c>0, a+b+c=1に対して、(1+ 1/a)(1+ 1/b)(1- 1/c) の取りうる値の範囲を求めよ。
0888132人目の素数さん
垢版 |
2018/11/19(月) 01:54:17.81ID:eL1RQpps
〔問題168〕
a,b,c>0 のとき
 (aa-bc)(b+c)^r + (bb-ca)(c+a)^r + (cc-ab)(a+b)^r ≧ 0, (0<r<1)
                          ≦ 0, (r>1, r<0)
  V.Cirtoaje:"Algeblaic inequalities"、1-1-7
 inequalitybot [168]
0889132人目の素数さん
垢版 |
2018/11/19(月) 02:03:20.14ID:eL1RQpps
>>888

 x = (b+c)^r,
 y = (c+a)^r,
 z = (a+b)^r,
とおくと
 a = (y^{1/r} + z^{1/r} - x^{1/r})/2,
 b = (z^{1/r} + x^{1/r} - y^{1/r})/2,
 c = (x^{1/r} + y^{1/r} - z^{1/r})/2,

 aa-bc = {y^(2/r) +z^(2/r) -x^(1/r)[y^(1/r) + z^(1/r)]}/2,
 bb-ca = {z^(2/r) +x^(2/r) -y^(1/r)[z^(1/r) + x^(1/r)]}/2,
 cc-ab = {x^(2/r) +y^(2/r) -z^(1/r)[x^(1/r) + y^(1/r)]}/2,

(左辺) = (aa-bc)x + (bb-ca)y + (cc-ab)z
 = {x^(2/r)y +xy^(2/r) -(x+y)(xy)^(1/r)}/2 + ……
 = (x^{1/r} - y^{1/r})(x^{1/r -1} - y^{1/r -1})xy + ……
0890132人目の素数さん
垢版 |
2018/11/19(月) 09:27:44.83ID:eL1RQpps
>888 訂正

〔問題168〕
a,b,c>0 のとき
 (aa-bc)(b+c)^r + (bb-ca)(c+a)^r + (cc-ab)(a+b)^r > 0, (r<1)
                          < 0, (r>1)
                          = 0, (r=1)
0891132人目の素数さん
垢版 |
2018/11/19(月) 09:57:43.41ID:eL1RQpps
>>885
y>0 とする。
 (1 + y/2)^2 > 1+y > 1,
∴ 1/(1+y/2)^2 < 1/(1+y) < 1,
0〜y で積分すると
 y/(1+y/2) < log(1+y) < y,
∴ (1+y)^(1/y) < e < (1+y)^(1/y + 1/2),
y=1/x とおく。
0894132人目の素数さん
垢版 |
2018/11/19(月) 10:34:04.63ID:eL1RQpps
>>892

log(k) > (1/2)log(kk-dd) = {log(k+d) + log(k-d)}/2,

y=log(x) は上に凸だから、x=kでの接線より下側にある。
k-d<x<k+d かつ接線より下の台形の面積は(接線の傾きによらず)2d log(k)
∴ 2d log(k) > ∫[k-d,k+d] log(x)dx
0897132人目の素数さん
垢版 |
2018/11/19(月) 13:19:03.28ID:Merao6vt
>>896
無理だった。

>>881
> = (m+n+3/2) log(m+n+3/2) -(m+n) -(3/2)log(3/2)
> > (m+n+3/2) log(m+n) +(3/2) - (m+n) - (3/2)log(3/2)

の部分で、以下はどうやって分かるのですか?
(m+n+3/2) log(m+n+3/2) > (m+n+3/2) log(m+n) + (3/2)
0900132人目の素数さん
垢版 |
2018/11/20(火) 00:02:55.19ID:5+1z6eBQ
>>899
さんくす。

x=a(>0) における log x の接線を考えて、
 (x-a)/a + log a ≧ log x.
x=1, a = (N+d)/N を代入すればいいのかな。
0902132人目の素数さん
垢版 |
2018/11/20(火) 02:59:32.68ID:5+1z6eBQ
>>877(2)左側

0≦x≦1 において f(x) = x^m (1-x)^n は x = m/(m+n) で最大値をとる.

I(m,n) = ∫[0,1] f(x)dx とおくと, I(m,n) ≦ f(m/(m+n)) より
(m!*n!)/{(m+n+1)!} ≦ {(m^m)(n^n)}/{(m+n)^{m+n}}

[東京医科歯科大学2013数学第3問]
0903132人目の素数さん
垢版 |
2018/11/21(水) 16:55:51.41ID:LdWYnCJ+
>>856-857
大昔のPutnumに、これより弱い不等式があったよね。

>>885
Moreau's inequality が思い浮かぶと同時に、一松先生を思い出す。(謎掛け)
0904132人目の素数さん
垢版 |
2018/11/21(水) 21:37:39.12ID:LdWYnCJ+
三角形の辺長 a,b,c に対して、
(1) Σ[cyc] aa(b+c-a) ≦3abc.
(2) Σ[cyc] aab(a-b) ≧0.

そもそも(1)は辺長でなくても非負実数で成り立つでおじゃるな。
0905132人目の素数さん
垢版 |
2018/11/22(木) 00:31:23.80ID:x/Au2Ugh
>>904
(1)
(右辺) - (左辺) = a(a-b)(a-c) + b(b-c)(b-a) + c(c-a)(c-b) = F1(a,b,c) ≧ 0,
 △である必要はない。

(2)
a = y+z、b = z+x、c = x+y とおく。(Ravi変換)
(左辺) = aab(a-b) + bbc(b-c) + cca(c-a)
= 2(xyyy+yzzz+zxxx) - 2xyz(x+y+z)
= (2/7)(2xyyy +yzzz +4zxxx -7xxyz) + cyc.
≧ 0,
 IMO-1983, A.6
 文献[9] 佐藤(訳) (2013) 問題2.24
 Inequalitybot [24]
0908132人目の素数さん
垢版 |
2018/11/22(木) 02:23:23.48ID:x/Au2Ugh
>>856 >>903 >>906

27th Putnum-1966
A2.
A triangle has sides a, b, c. The radius of the inscribed circle is r. Show that
 1/(b+c-a)^2 + 1/(c+a-b)^2 + 1/(a+b-c)^2 ≧ 1/(2r)^2,
0913132人目の素数さん
垢版 |
2018/11/22(木) 16:30:26.57ID:x/Au2Ugh
>>910

√(x-1) = X,
√(y-1) = Y,
とおく。
XX-X+1 ≧ X,
YY-Y+1 ≧ Y,
(右辺) - (左辺) = xy - x√(y-1) - y√(x-1)
 = (XX+1)(YY+1) - (XX+1)Y - (YY+1)X
 = {(XX+1)-X} {(YY+1)-Y} - XY
 ≧ XY - XY
 = 0,

あるいは
x = (cosh u)^2, y = (cosh v)^2 とおく。
0917132人目の素数さん
垢版 |
2018/11/23(金) 11:36:47.43ID:hLVWs+G2
>>916 は「美しい不等式pp.69-70」にあるが、証明が美しくないよな。
普通に差をとったら綺麗にできるのになあ。
0920132人目の素数さん
垢版 |
2018/11/23(金) 15:59:31.52ID:eRIDJVQi
>>916 >>917
 x = (b+c-a)/2, y = (c+a-b)/2, z = (a+b-c)/2,
とおく。(Ravi変換)
普通に差をとったら出来ますね^^
 2bc - a(b+c-a) = 2(x+z)(x+y) - 2x(y+z) = 2xx + 2yz ≧ 0,

文献[9] 佐藤(訳) §2.2 例2.2.1 p.69 (2013)
0921132人目の素数さん
垢版 |
2018/11/23(金) 16:55:45.35ID:hLVWs+G2
>>917 >>920
なるほど、Ravi変換は無敵でござるな。

この問題を問題集で見て感じたのは、三角形の成立条件を使った例なのに、
三角形の成立条件が一目で分かりにくい小汚い計算をしていた点。

b,cについて対称だから b≧cとする所まではいい。次のようにした方が美しいと思わん?

aが最大または最小のとき、
 2bc - a(b+c-a) = bc + (a-b)(a-c) > 0.

aがbとcの間の数のとき、b≧a≧cだから、
 2bc - a(b+c-a) = (a-c)(c+a-b) + c(b+c-a) > 0.
0923132人目の素数さん
垢版 |
2018/11/24(土) 01:13:59.36ID:OpCiwKZy
>>922
a, b, c, r > 0 に対して (ab)^{r+1/2} (aa+bb-2cc) ≧ (ab-cc) (a+b) (cc)^r,

(略証)
(左辺) - (右辺)
≧ (a+b)(ab)^{r+1} - (a+b)(ab)^r・cc - (ab-cc)(a+b)(cc)^r
= (a+b)(ab-cc) [(ab)^r - (cc)^r]
≧ 0,
0924132人目の素数さん
垢版 |
2018/11/24(土) 01:30:44.11ID:R0eGczxp
>>915 + >>922
a, b >0 に対して、aabb(aa+bb-2) ≧ ab(ab-1)(a+b) ≧ (ab-1)(a+b).

つまり改造後の不等式は、より強い式でござる ( ゚∀゚) ウヒョッ!
調子に乗って、さらに改造すると、

a, b >0 に対して、{√(ab)}*(aa+bb-2) ≧ (ab-1)(a+b).
0926132人目の素数さん
垢版 |
2018/11/24(土) 04:44:14.38ID:OpCiwKZy
>>923
(左辺) ≧ (a+b)(ab)^{r+1} - (a+b)(ab)^r・cc
= (a+b)(ab-cc)(ab)^r
≧ (a+b)(ab-cc)(ab)^(r-1) cc
≧ ……
≧ (a+b)(ab-cc)(ab)(cc)^(r-1)
≧ (a+b)(ab-cc)(cc)^r
= (右辺),
0927132人目の素数さん
垢版 |
2018/11/24(土) 22:03:32.05ID:R0eGczxp
(1) a,b,c∈Rに対して、
  (a^5+b^5+c^5)^2 ≧ 3abc(a^7+b^7+c^7).
(2) x,y,z>0, xyz=1に対して、
  (x^10+y^10+z^10)^2 ≧ 3(x^13+y^13+z^13).
0929132人目の素数さん
垢版 |
2018/11/25(日) 00:37:10.99ID:AuW29Ma5
>>522 (D1)

 f '(x) < (3/2)^(1/3) f(x) = 1.14471424 f(x),

60th Putnam (1999/Dec/04) B-4

〔補題〕
 lim(x→-∞) F(x) ≧0,
 F '(x) > 0 for all x∈R
ならば
 F(x) > 0 for all x∈R
(背理法で示せる。)

 g(x) = (3/2)f(x)^3 - {f '(x)}^3,
とおくと
 g '(x) = 3f '(x) {(3/2)f(x)^2 - f '(x)f "(x)} ≡ 3f '(x) h(x),
 h '(x) = 3f(x) f '(x) - f '(x) f '''(x) - {f "(x)}^2
  = f '(x) {f(x) - f '''(x)} + {2f(x) f '(x) - [f "(x)]^2}
  ≡ f '(x) {f(x) - f '''(x)} + L(x),
 L '(x) = 2f '(x){f(x) - f '''(x)} + {f '(x)}^2 > 0,
補題により
 L(x) = 2f(x) f '(x) - [f "(x)]^2 > 0,
 h '(x) > 0,
補題により
 h(x) = (3/2)f(x)^2 - f '(x)f "(x) > 0,
 g '(x) > 0,
補題により
 g(x) = (3/2)f(x)^3 - {f '(x)}^3 > 0,
 f '(x) < (3/2)^(1/3) f(x),
0933132人目の素数さん
垢版 |
2018/11/25(日) 22:54:06.18ID:Lv92uXqz
(1)
a,b,c∈R, r>0 に対して、
a(b+c)^r + a(b+c)^r + a(b+c)^r ≧0.

(2)
a,b,c>0 に対して
√(aa+ab+bb) + √(bb+bc+cc)√(cc+ca+aa) ≧ 3√(ab+bc+ca).

(3)
a,b,c∈R に対して、次式をみたすkの最大値を求めよ.
abc(a+b+c)^2 ≧ k(ab+bc+ca)(a+b-c)(b+c-a)(c+a-b).

------------------------------------------
http://www.imomath.com/othercomp/Journ/ineq.pdf
1831, 1394, 1120
0934132人目の素数さん
垢版 |
2018/11/25(日) 23:17:46.77ID:AuW29Ma5
>>931

>>533 >>534 より

(A^3 + B^3 + C^3)^2 - (AB+BC+CA)(A^4 + B^4 + C^4) = F_0(A, B, C) F_0(AA, BB, CC) + F_1(BC, CA, AB) ≧ 0,

F_n(x,y,z) = (x^n)(x-y)(x-z) + (y^n)(y-z)(y-x) + (z^n)(z-x)(z-y) ≧ 0,
0935132人目の素数さん
垢版 |
2018/11/26(月) 04:45:07.58ID:JAq6ovHt
>>933

1831. (p.74)
 a,b,c ∈ R, r>0 は奇数 のとき
 a(a+b)^r + b(b+c)^r + c(c+a)^r ≧ 0,

1394. (p.51)
(略解)
AM-GM で
(左辺) ≧ 3{(ab+bb+aa)(bb+bc+cc)(aa+cc+ca)}^(1/6)
 ≧ 3√(ab+bc+ca),   (←コーシー)

1120. (p.34)
(略解)
a,b,c ≧ 0 とする。
{b+c-a, c+a-b, a+b-c} の中の2つの和は非負だから、負であるものは高々1つ。
いずれかが負の場合は、任意のk>0 について
(左辺) ≧ 0 ≧ (右辺).
以下では b+c-a≧0, c+a-b≧0, a+b-c≧0, k=3 とする。
(左辺) - (右辺) = abc(a+b+c)^2 - 3(ab+bc+ca)(a+b-c)(b+c-a)(c+a-b)
= c(a+b-c)(3a+3b-c)(a-b)^2 + a(b+c-a)(3b+3c-a)(b-c)^2 + b(c+a-b)(3c+3a-b)(c-a)^2 ≧ 0,
等号成立は a=b=c のとき。
0936132人目の素数さん
垢版 |
2018/11/26(月) 05:26:09.09ID:JAq6ovHt
>>933

1126. (p.34)
 0 < x ≦ 1 に対して次を示せ。
 x < sinh(x) < 3x/{1+1+√(1-xx)} < tan(x),

1270. (p.44)
 x>0 に対して次を示せ。
 x/√(1+xx) < tanh(x) < √{1-exp(-xx)} < x,
0937132人目の素数さん
垢版 |
2018/11/26(月) 06:28:31.22ID:P3VGSuRj
>>933 >>935
> (2)
> a,b,c>0 に対して
> √(aa+ab+bb) + √(bb+bc+cc)√(cc+ca+aa) ≧ 3√(ab+bc+ca).

my collection に次式を発見、しかし詳細不明。
a,b,c∈R に対して、
√(aa+ab+bb) + √(bb+bc+cc)√(cc+ca+aa) ≧ (3/2)*(a+b+c).
0938132人目の素数さん
垢版 |
2018/11/26(月) 09:39:29.12ID:P3VGSuRj
>>859-861
n≧4では、逆向きが成り立つという仮説を立ててみた。
n=2,3のとき、 (3/4)*(1 + A/H)^2 ≧ (A/G)^n + (G/H)^n + 1
n≧4のとき、 (3/4)*(1 + A/H)^2 ≦ (A/G)^n + (G/H)^n + 1
0941132人目の素数さん
垢版 |
2018/11/27(火) 02:28:27.59ID:QMhuYErk
>>938
n≧4 のとき
(右辺) = (A/G)^n + (G/H)^n + 1^n
≧ 2(A/H)^(n/2) + 1   (AM-GM)
≧ (3/4){1 + (A/H)^(n/4)}^2,

2xx +1 - (3/4)(x+1)^2 = (x-1)(5x-1)/4 ≧ 0, (x≧1)

>>939
That's what I wanna know. (それは こっちが訊きたい...)

>>940
5点で等号成立ですね…
(a,b,c,d) = (1,1,1,1) (3,1,1,1) (1,3,1,1) (1,1,3,1) (1,1,1,3)
0942132人目の素数さん
垢版 |
2018/11/27(火) 03:19:22.59ID:QMhuYErk
>>937
 xx+xy+yy = (3/4)(x+y)^2 + (1/4)(x-y)^2 ≧ (3/4)|x+y|^2,
より
 (左辺) ≧ (√3)/2・(|a+b|+|b+c|+|c+a|)
 ≧ (√3)/2・|2a+2b+2c|
 = (√3)|a+b+c|,

ab+bc+ca ≧ 0 ならば 1394. が成立。 >>933 (2)
0944132人目の素数さん
垢版 |
2018/11/27(火) 03:51:33.88ID:oixSVMNZ
>>937
a,b,c∈R に対して、
√(aa+ab+bb) + √(bb+bc+cc) + √(cc+ca+aa) ≧ (3/2)*|a+b+c|.

[証]
xx+xy+yy - (x+ y/2)^2 = (3/4)*y^2 ≧0
∴ √(xx+xy+yy) ≧ |x+ y/2|

√(aa+ab+bb) + √(bb+bc+cc) + √(cc+ca+aa)
≧ |a+ b/2| + |b+ c/2| + |c+ a/2|
≧ |(a+ b/2) + (b+ c/2) + (c+ a/2)|
= (3/2)*|a+b+c|.

等号成立条件は a=b=c=0.
---------------------

>>942の等号成立条件は a=b=c だから、上式は緩くて次式が良いってことですかね?
√(aa+ab+bb) + √(bb+bc+cc) + √(cc+ca+aa) ≧ (√3)|a+b+c|.
0946132人目の素数さん
垢版 |
2018/11/27(火) 17:31:28.13ID:oixSVMNZ
>>911で紹介して頂いたサイトから検索してみた。

>>887
>  n=4 のとき、(A-G)/(G-H) ≧ 9/16
>  CGMO-2011 A.4
>  inequalitybot [35]

a,b,c,d>0, abcd=1に対して、
1/a + 1/b + 1/c + 1/d + 9/(a+b+c+d) ≧(25/4).

(3数) https://artofproblemsolving.com/community/c6h497071
(3数) https://artofproblemsolving.com/community/c6h506861p2847126
(4数) https://artofproblemsolving.com/community/c6h422665p2389389
(n数) https://artofproblemsolving.com/community/c6h354052
(類題)http://www.mathoe.com/dispbbs.asp?boardID=107&;ID=44556 →>>940

うーむ、分からん…。
0947132人目の素数さん
垢版 |
2018/11/27(火) 23:19:46.82ID:QMhuYErk
>>938
 n≧4 のとき
 (A/G)^n + (G/H)^n + 1 ≧ 3([(A/G)^2 + (G/H)^2 + 1]/3)^{n/2} ≧ 3([(A/H) + (A/H) + 1]/3)^{n/2},

 (A/G)^n + (G/H)^n + 1 ≧ (A/H)^{n/2} + (A/H)^{n/2} + 1≧ 3([(A/H) + (A/H) + 1]/3)^{n/2},

もある…
0953132人目の素数さん
垢版 |
2018/12/04(火) 04:30:49.15ID:MMKOwiX9
>>950

s = a+b+c,u = abc とおく。

(左辺) - (右辺)
= {c(a-b)^2 + a(b-c)^2 + b(c-a)^2}/u - (8/ss){(a-b)^2 + (b-c)^2 + (c-a)^2}
= (c/u -8/ss)(a-b)^2 + (a/u -8/ss)(b-c)^2 + (b/u -8/ss)(c-a)^2
= p(a-b)(a-c) + q(b-c)(b-a) + r(c-a)(c-b),

(b+c)ss ≧ 4a(b+c)^2 ≧ 4a(4bc) = 16u より p = (b+c)/u -16/ss ≧ 0,
同様にして
 q = (c+a)/u - 16/ss ≧ 0,
 r = (a+b)/u - 16/ss ≧ 0,
また、(a,b,c) と (p,q,r) は逆順序だから、Schur の拡張により
 p(a-b)(a-c) + q(b-c)(b-a) + r(c-a)(c-b) ≧ 0,
0954132人目の素数さん
垢版 |
2018/12/04(火) 04:47:43.20ID:OyfgKqsW
>>953
瞬殺ですな、乙でござる。

さて、>>950を書き込む際に出典を探したが見つけらず。
おそらく Vasile Cirtoaje だろうが、検索したが閲覧できず。
2年前には未完成のpdfが閲覧できたが、2018.07以降の書籍化が原因だろう。

7.Cirtoaje V. - Mathematical Inequalities, Volumes 1-5 (p. 344, 400, 486, 522, 544), Lambert Academic Publishing, 2018.
http://ac.upg-ploiesti.ro/vcirtoaje/vcirtoaje.php
0956132人目の素数さん
垢版 |
2018/12/04(火) 15:04:24.55ID:MMKOwiX9
>>950
4点で等号が成立
(a,b,c) = (1,1,1) (2,1,1) (1,2,1) (1,1,2)
          p=0  q=0  r=0
 
…てことは、これで最良でござる。  >>955
0957132人目の素数さん
垢版 |
2018/12/04(火) 23:43:56.06ID:OyfgKqsW
>>956
ほんとだ、不等号の向きを勘違いしていた。つまりこういうことですな。

a,b,c>0 に対して、
(a+b+c)(1/a + 1/b + 1/c) ≧ 1 + 24(aa+bb+cc)/{(a+b+c)^2}.≧9.
0958132人目の素数さん
垢版 |
2018/12/05(水) 00:21:40.27ID:FGF1AG7S
(1) a,b,c∈R に対して、(aa+bb+cc)^2 ≧ 3(a^3b+b^3c+c^3a).

(2) a,b,c,d>0, abcd=1 に対して、a^4b+b^4c+c^4d+d^4a ≧ a+b+c+d.

(3) a,b,c>0 に対して、3/4 ≦ Σ[cyc] ab/{(b+c)(c+a)} < 1.
0959132人目の素数さん
垢版 |
2018/12/05(水) 18:21:09.37ID:fLK6/i8S
>>958
(1)
 A = aa -bb +3bc,
 B = bb -cc +3ca,
 C = cc -aa +3ab,
とおくと
(左辺) - (右辺) = {(A-B)^2 + (B-C)^2 + (C-A)^2}/6 ≧ 0,
 >>244 (1) >>247

(2) AM-GMで
 {23(a^4)b +7(b^4)c +11(c^4)d +10(d^4)a}/51 ≧ a(abcd) = a,
巡回的にたす。

(3)
右)
 (中辺) = 1 - 2abc/{(a+b)(b+c)(c+a)} < 1,
左)
 (中辺) - 3/4 = {(a+b)(b+c)(c+a)-8abc}/(a+b)(b+c)(c+a) ≧ 0, (AM-GM)
0960132人目の素数さん
垢版 |
2018/12/06(木) 01:12:38.36ID:9xq/7gl/
>>958 (2)
ついでだけど、n文字の場合も AM-GM で
 Σ[j=1,n] k_j (a_j)^n a_{j+1} ≧ a_1(a_1・a_2…a_n) = a_1,
巡回的にたす。
 k_j = 1/(n+1) - (-1)^j・n^{n-j}/[n^n - (-1)^n] > 0,
 a_{n+1} = a_1,
0963132人目の素数さん
垢版 |
2018/12/09(日) 16:38:35.94ID:GNoL1vgk
a,b,c>0, aa+bb+cc=3 に対して、
(a^5)/(b^3+c) + (b^5)/(c^3+a) + (c^5)/(a^3+b) ≧ (3/2)*(abc)^2.

バスク大佐の不等式を使って証明できるらしい…
0964132人目の素数さん
垢版 |
2018/12/10(月) 04:00:08.91ID:IsN+FPDR
>>963
aa+bb+cc = S とおく。
a(b^3 +c) + b(c^3 +a) + c(a^3 +b) = (ab^3 +bc^3 +ca^3) + (ab+bc+ca) ≦ SS/3 + S,  (>>958 (1))

コーシーで
{a(b^3 +c) + b(c^3 +a) + c(a^3 +b)}(左辺) ≧ (a^3 +b^3 +c^3)^2 ≧ (aa+bb+cc)^3 /(1+1+1) = (1/3) S^3,

∴ (左辺) ≧ (S^3)/(SS+3S) = SS/(S+3),
0965132人目の素数さん
垢版 |
2018/12/10(月) 14:28:08.88ID:IsN+FPDR
>>959 (1)
4点で等号成立。
A=B=C より
(a, b, c) = (1, 1, 1) (1, 1+t, 3+1/t) = (1, 1+t, tt-3t-1)
 t は t^3 -3t^2 -4t -1 = 0 の根
 t = -0.69202147163 -0.3568958679 4.0489173395
0966132人目の素数さん
垢版 |
2018/12/13(木) 13:13:26.57ID:7ZQJsx3/
n次元ベクトル a_1、…、a_m の相加平均を A = (a_1+…+a_m)/m とおく。
任意のn次元ベクトル x に対して、
Σ[k=1 to m] |a_k - x|^2 = Σ[k=1 to m] |a_k - A|^2 + m*|x - A|^2.

むむっ、不等式じゃないな…
0967132人目の素数さん
垢版 |
2018/12/13(木) 17:55:55.30ID:hPiKmmhx
どれも-1以上である実数a,b,c,d,eはa+b+c+d+e=5であるという
このときの(a+b)(b+c)(c+d)(d+e)(e+a)の最大値と最小値を求めよ

2019年度中国数学オリンピック第一問
0968132人目の素数さん
垢版 |
2018/12/14(金) 06:52:26.17ID:DbCBFUHo
>>966

n次元ベクトル y = (y_1, y_2, …, y_n) に対して
|y|^2 = (y_1)^2 + (y_2)^2 + …… + (y_n)^2
∴ n個の成分に分けて考え、和をとればよい。
∴ n=1 (スカラー) の場合に帰着する。

a_k - x = (a_k - A) - (x - A),
(a_k -x)^2 = (a_k - A)^2 - 2(a_k - A)(x - A) + (x - A)^2,
となるが、右辺第2項は 和をとれば
納k=1,m] (a_k - A) = (a_1 + a_2 + … + a_m) - mA = 0,
となって消える。
0969132人目の素数さん
垢版 |
2018/12/15(土) 02:39:44.86ID:xhjRoR3J
>>967
 最大値 32  a=b=c=d=e=1 のとき。
 最小値 -512 {a,b,c,d,e} = {9,-1,-1,-1,-1} のとき。
かな
0970132人目の素数さん
垢版 |
2018/12/15(土) 03:42:27.66ID:xhjRoR3J
>>951
大阪の「三豊麺」もある。
http://sanpomen.jp/
 
重量(茹で上がり)
 並盛 1玉  350g
 大盛 1.5玉 550g
 特盛 2玉  750g
 山盛 2.5玉 900g (+100円)

(ミツトヨの秤で量ったんぢゃないけど)

大阪のみなみ周辺では、千日前、体育館前、日本橋に店舗があります。
0971132人目の素数さん
垢版 |
2018/12/15(土) 10:03:02.54ID:iiLDF3is
>>970
もしかして面白いと思って書いているのだとしたら、反省した方がいいでござるよ。
荒らすのは止めましょう。
0973132人目の素数さん
垢版 |
2018/12/15(土) 10:20:56.92ID:iiLDF3is
>>968-969>>972
たとえば、(a,b,c,d,e) = (-1,-1,-1,4,4)のとき、288.

この手の不等式を見たときに、方針がぱっと出てこない悲しさ。
0974132人目の素数さん
垢版 |
2018/12/15(土) 10:43:58.04ID:iiLDF3is
最大値や最小値をとるときの変数に、限界値の-1が入るのは理由があるのかな?
0975132人目の素数さん
垢版 |
2018/12/16(日) 13:25:07.42ID:u7Il/9pT
>>967
(1) a,b,c>-1, a+b+c=3 のとき、-32≦(a+b)(b+c)(c+a)≦8.
(2) a,b,c,d>-1, a+b+c=4 のとき、-48≦(a+b)(b+c)(c+a)≦144.
(3) a,b,c,d,e>-1, a+b+c+d+e=5 のとき、-512≦(a+b)(b+c)(c+d)(d+e)(e+a)≦288.
0976132人目の素数さん
垢版 |
2018/12/16(日) 13:54:54.32ID:u7Il/9pT
書き直し
>>967
(1) a,b,c>-1, a+b+c=3 のとき、-32≦(a+b)(b+c)(c+a)≦8.
(2) a,b,c,d>-1, a+b+c=4 のとき、-48≦(a+b)(b+c)(c+d)(d+a)≦144.
(3) a,b,c,d,e>-1, a+b+c+d+e=5 のとき、-512≦(a+b)(b+c)(c+d)(d+e)(e+a)≦288.
0979132人目の素数さん
垢版 |
2018/12/16(日) 15:54:47.11ID:u7Il/9pT
もう一度書き直し
>>967
(1) a,b,c>-1, a+b+c=3 のとき、-32≦(a+b)(b+c)(c+a)≦8.
(2) a,b,c,d>-1, a+b+c+d=4 のとき、-48≦(a+b)(b+c)(c+d)(d+a)≦144.
(3) a,b,c,d,e>-1, a+b+c+d+e=5 のとき、-512≦(a+b)(b+c)(c+d)(d+e)(e+a)≦288.
0980132人目の素数さん
垢版 |
2018/12/17(月) 06:11:59.37ID:X9YWLOp7
〔予想〕
 a_1, a_2, …, a_n ≧ -1, a_1+a_2+…+a_n = n, のとき

・n:奇数 (n≧5) ならば
 -(2^n)(n-1)^2 ≦ Π(a_j + a_{j+1}) ≦ 2^{n-2} (n-2)^2 (n-1),

・n:偶数 ならば
 -2^{n-2} (n-2)^2 (n-1) ≦ Π(a_j + a_{j+1}) ≦(2^n)(n-1)^2,

「良そう」「止そう」と意見が割れるかも知りませぬが…

(不等式スレも平成のうちに2桁に到達でござる。思えば長い道でござった。)
0981132人目の素数さん
垢版 |
2018/12/17(月) 15:34:10.00ID:zsFrN7jo
第1章が2003年。
スレ立て放置されていた不等式スレを占拠して15年も経つのか…。
0982132人目の素数さん
垢版 |
2018/12/17(月) 17:41:33.58ID:X9YWLOp7
[初代スレ.019]
いいよ!

[初代スレ.034]
正の数 a,b,x,y,X,Y に対して
 (axX+byY)^3 ≦ (a^3+b^3)(x^3+y^3)(X^3+Y^3),

(右辺) - (左辺)
= (axY)^3 + (ayX)^3 + (bxX)^3 -3(byY)(axX)^2
 + (bxY)^3 + (byX)^3 + (ayY)^3 -3(axX)(byY)^2
= (axY)^3 + (ayX)^3 + (bxX)^3 -3(axY)(ayX)(bxX)
 + (bxY)^3 + (byX)^3 + (ayY)^3 -3(bxY)(byX)(ayY)
≧ 0,   (AM-GM)
スッキリ。

[初代スレ.039]
(参考書)
秋山 仁 + ピーター・フランクル「[完全攻略] 数学オリンピック」日本評論社 (1991/Nov)
p.24-25 [例4-3]
0983132人目の素数さん
垢版 |
2018/12/17(月) 20:12:55.21ID:zsFrN7jo
懐かしい。当時は tanスレや nCrスレや、おいらには解けないスレとかあったよなぁ…。
パソコンを何度も買い替えて、今となっては過去ログが見れないが。
0984132人目の素数さん
垢版 |
2018/12/18(火) 03:40:20.65ID:e1oKVpnI
>>979
(1) a+b, b+c, c+a のうち負は高々1個。

a+b, b+c, c+a ≧0のとき、AM-GMより、
0 ≦ (a+b)(b+c)(c+a) ≦ [ {(a+b)+(b+c)+(c+a)}/3]^3 = 8.

1つだけ負のとき、対称性から a+b < 0 ≦ b+c, c+a としてよい。
このとき、条件より 3<c≦5 で、AM-GMより、
0 ≦ -2(a+b)(b+c)(c+a) ≦ [ {-2(a+b)+(b+c)+(c+a)}/3]^3 = (c-1)^3 ≦ 64.
(証明終)
0985132人目の素数さん
垢版 |
2018/12/18(火) 07:27:44.02ID:TqVGX/9j
〔補題〕
正の実数a,b,cに対して
[1] (b/a) + (c/b) + (a/c) - {a/(b+c) + b/(c+a) + c/(a+b)} ≧ (ab+bc+ca)^2 /{2(a+b+c)abc} ≧ 3/2,
[2] a/(b+c) + b/(c+a) + c/(a+b) ≧ (a+b+c)^2 /2(ab+bc+ca) ≧ 3/2,

(略証)
[1]
(左辺) - (右辺)
 = ab/(c(b+c)) + bc/(a(c+a)) + ca/(b(a+b))
 = (ab+bc+ca)^2 /{abc[(b+c) + (c+a) + (a+b)]}
 ≧ (ab+bc+ca)^2 /{2(a+b+c)abc},   (←コーシー)
 JMO-2004、[初代スレ.058]

[2] もコーシーで出る。

なお、1/a = A, 1/b = B, 1/c = C とおくと
 (ab+bc+ca)^2 /{(a+b+c)abc} = (A+B+C)^2 /(AB+BC+CA),
 (a+b+c)^2 /(ab+bc+ca) = (AB+BC+CA)^2 /{(A+B+C)ABC},

(類) Nesbitt-Igarashi  >>627 >>835
0987132人目の素数さん
垢版 |
2018/12/18(火) 19:23:44.68ID:TqVGX/9j
>>985
 辺々たして…

[3] (b/a) + (c/b) + (a/c) ≧ tt/2su + ss/2t ≧ √(st/u) ≧ 3,
 ここに s = a+b+c, t = ab+bc+ca, u = abc.

 st-9u = a(b-c)^2 + b(c-a)^2 + c(a-b)^2 ≧ 0,
0988132人目の素数さん
垢版 |
2018/12/18(火) 19:41:13.00ID:TqVGX/9j
>>986

{-1,……,-1,2n-1} のとき (-1)^n・(2^n)(n-1)^2
{-1,…,-1,n-1,n-1} のとき - (-1)^n・2^{n-2} (n-2)^2 (n-1)
かとオモタ。
0989132人目の素数さん
垢版 |
2018/12/18(火) 21:15:32.22ID:e1oKVpnI
>>979
よく見たら、条件式のa,b,c>-1のところが、書き間違ってるな。
正しくは a,b,c≧-1など。
0990132人目の素数さん
垢版 |
2018/12/18(火) 21:42:52.76ID:e1oKVpnI
>>979
(1)
a+1=A, b+1=B, c+1=C とおくと、問題は
『A,B,C≧0, A+B+C=6 のとき、32≧(A-4)(B-4)(C-4)≧-8』.

A+B+C=s(=6), AB+BC+CA=t, ABC=u とおくと、不等式は 0≦4t-u≦40.

う〜む、非同次は難しい。
s=6だから、無理やり同時にすると、0≦2st-3u≦120.
これではダメか…。
0991132人目の素数さん
垢版 |
2018/12/18(火) 23:13:50.02ID:TqVGX/9j
>>990 (1)
s, t, u で表わせば
(与式) = (a+b)(b+c)(c+a)
 = (A+B - s/3)(B+C - s/3)(C+A - s/3)
 = (2s/3 - C)(2s/3 - A)(2s/3 - B)
 = -4(s/3)^3 + (2/3)st -u,

(与式) = - 4(s/3)^3 + (1/9){5st+(st-9u)} ≧ -4(s/3)^3,
(与式) = (s/3)^3 - (2/27)s(ss-3t) - (1/9)(s^3 -4st+9u) ≦ (s/3)^3,
0992132人目の素数さん
垢版 |
2018/12/19(水) 04:07:16.90ID:K5b8go44
>>990の続きだけど、これで合ってるかな?

> (1) a,b,c>-1, a+b+c=3 のとき、-32≦(a+b)(b+c)(c+a)≦8.
>
> a+1=A, b+1=B, c+1=C とおくと、問題は
> 『A,B,C≧0, A+B+C=6 のとき、32≧(A-4)(B-4)(C-4)≧-8』.
>
> A+B+C=s(=6), AB+BC+CA=t, ABC=u とおくと、不等式は 0≦4t-u≦40.


s=6を使って、同次化して、0 ≦ 9(2st-3u) ≦ 5s^3.
これを証明する。

右側 : 5s^3 - 9(2st-3u) = 5(s^3-4st+9u) + 2(st-9u) ≧0.
左側 : 9(2st-3u) = 15st + 3(st-9u) ≧0.
0993132人目の素数さん
垢版 |
2018/12/19(水) 06:03:45.97ID:5zoTD2o3
>>992
 正解です!

なお、
st - 9u = a(b-c)^2 + b(c-a)^2 + c(a-b)^2 ≧ 0,

s^3 - 4st + 9u = a(a-b)(a-c) + b(b-c)(b-a) + c(c-a)(c-b)
= {ab(aa-bb)^2 + bc(bb-cc)^2 + ca(cc-aa)^2}/{(a+b)(b+c)(c+a)}
≧ 0,

http://dic.nicovideo.jp/a/シューアの不等式
0995132人目の素数さん
垢版 |
2018/12/19(水) 06:44:01.18ID:5zoTD2o3
>>984 >>986 >>989 >>990 >>992 >>994
 続きは次スレで…

   ___
 ./  ≧ \
 |::::  \ ./ |
 |::::: (● (● | グッジョブ!
 ヽ::::... .ワ....ノ    n  
 ̄ ̄   \    ( E)
フ     /ヽ ヽ_//

[初代スレ.998]
0996132人目の素数さん
垢版 |
2018/12/19(水) 06:44:55.05ID:5zoTD2o3
        ハァハァ   ∩
               ( ⌒)_   ∩_ _ グッジョブ!!
グッジョブ!! .___  //,. ノ≧ \ .i .,,E)__
     / nCr  \| / /\ ./ |/ / cos \
  _n  .|::::\ ./  |/ /(● (● | ノ\ ./ |
 ( l  |::●) ●) .| /:::... .ワ ....ノ/(● (● |   グッジョブ!!
  \ \ヽ:::::.∀   .ノ      /ヽ:::::... .▽....ノ  n
   ヽ__ ̄   ノ ヽ      |  ̄     \    ( E)
     /    /    \    ヽ フ    / ヽ ヽ_//

[第5章.991]
0997132人目の素数さん
垢版 |
2018/12/19(水) 06:45:32.33ID:5zoTD2o3
    ___
  ./  ≧ \   神降臨キタ━(゚∀゚)━!!!
  |::::  \ ./ | ハァハァ
  |::::: (● (● |
  ヽ::::... .∀....ノ /  チン ☆
 _(  ⊃  ⊃  チン ☆
 |\ ̄ ̄ ̄ ̄旦 ̄\
 | | ̄ ̄ ̄ ̄ ̄ ̄ ̄|
 \|  愛媛みかん |
    ̄ ̄ ̄ ̄ ̄ ̄ ̄
[第5章.996]
0998132人目の素数さん
垢版 |
2018/12/19(水) 06:46:20.72ID:5zoTD2o3
 "; ;ヾ; ;ヾ; ;メヾ "ゞ ;ヾ ;ゞ ;" "ゞ ; ; ; ゞ ;" "ゞ";ヾ ; ヾ ;ゞ; ;ゞ ;ゞ ;" "ゞ        /.             ヽ
 ;" "ゞ ; ; ; ゞ ; ;ヾ ; ; ヾ ;ゞ;ヾ ; ;";ヾ; ;"/" ; ;ヾ ;ヾ; ヾ ; ヾ ;ゞ; ;ゞ ;" ";ゞ ; ;ヾ      l             l
 " ;ヾ ; ;";ヾ; ;"/" ; ;ヾ ;ヾ;ヾ ; ;ヾ ; ; ヾ ;ゞ  " ;ヾ ; ;";ヾゝゝ" ;ヾゞ           ヽ            /
,." ;ヾ ; ;";ヾ; ;"/" ; ;ヾ ;ヾ;ヾ ; ;ヾ ; ; ヾ ;ゞ  " ;ヾ ; ;";ヾゝゝ" ;ヾ ; ; ヾ ;ゞ;        \        /
 ゞヾ ; ;" ; ; ;; ;"iiiiii;;;;;::::: :)_/ヽ,.ゞ:,,ヾゞヾゞ__;::/        `      `        `   ー ─ ' `
   ゞヾゞ;\\iiiiii;;;;::::: :|;:/ヾ; ;ゞ "ゝゞ ; ;`
 " ;゛ ; ;" ; ;ゞ "|iiiiii;;;;::: : |:/ ヾゞ        `         `      ` `
  `      ,|i;iiiiiii;;;;;;::: :| `    `         `     `      ` `   `
        ,|iiii;iiii;;;;:;_ _: :| ___  秋の夜長に不等式    `        `        `,
   `    |iiiiiii;;;;;;((,,,):::|/  ≧ \                    ヾ从//"
    `   |iiiiiiii;;;;ii;;;;;;;;::|::::: (● (● |           `  ゙  `    ヾ'./"
         |iiiiii;iii;;;;i;;:: ::::|ヽ::::......ワ...ノ                 ○     .||.       ,
    `   |iii;;iiiii;::;:;;;;::::::| ( つ且 ~      `              ○○   | |
  , , .,.. ,..M|M|iMii;;ii:i;;i:i;:; ゝ つつ.,.. ,...... ,.... ,,,.,.. ,.... ,,,.,.. ,..,,,,.,...,..,.,| ̄ ̄|,.,..(  ).. ,,,..,,.. ,.... ,,,.,...,.. .. ,.... ,,,.,.. ,.... ,,,

[前スレ.998]
0999132人目の素数さん
垢版 |
2018/12/19(水) 06:47:12.97ID:5zoTD2o3
_| ::|_
 ̄| ::|/|           ┌──┐
  | ::|  |     .┌──┐| ∧_∧  いいな、俺たちの誰かが殉職したら・・
/|_|  |┌──┐| ∧_∧|(・ω・` )
  |文|  | | ∧_∧(    )⊂   )
  | ̄|  | | (    )⊂   ) (_Ο Ο :::
  | ::|  | | ⊂   ) (_Ο Ο わかってる、生き延びた奴が
  | ::|/ .|_ (_Ο Ο ::::::::: :::::: 不等式を収集し、証明する !
  | ::| :::::::::::::::::::::::::::::::: 俺たちゃ死んでも仲間だぜ !!

[前スレ.999]
1000132人目の素数さん
垢版 |
2018/12/19(水) 06:48:05.42ID:5zoTD2o3
  ┏━━━┓
  ┃ Q.E.D. ┃
  ┗━┳━┛
( ゚∀゚) ノ

[前スレ.1000]
10011001
垢版 |
Over 1000Thread
このスレッドは1000を超えました。
新しいスレッドを立ててください。
life time: 461日 19時間 28分 2秒
10021002
垢版 |
Over 1000Thread
5ちゃんねるの運営はプレミアム会員の皆さまに支えられています。
運営にご協力お願いいたします。


───────────────────
《プレミアム会員の主な特典》
★ 5ちゃんねる専用ブラウザからの広告除去
★ 5ちゃんねるの過去ログを取得
★ 書き込み規制の緩和
───────────────────

会員登録には個人情報は一切必要ありません。
月300円から匿名でご購入いただけます。

▼ プレミアム会員登録はこちら ▼
https://premium.5ch.net/

▼ 浪人ログインはこちら ▼
https://login.5ch.net/login.php
レス数が1000を超えています。これ以上書き込みはできません。

ニューススポーツなんでも実況